Text
                    А. Ф. Филиппов
СБОРНИК ЗАДАЧ ПО ДИФФЕРЕНЦИАЛЬНЫМ УРАВНЕНИЯМ
Научно-издательский центр «Регулярная и хаотическая динамика»
2000
УДК 517.9
ББК 517.2
Ф 53
Филиппов А. Ф.
Сборник задач по дифференциальным уравнениям. — Ижевск: НИЦ «Регулярная и хаотическая динамика», 2000, 176 стр.
Сборник содержит материалы для упражнений по курсу дифференциальных уравнений для университетов и технических вузов с повышенной математической программой.
В настоящее издание добавлены задачи, предлагавшиеся на письменных экзаменах на механико-математическом факультете МГУ.
ISBN 5-93972-008-0
ББК 517.2
© НИЦ «Регулярная
и хаотическая динамика», 2000
СОДЕРЖАНИЕ
Предисловие ........................................ 5
§ 1.	Изоклины.
Составление дифференциального уравнения семейства кривых................................ 6
§ 2.	Уравнения с разделяющимися переменными .... 10
§ 3.	Геометрические и физические задачи............ 12
§ 4.	Однородные уравнения.......................... 17
§ 5.	Линейные уравнения первого порядка............ 20
§ 6.	Уравнения в полных дифференциалах. Интегрирующий множитель.................................... 25
§ 7.	Существование и единственность решения...... 29
§ 8.	Уравнения, не разрешенные относительно производной ........................................... 34
§ 9.	Разные уравнения первого порядка.............. 39
§ 10.	Уравнения, допускающие понижение порядка	....	44
§ 11.	Линейные уравнения с постоянными коэффициентами ............................................. 49
§ 12.	Линейные уравнения с переменными коэффициентами ............................................. 62
§ 13.	Краевые задачи............................... 71
§ 14.	Линейные системы с постоянными коэффициентами 74
§ 15.	Устойчивость ................................ 87
§ 16.	Особые точки ................................ 97
§ 17.	Фазовая плоскость ...........................104
§ 18.	Зависимость решения от начальных условий и параметров. Приближенное решение дифференциальных уравнений ....................................109
§ 19.	Нелинейные системы...........................119
§ 20.	Уравнения в частных производных	первого порядка 122
§ 21.	Существование и единственность решения.....129
4	Содержание
§ 22.	Общая теория линейных уравнений и систем .... 133
§ 23.	Линейные уравнения и системы с постоянными коэффициентами ....................................137
§ 24.	Устойчивость ...............................142
§ 25.	Фазовая плоскость ..........................144
§ 26.	Дифференцирование решения по параметру и по начальным условиям...............................148
§ 27.	Уравнения с частными производными первого порядка ...........................................149
Ответы............................................152
Ответы к добавлению...............................171
Таблицы показательной функции и логарифмов........175
ПРЕДИСЛОВИЕ
Сборник содержит задачи по курсу обыкновенных дифференциальных уравнений в соответствии с программой, принятой на механико-математическом факультете МГУ. Часть задач взята из известных задачников Н. М. Гюнтера и Р. О. Кузьмина, Г. Н. Бермана, М. Л. Краснова и Г. И. Макаренко, учебников В. В. Степанова, Г. Филипса; большинство задач составлено заново. Более трудные задачи отмечены звездочкой.
В начале каждого параграфа изложены основные методы, необходимые для решения задач этого параграфа, или даны ссылки на учебники. В ряде случаев приведены подробные решения типовых задач.
В это издание включено «Добавление» (§§21-27), содержащее задачи, предлагавшиеся на письменных экзаменах и коллоквиумах на механико-математическом факультете МГУ в 1992-1996 годах. Задачи составлены преподавателями МГУ Ю. С. Ильяшенко, В. А. Кондратьевым, В. М. Миллионщи-ковым, Н. X. Розовым, И. Н. Сергеевым, А. Ф. Филипповым.
В книге приняты условные обозначения учебников:
[1]	В. В. Степанов. Курс дифференциальных уравнений.
[2]	И. Г. Петровский. Лекции по теории обыкновенных дифференциальных уравнений.
[3]	Л. С. Понтрягин. Обыкновенные дифференциальные уравнения.
[4]	Л. Э. Эльсгольц. Дифференциальные уравнения и вариационное исчисление.
[5]	Б. П. Демидович. Лекции по математической теории устойчивости.
§1. изоклины.
СОСТАВЛЕНИЕ ДИФФЕРЕНЦИАЛЬНОГО УРАВНЕНИЯ СЕМЕЙСТВА КРИВЫХ
1.	Решение уравнения у' = f(x^ у), проходящее через точку (ж, ^/), должно иметь в этой точке производную у', равную /(ж, у), т. е. оно должно касаться прямой, наклоненной под углом а = = arctg /(ж, у) к оси Ох. Геометрическое место точек плоскости (ж, у), в которых наклон касательных к решениям уравнения у' = = /(ж, у) один и тот же, называется изоклиной. Следовательно, уравнение изоклины имеет вид /(ж, у) = fc, где к — постоянная.
Чтобы приближенно построить решения уравнения у' = = /(ж, у), можно начертить достаточное число изоклин, а затем провести решения, т. е. кривые, которые в точках пересечения с изоклинами /(ж, у) = &i, /(ж, у) = &2, ••• имеют касательные с угловыми коэффициентами соответственно fci, &2. ... Пример применения этого метода см. [1], гл. I, § 1, п. 3, или [4], гл. I, § 1.
2.	Чтобы построить дифференциальное уравнение, которому удовлетворяют кривые семейства
</>(ж, ?/, Ci, ..., Сп) = 0,	(1)
надо продифференцировать равенство (1) п раз, считая у функцией от ж, а затем из полученных уравнений и уравнения (1) исключить произвольные постоянные (71, ..., Сп-
Пример. Составить дифференциальное уравнение семейства кривых
С1Х + (з/ - С2)2 = 0.	(2)
Так как уравнение семейства содержит два параметра, дифференцируем его два раза, считая у = у(х):
Ci + 2(2/ - С2)у' = 0,	(3)
2у'2 + 2(2/ - С2)у" = 0.	(4)
§1. Изоклины	7
Исключаем Ci. Из уравнения (3) имеем Ci = — 2 (у — СъУу', подставляя это в (2), получим
-2«г/'(г/ - С2) + (у - с2)2 = о.	(5)
Исключаем С2. Из уравнения (4) имеем у — С2 = — у'2/у", подставляя это в (5), получим после упрощений дифференциальное уравнение
у + 2ху" = 0.
3.	Линии, пересекающие все кривые данного семейства под одним и тем же углом </?, называются изогональными траекториями. Углы р и а наклона траектории и кривой к оси Ох связаны соотношением Р = а ± ip. Пусть
У = f(x, у)	(6)
— дифференциальное уравнение данного семейства кривых, а
у' = fit?, у)	(7)
— уравнение семейства изогональных траекторий. Тогда tga = = /(ж, у), tg/З = /1(ж, у). Следовательно, если уравнение (6) написано и угол </? известен, то легко найти tg /3 и затем написать дифференциальное уравнение траекторий (7).
Если уравнение данного семейства кривых написано в виде
F(x, У, у') = °-	(8)
то при составлении уравнения изогональных траекторий можно обойтись без разрешения уравнения (8) относительно у'. В этом случае в (8) надо заменить у1 на tga = tg(/3 =F где tg /3 = у' — угловой коэффициент касательной к траектории.
Если же уравнение семейства кривых дано в виде ^>(ж, у, С) = = 0, то сначала нужно составить дифференциальное уравнение этого семейства и только после этого — дифференциальное уравнение траекторий.
В задачах 1—14 с помощью изоклин начертить (приближенно) решения данных уравнений.
1-	у' = у-х2.	2. 2(у + у') = ж + 3.
3.	у' =	~ 1.	4. (у2 + 1)у' = у-х.
5. уу' + х = 0.	6. ху' = 2у.
8
§1. Изоклины
7. ху' + у = 0.
9. у' = х — е27.
11.
У~3х ж+3?/ ’
13. х2 + y2yf = 1.
8- У' + У = (ж - у')3.
Ю. у(у' + ж) = 1.
12. у' = -%-. и х+у
14. (ж2 + у2)у' = 4ж.
У
15. Написать уравнение геометрического места точек (ж, у), являющихся точками максимума или минимума решений уравнения у' = /(ж, у). Как отличить точки максимума от точек минимума?
16. Написать уравнение геометрического места точек перегиба графиков решений уравнений
а) у' = у — ж2:	б) у' = х — е27:
в) ж2 + у2 у' = 1;	г) у' = f(x, у).
В задачах 17—29 составить дифференциальные уравнения данных семейств линий.
17.	у = еСж.	18. у = (х - С)3.
19.	у = Сж3.	20. у = sin(# + С).
21.	х2 + Су2 = 2у.	22. у2 + Сх = ж3.
23.	у = С(х — С)2.	24. Су = sinCa?.
25.	у = ах2 + Ьех.	26. (ж — а)2 + by2 = 1.
27.	In = ах + by.	28. у = ах3 + Ъх2 + сх.
29. х = ay2 +Ьу + с.
30.	Составить дифференциальное уравнение окружностей радиуса 1, центры которых лежат на прямой у = 2х.
31.	Составить дифференциальное уравнение парабол с осью, параллельной Оу, и касающихся одновременно прямых у = 0 и у = х.
32.	Составить дифференциальное уравнение окружностей, касающихся одновременно прямых 1/ = 0иж = 0и расположенных в первой и третьей четвертях.
33.	Составить дифференциальное уравнение всех парабол с осью, параллельной Оу, и проходящих через начало коорди
нат.
§1. Изоклины
9
34.	Составить дифференциальное уравнение всех окружностей, касающихся оси абсцисс.
В задачах 35 -36 найти системы дифференциальных уравнений, которым удовлетворяют линии данных семейств.
35.	ах + z = 6, у2 + z2 = Ь2.
36.	х2 + у2 = z2 — 26z, у = ах + b.
В задачах 37—50 составить дифференциальные уравнения1 траекторий, пересекающих линии данного семейства под
данным	углом		
37.	у = Сж4,		= 90°.
38.	У2 = х + С,		= 90°.
39.	х2 = у + Сх,		= 90°.
40.	х2 + у2 = а2,		= 45°.
41.	у = кх.		= 60°.
42.	Зж2 + у2 = С,		= 30°.
43.	?/2 = 2рж.		= 60°.
44.	г = а + cos 0,		= 90°.
45.	г = a cos2 в,		= 90°.
46.	г = a sin#,		= 45°.
47.	у = ж In ж + Сж,		= arctg2
48.	х2 + у2 = 2аж,		= 45°.
49.	.т2 + С2 = 2Су,		= 90°.
50.	у = Сх + С\		= 90°.
1 Уравнения, получаемые в задачах 37—50, могут быть решены методами, излагаемыми в дальнейших параграфах.
10	§2. Уравнения с разделяющимися переменными
§ 2. УРАВНЕНИЯ С РАЗДЕЛЯЮЩИМИСЯ ПЕРЕМЕННЫМИ
1. Уравнения с разделяющимися переменными могут быть записаны в виде
У = f(x)g(y).	(1)
а также в виде
M(x)N(y) Ож + P(x)Q(y) dy = 0.	(2)
Для решения такого уравнения надо обе его части умножить или разделить на такое выражение, чтобы в одну часть уравнения входило только ж, в другую — только ?/, и затем проинтегрировать обе части.
При делении обеих частей уравнения на выражение, содержащее неизвестные ж и у, могут быть потеряны решения, обращающие это выражение в нуль.
Пример. Решить уравнение
х2у2у' + 1 = 3/.	(3)
Приводим уравнение к виду (2):
х2 у2	= у — 1; х2у2 dy = (у — 1) dx.
Дж
Делим обе части уравнения на ж2 (у — 1):
Переменные разделены. Интегрируем обе части уравнения:
Г у2	Г dx у2	.	.	1	„
J ^1^ = ]	y+3/ + lnh-1| = -- + С.
При делении на ж2 (у — 1) могли быть потеряны решения ж = 0 и у — 1 = 0, т. е. у = 1. Очевидно, у = 1 — решение уравнения (3), а ж = 0 — нет.
2. Уравнения вида у' = f(ax + by) приводятся к уравнениям с разделяющимися переменными заменой z = ах + Ъу (или z = аж + + by + с. где с любое).
В задачах 51—65 решить данные уравнения и для каждого из них построить несколько интегральных кривых. Найти
§2. Уравнения с разделяющимися переменными
11
также решения, удовлетворяющие начальным условиям (в тех задачах, где указаны начальные условия).
51.	ху da? + (х + 1) dy = 0.
52.	+ 1 da? = ху dy.
53.	(а?2 — 1)уг + 2ху2 = 0; ?/(0) = 1.
54.	у' ctga? + у = 2; у(х) —1 при а? —> 0.
55.	у' = 3</^; j/(2) =0.
56.	ху’ + у = у2; у(1) = 0.5.
57.	2х2уу' + у2 = 2.	58.	у' — ху2 = 2ху.
59.		60.	z' = юж+г.
61.	^+t = l.	62.	у' = cos(y - ж).
63.	у' — у = 2х —	3.	
64. (ж + 2у)у' = 1; у(0) = -1.
65.	у' = ^/4а? + 2т/ — 1.
В задачах 66—67 найти решения уравнений, удовлетворяющие указанным условиям при а? +оо.
66.	х2у' — cos 2у = 1; 2/(+оо) = 9тг/4.
67.	3>у2у’ + 16а? = 2а??/3; у(х) ограничено при а? +оо.
68.	Найти ортогональные траектории к линиям следующих семейств: а) у = Са?2: б) у = Сеж: в) Сх2 + у2 = 1.
В задачах 69* и 70* переменные разделяются, но получаемые интегралы не могут быть выражены через элементарные функции. Однако, исследовав их сходимость, можно дать ответ на поставленные вопросы.
69*	. Показать, что каждая интегральная кривая уравнения у’ = ^/^4^ имеет две горизонтальные асимптоты.
70*	. Исследовать поведение интегральных кривых уравнения у' =	в окрестности начала координат. Пока-
зать, что из каждой точки границы первого координатного угла выходит одна интегральная кривая, проходящая внутри этого угла.
12
§3. Геометрические и физические задачи
§ 3. ГЕОМЕТРИЧЕСКИЕ И ФИЗИЧЕСКИЕ ЗАДАЧИ1
1. Чтобы решить приведенные ниже геометрические задачи, надо построить чертеж, обозначить искомую кривую через у = = у(х) (если задача решается в прямоугольных координатах) и выразить все упоминаемые в задаче величины через ж, у и у . Тогда данное в условии задачи соотношение превращается в дифференциальное уравнение, из которого можно найти искомую функцию у(х).
2. В физических задачах надо прежде всего решить, какую из величин взять за независимое переменное, а какую — за искомую функцию. Затем надо выразить, на сколько изменится искомая функция ?/, когда независимое переменное х получит приращение Дж, т. е. выразить разность у(х + Дж) — у(х) через величины, о которых говорится в задаче. Разделив эту разность на Дж и перейдя к пределу при Дж 0, получим дифференциальное уравнение, из которого можно найти искомую функцию. В большинстве задач содержатся условия, с помощью которых можно определить значения постоянных, входящих в общее решение дифференциального уравнения. Иногда дифференциальное уравнение можно составить более простым путем, воспользовавшись физическим смыслом производной (если независимое переменное — время t, то есть скорость изменения величины у).
В некоторых задачах при составлении уравнения следует использовать физические законы, сформулированные в тексте перед задачей (или перед группой задач).
Пример. В сосуд, содержащий 10 л воды, непрерывно поступает со скоростью 2 л в минуту раствор, в каждом литре которого содержится 0,3 кг соли. Поступающий в сосуд раствор перемешивается с водой, и смесь вытекает из сосуда с той же скоростью. Сколько соли будет в сосуде через 5 минут?
Решение. Примем за независимое переменное время t, а за искомую функцию y(t) — количество соли в сосуде через t минут после начала опыта. Найдем, на сколько изменится количество соли за промежуток времени от момента t до момента t + At. В одну минуту поступает 2 л раствора, а в At минут — 2At литров; в этих
хВсе задачи этого параграфа сводятся к уравнениям с разделяющимися переменными. Задачи, приводящиеся к уравнениям других типов, можно найти в соответствующих параграфах. Необходимые для решения задач значения показательной функции и логарифмов можно брать из таблицы в конце задачника.
§3. Геометрические и физические задачи
13
2 At литрах содержится 0,3-2 At = 0,6At кг соли. С другой стороны, за время At из сосуда вытекает 2At литров раствора. В момент t во всем сосуде (10 л) содержится y(t) кг соли, следовательно, в 2At литрах вытекающего раствора содержалось бы 0,2At • y(t) кг соли, если бы за время At содержание соли в сосуде не менялось. Но так как оно за это время меняется на величину, бесконечно малую при At —> 0, то в вытекающих 2At литрах содержится 0,2At(?/(t) + a) кг соли, где а —> 0 при At 0.
Итак, в растворе, втекающем за промежуток времени (t, t + + At), содержится 0,6At кг соли, а в вытекающем — 0,2At • (y(t) + + q) кг. Приращение количества соли за это время y(t + At) — y(t) равно разности найденных величин, т. е.
y(t + At) — y(t) = 0,6At — 0,2At • (?/(t) + q).
Разделим на At и перейдем к пределу при At —> 0. В левой части получится производная yf(t)j а в правой получим 0,6 — 0,2?/(t), так как q —> 0 при At 0.
Итак, имеем дифференциальное уравнение у'(Г) = 0,6 —0,2?/(t). Решая его, получим
y(t) = 3 - Се-0'24.	(1)
Так как при t = 0 соли в сосуде не было, то ?/(0) = 0. Полагая в (1) t = 0, найдем ?/(0) = 3 — С; 0 = 3 — С; С = 3. Подставляя это значение С в (1), получим у(Г) = 3 — Зе-0’2*. При t = 5 в сосуде будет
2/(5) = 3 - Зе-0’2'5 = 3 - Зе-1 « 1,9 кг соли.
71.	Найти кривые, для которых площадь треугольника, образованного касательной, ординатой точки касания и осью абсцисс, есть величина постоянная, равная а2.
72.	Найти кривые, для которых сумма катетов треугольника, построенного как в предыдущей задаче, есть величина постоянная, равная Ь.
73.	Найти кривые, обладающие следующим свойством: отрезок оси абсцисс, отсекаемый касательной и нормалью, проведенными из произвольной точки кривой, равен 2а.
74.	Найти кривые, у которых точка пересечения любой касательной с осью абсцисс имеет абсциссу, вдвое меньшую абсциссы точки касания.
75.	Найти кривые, обладающие следующим свойством: если через любую точку кривой провести прямые, параллель
14	§3. Геометрические и физические задачи
ные осям координат, до встречи с этими осями, то площадь полученного прямоугольника делится кривой в отношении 1:2.
76.	Найти кривые, касательные к которым в любой точке образуют равные углы с полярным радиусом и полярной осью.
В задачах 77—79 считать, что втекающий газ (или жидкость) вследствие перемешивания распределяется по всему объему вместилища равномерно.
77.	Сосуд объемом в 20 л содержит воздух (80% азота и 20% кислорода). В сосуд втекает 0,1 л азота в секунду, который непрерывно перемешивается, и вытекает такое же количество смеси. Через сколько времени в сосуде будет 99% азота?
78.	В баке находится 100 л раствора, содержащего 10 кг соли. В бак непрерывно подается вода (5 л в минуту), которая перемешивается с имеющимся раствором. Смесь вытекает с той же скоростью. Сколько соли в баке останется через час?
79.	В воздухе комнаты объемом 200 ж3 содержится 0,15% углекислого газа (СО2). Вентилятор подает в минуту 20 ж3 воздуха, содержащего 0,04% СО2. Через какое время количество углекислого газа в воздухе комнаты уменьшится втрое?
В задачах 80—82 принять, что скорость остывания (или нагревания) тела пропорциональна разности температур тела и окружающей среды.
80.	Тело охладилось за 10 мин от 100° до 60°. Температура окружающего воздуха поддерживается равной 20°. Когда тело остынет до 25°?
81.	В сосуд, содержащий 1 кг воды при температуре 20°, опущен алюминиевый предмет с массой 0,5 кг, удельной теплоемкостью 0,2 и температурой 75°. Через минуту вода нагрелась на 2°. Когда температура воды и предмета будет отличаться одна от другой на 1°? Потерями тепла на нагревание сосуда и прочими пренебречь.
82.	Кусок металла с температурой а градусов помещен в печь, температура которой в течение часа равномерно повышается от а градусов до Ъ градусов. При разности температур
§3. Геометрические и физические задачи
15
печи и металла в Т градусов металл нагревается со скоростью кТ градусов в минуту. Найти температуру металла через час.
83.	Лодка замедляет свое движение под действием сопротивления воды, которое пропорционально скорости лодки. Начальная скорость лодки 1,5 м/сек, через 4 сек скорость ее 1 м/сек. Когда скорость уменьшится до 1 см/секЧ Какой путь может пройти лодка до остановки?
В задачах 84—86 использовать закон радиоактивного распада: количество радиоактивного вещества, распадающегося за единицу времени, пропорционально количеству этого вещества, имеющемуся в рассматриваемый момент.
84.	За 30 дней распалось 50% первоначального количества радиоактивного вещества. Через сколько времени останется 1% от первоначального количества?
85.	Согласно опытам, в течение года из каждого грамма радия распадается 0,44 мг. Через сколько лет распадется половина имеющегося количества радия?
86.	В исследованном куске горной породы содержится 100 мг урана и 14 мг уранового свинца. Известно, что уран распадается наполовину за 4.5-109 лет и что при полном распаде 238 г урана образуется 206 г уранового свинца. Определить возраст горной породы. Считать, что в момент образования горная порода не содержала свинца, и пренебречь наличием промежуточных радиоактивных продуктов между ураном и свинцом (так как они распадаются намного быстрее урана).
87.	Количество света, поглощаемое слоем воды малой толщины, пропорционально количеству падающего на него света и толщине слоя. Слой воды толщиной 35 см поглощает половину падающего на него света. Какую часть света поглотит слой толщиной в 2 ж?
Для составления дифференциального уравнения в задачах 88—90 за неизвестную функцию удобнее взять скорость. Ускорение силы тяжести считать равным 10 м/сек2.
88.	Парашютист прыгнул с высоты 1,5 км, а раскрыл парашют на высоте 0,5 км. Сколько времени он падал до раскрытия парашюта? Известно, что предельная скорость падения человека в воздухе нормальной плотности составляет
16
§3. Геометрические и физические задачи
50 м/сек. Изменением плотности с высотой пренебречь. Сопротивление воздуха пропорционально квадрату скорости.
89.	Футбольный мяч весом 0,4 кГ брошен вверх со скоростью 20 м/сек. Сопротивление воздуха пропорционально квадрату скорости и равно 0,48 Г при скорости 1 м/сек. Вычислить время подъема мяча и наибольшую высоту подъема. Как изменятся эти результаты, если пренебречь сопротивлением воздуха?
90.	Вычислить время падения мяча с высоты 16,3 м без начальной скорости с учетом сопротивления воздуха (см. задачу 89). Найти скорость в конце падения.
В задачах 91—95 принять, что жидкость из сосуда вытекает со скоростью, равной 0,6y/2gH, где g = 10 м/сек2 — ускорение силы тяжести, h — высота уровня воды над отверстием.
91.	За какое время вытечет вся вода из цилиндрического бака диаметром 2R = 1,8 м и высотой Н = 2,45 м через отверстие в дне диаметром 2г = 6 см2 Ось цилиндра вертикальна.
92.	Решить предыдущую задачу в предположении, что ось цилиндра расположена горизонтально, а отверстие находится в самой нижней части цилиндра.
93.	Цилиндрический бак поставлен вертикально и имеет отверстие в дне. Половина воды из полного бака вытекает за 5 минут. За какое время вытечет вся вода?
94.	Воронка имеет форму конуса радиуса R = 6 см и высоты Н = 10 см, обращенного вершиной вниз. За какое время вытечет вся вода из воронки через круглое отверстие диаметра 0,5 см, сделанное в вершине конуса?
95.	В прямоугольный бак размером 60 см х 75 см и высотой 80 см поступает 1,8 л воды в секунду. В дне имеется отверстие площадью 2,5 см2. За какое время наполнится бак? Сравнить результат с временем наполнения такого бака без отверстия в дне.
96.	Резиновый шнур длиной в 1 м под действием силы / кГ удлиняется на kf метров. На сколько удлинится такой же шнур длины I и веса Р под действием своего веса, если его подвесить за один конец?
§4. Однородные уравнения
17
97.	Найти атмосферное давление на высоте h. если на поверхности земли давление равно 1 кГ/см2 и плотность воздуха 0,0012 г/см3. Использовать закон Бойля—Мариотта, в силу которого плотность пропорциональна давлению (т. е. пренебречь изменением температуры воздуха с высотой).
98.	Для остановки речных судов у пристани с них бросают канат, который наматывают на столб, стоящий на пристани. Какая сила будет тормозить судно, если канат делает три витка вокруг столба, коэффициент трения каната о столб равен 1/3, и рабочий на пристани тянет за свободный конец каната с силой 10 кГЧ
99.	В закрытом помещении объемом v м3 находится открытый сосуд с водой. Скорость испарения воды пропорциональна разности между количеством Qi водяного пара, насыщающего 1 м3 воздуха при данной температуре, и количеством q водяного пара, имеющемся в 1 л3 воздуха в рассматриваемый момент (считаем, что температура воздуха и воды, а также величина площади, с которой происходит испарение, остаются неизменными). В начальный момент в сосуде было то грамм воды, а в 1 м3 воздуха qo грамм пара. Сколько воды останется в сосуде через промежуток времени t?
100.	Масса ракеты с полным запасом топлива равна М, без топлива т, скорость истечения продуктов горения из ракеты равна с, начальная скорость ракеты равна нулю. Найти скорость ракеты после сгорания топлива, пренебрегая силой тяжести и сопротивлением воздуха (формула Циолковского).
§ 4.	ОДНОРОДНЫЕ УРАВНЕНИЯ
1.	Однородные уравнения могут быть записаны в виде у' = = f (^), а также в виде М(ж, у) dx + ЛЦж, у) dy = 0, где М(ж, у) и 7У(ж, у) — однородные функции одной и той же степени1. Чтобы решить однородное уравнение, можно сделать замену у = tx. после чего получается уравнение с разделяющимися переменными.
Пример. Решить уравнение х dy = (ж + у) dx.
хФункция М(ж, у) называется однородной функцией степени п, если для всех к > 0 имеем М(кх, ку) = кпМ(х, у).
18
§ 4. Однородные уравнения
Это уравнение — однородное. Полагаем у = tx. Тогда dy = = х dt 4- t dx. Подставляя в уравнение, получим
х(х dt + t Пж) = (х + tx) (1ж: ж dt = (1ж.
Решаем полученное уравнение с разделяющимися переменными
dt = —; t = In |ж| + С.
х
Возвращаясь к старому переменному у. получим у = ж(1п|ж| + С). Кроме того, имеется решение ж = 0, которое было потеряно при делении на ж.
2.	Уравнение вида у' = f (^ах+Ьу+с1) приводится к однородному с помощью переноса начала координат в точку пересечения прямых ах + by + с = 0 и а±х + biy + ci = 0. Если же эти прямые не пересекаются, то еиж + biy = к(ах + by), следовательно, уравнение имеет вид у' = F(ax + by) и приводится к уравнению с разделяющимися переменными заменой z = а,х + by (или z = а,х + by + с), см. § 2. п. 2.
3.	Некоторые уравнения можно привести к однородным заменой у = zm. Число т обычно заранее не известно. Чтобы его найти, надо в уравнении сделать замену у = zm. Требуя, чтобы уравнение было однородным, найдем число т, если это возможно. Если же этого сделать нельзя, то уравнение не приводится к однородному этим способом.
Пример. Дано уравнение 2хАуу +уА = 4ж6. После замены у = = zm уравнение примет вид 2тж4z2m-1 z + z4m = 4ж6. Это уравнение будет однородным в том случае, когда степени всех его членов равны между собой, т. е. 4 + (2m — 1) = 4m = 6. Эти равенства удовлетворяются одновременно, если m = 3/2. Следовательно, уравнение можно привести к однородному заменой у =	.
Решить уравнения 101—129.
101.	(ж + 2у) dx — xdy = 0.
102.	(ж — у) (1ж + (ж + у) dy = 0.
103.	(у2 — 2ху) (1ж + ж2 dy = 0.
104.	2х3уг = у(2х2 — у2).
105.	у2 + х2уг = хуу'.
106.	(ж2 + у2)у' — 2ху.
§ 4. Однородные уравнения
19
107.	ху' - у = xtg%.
108.	ху' = у — х&!х.
109.	ху1 — у = (ж + у) In
110.	ху' = у cos In
111.	(у + y/ху) dx = х dy.
112.	ху' = у/х2 — у2 + у.
113.	(2х — dy + 6) da? + (х + у — 3) dy = 0.
114. (: 115. х	2х + у + 1) ёж — (4х + 2у — 3) dy = 0. -у-1 + (у~х + W = 0-
116. (: 117. G	г + 4у)у' = 2х + Зу - 5. е/ + 2) dx = (2ж + у — 4) dy.
118. у	1 = 2( У_+2 V \х + у — 1J
119. G	/+1)1п "±4 = ^. 4	7 ж+3	ж+3
120. у	, у + 2	у — 2х х+1 ь ж + 1
121. х	3(у' - ж) = у2.
122.	2 123.	2 124.	у 125.	2 126.	у 127.	2 128.	| 129.	2	с II о	<	• А 73	1	II •	1 н 555 >> & + +	> । А 1	«о	1—1 +	g5 > ks,	I « ъ	ls II > + S*	<-> Il ।	11 11 + X « i J1 A >	? Il‘ a g +
130.	Найти траектории, пересекающие кривые данного семейства под углом в 45°, причем этот угол от касательной
20
§5. Линейные уравнения первого порядка
к кривой до касательной к траектории отсчитывается в отрицательном направлении.
а) у = ж1пСж; б) (ж — З?/)4 = Сху^.
131.	Найти кривую, у которой точка пересечения любой касательной с осью абсцисс одинаково удалена от точки касания и от начала координат.
132.	Найти кривую, у которой расстояние любой касательной от начала координат равно абсциссе точки касания.
133.	При каких а и (3 уравнение у1 = ах(у+ +Ьу& приводится к однородному с помощью замены у = zml
134*. Пусть — корень уравнения f(k) = к. Показать, что:
1) если f'(ko) < 1, то ни одно решение уравнения у' = = f(y/x} не касается прямой у = к^х в начале координат;
2) если /'(fc0) > 1, то этой прямой касается бесконечно много решений.
135.	Начертить приближенно интегральные кривые следующих уравнений (не решая уравнений):
_ у(2у-Ж)
’ у ~ х2 ’
= 2</3-Жу
,У 2х2у-х^
б) у'=2-^3 ху
г*) ху' = у +
Указание. Тангенс угла между лучом у = кх и пересекающей его интегральной кривой уравнения у' = f(y/x) равен (f (fe) — к) / (1 + kf(k\) (почему?). Для приближенного построения интегральных кривых надо исследовать знак этой дроби в зависимости от к.
§ 5.	ЛИНЕЙНЫЕ УРАВНЕНИЯ ПЕРВОГО ПОРЯДКА
1.	Уравнение
у' + а(х)у = b(x)	(1)
называется линейным. Чтобы его решить, надо сначала решить уравнение
у' + а(х)у = 0
(2)
§5. Линейные уравнения первого порядка	21
(это делается путем разделения переменных, см. § 2) и в общем решении последнего заменить произвольную постоянную С на неизвестную функцию С(ж). Затем выражение, полученное для у, подставить в уравнение (1) и найти функцию С(ж).
2.	Некоторые уравнения становятся линейными, если поменять местами искомую функцию и независимое переменное. Например, уравнение у = (2ж + я/3)?/, в котором у является функцией от ж, — нелинейное. Запишем его в дифференциалах: ydx — (2ж + + у3) dy = 0. Так как в это уравнение ж и da? входят линейно, то уравнение будет линейным, если ж считать искомой функцией, а у — независимым переменным. Это уравнение может быть записано в виде
dж 2	2
------ж = у &У У
и решается аналогично уравнению (1).
3.	Чтобы решить уравнение Бернулли, т. е. уравнение
у' + а{х)у = Ь(х)уп, (я 1),
надо обе его части разделить на уп и сделать замену 1/уп-1 = z. После замены получается линейное уравнение, которое можно решить изложенным выше способом. (Пример см. в [1], гл. I, §4, п. 2, пример 10.)
4.	Уравнение Риккати, т. е. уравнение
у' + а(х)у + Ь(х)у2 = с(ж),
в общем случае не решается в квадратурах. Если же известно одно частное решение ?/1(ж), то заменой у = ?/1(ж) + z уравнение Риккати сводится к уравнению Бернулли и таким образом может быть решено в квадратурах.
Иногда частное решение удается подобрать, исходя из вида свободного члена уравнения (члена, не содержащего у). Например, для уравнения у'+у2 = ж2 —2ж в левой части будут члены, подобные членам правой части, если взять у = ах+b. Подставляя в уравнение и приравнивая коэффициенты при подобных членах, найдем а и b (если частное решение указанного вида существует, что вовсе не всегда бывает). Другой пример: для уравнения у' + 2у2 = 6/ж2 те же рассуждения побуждают нас искать частное решение в виде у = а/х. Подставляя у = а/х в уравнение, найдем постоянную а.
Решить уравнения 136—160.
136.	ху1 — 2у = 2ж4.
22
§5. Линейные уравнения первого порядка
137.	(2ж + 1)у' = 4ж + 2у.
138.	у' + ytg х = sec х.
139.	(ху + еж) da? — х dy = 0.
140.	x2yf + ху + 1 = 0.
141.	у = х(у' — X cos а?).
142.	2 a? (a?2 + y)dx = dy.
143.	(xyf — 1) Ina? = 2y.
144.	xyf + (a? + l)y = За?2е-Ж.
145.	(a? + y2) dy = у da?.
146.	(2e^ - x}y' = 1.
147.	(sin2 у + a? ctg y\y’ = 1.
148.	(2a? + y) dy = у da? + 4 In у dy.
150.	(1 - 2xy)yf = y(y - 1).
151.	y’ + 2y = y2ex.
152.	(ж + 1)(t/' + 7/2) = -y.
153.	y' = y* cos a? + ytgx.
154.	xy2y’ = a?2 + y3.
155.	xydy = (y2 + a?) da?.
156.	xyf — 2x2y/y = dy.
157.	ху’ + 2y + а?5?/3еж = 0.
158.	2y' — - = У у	ж2 —1
159.	y'x3 sin у = xy' — 2y.
160.	(2x2y In у — x)y' = y.
С помощью замены переменных или дифференцирования привести уравнения 161—166 к линейным и решить их.
§5. Линейные уравнения первого порядка	23
161.	х da? = (ж2 — 2у + 1) dy.
162.	(ж + 1)(уу' - 1) = у2.
163.	- у') = 2.
164.	(ж2 — 1)т/' sin т/ + 2a?cos?/ = 2х — 2ж3.
165.	у(х) = / y(t) dt + х + 1. о
166.	f(x — t)y(t) dt = 2x + f y(t) dt. о	о
В задачах 167—171, найдя путем подбора частное решение, привести данные уравнения Риккати к уравнениям Бернулли и решить их.
167.	х2у' + ху + х2у2 = 4.
168.	3>у' + у2 +	= 0.
169.	ху' — (2х + 1)у + у2 = —х2.
170.	у’ — 2ху + у2 = 5 — х2.
171.	у' + 2уех — у2 = е2ж + еж.
172.	Найти траектории, ортогональные к линиям семейства у2 = Сеж + х + 1.
173.	Найти кривые, у которых площадь трапеции, ограниченной осями координат, касательной и ординатой точки касания, есть величина постоянная, равная За2.
174.	Найти кривые, у которых площадь треугольника, ограниченного касательной, осью абсцисс и отрезком от начала координат до точки касания, есть величина постоянная, равная а2.
175.	В баке находится 100 л раствора, содержащего 10 кг соли. В бак втекает 5 л воды в минуту, а смесь с той же скоростью переливается в другой 100-литровый бак, первоначально наполненный чистой водой. Избыток жидкости из него выливается. Когда количество соли во втором баке будет наибольшим? Чему оно равно?
176.	За время А/ (где At очень мало и выражено в долях года) из каждого грамма радия распадается 0,00044 At грамма
24	§5. Линейные уравнения первого порядка
и образуется 0,00043 At грамма радона. Из каждого грамма радона за время At распадается 70 At грамма. В начале опыта имелось некоторое количество ж0 чистого радия. Когда количество образовавшегося и еще не распавшегося радона будет наибольшим?
177.	Даны два различных решения у\ и у? линейного уравнения первого порядка. Выразить через них общее решение этого уравнения.
178.	Найти то решение уравнения
у1 sin2# = 2(у + cos ж),
которое остается ограниченным при х —> тг/2.
179*. Пусть в уравнении ху1 + ау = /(ж) имеем а = = const > 0, /(ж) —> b при х —> 0. Показать, что только одно решение уравнения остается ограниченным при ж —> 0, и найти предел этого решения при —> 0.
180*. Пусть в уравнении предыдущей задачи а = = const < 0, /(ж) b при —> 0. Показать, что все решения этого уравнения имеют один и тот же конечный предел при х —> 0. Найти этот предел.
В задачах 181—183 искомое решение выражается через интеграл с бесконечным пределом.
181*. Показать, что уравнение + х = где |/(t)| М при —оо < t < -hoc, имеет одно решение, ограниченное при —оо < t < +оо. Найти это решение. Показать, что найденное решение периодическое, если функция /(t) периодическая.
182*. Показать, что только одно решение уравнения ху’ — — (2ж2Ч-1)т/ = х2 стремится к конечному пределу при х —> +оо, и найти этот предел. Выразить это решение через интеграл.
183*. Найти периодическое решение уравнения
у' = 2у cos2 х — sin х.
184*. Пусть в уравнении + a(t)x = /(t) a(t) с > О, /(t) -> 0 при t —> +оо. Доказать, что каждое решение этого уравнения стремится к нулю при t —> +оо.
§6. Уравнения в полных дифференциалах
25
185*. Пусть в уравнении предыдущей задачи имеем a(t) с> 0 и пусть a?o(t) — решение с начальным условием жо(0) = Ь. Показать, что для любого е > 0 существует такое S > 0, что если изменить функцию f(t) и число b меньше, чем на S (т. е. заменить их на такую функцию /i(t) и число Ь1? что |/i(t) — f(t)\ < <5, |bi — b\ < <5), то решение xo(t) изменится при t 0 меньше, чем на e. Это свойство решения называется устойчивостью по постоянно действующим возмущениям.
§6. УРАВНЕНИЯ
В ПОЛНЫХ ДИФФЕРЕНЦИАЛАХ. ИНТЕГРИРУЮЩИЙ МНОЖИТЕЛЬ
1.	Уравнение
М(ж, у) dx + 7У(ж. у) dy = О
(1)
называется уравнением в полных дифференциалах, если его левая часть является полным дифференциалом некоторой функции Р(ж, у). Это имеет место, если	Чтобы решить урав-
(J у	(J JU
нение (1), надо найти функцию Р(ж, ?/), от которой полный дифференциал dF(x, у) = F'x dx + Fy dy равен левой части уравнения (1). Тогда общее решение уравнения (1) можно написать в виде Р(ж, у) = С, где С — произвольная постоянная.
Пример. Решить уравнение
(2ж + Зж2?/) dx + (ж3 — З?/2) dy = 0.	(2)
Так как ^-(2ж + Зж2?/) = Зж2, ^-(ж3 —З?/2) = Зж2, то уравнение (2)
является уравнением в полных дифференциалах. Найдем функцию Р(ж, у), полный дифференциал которой dF = Fx dx + Fy dy был бы равен левой части уравнения (2), т. е. такую функцию F. что
Fx = 2ж + Зж2 у,	Fy = ж3 — Зу2.	(3)
Интегрируем по ж первое из уравнений (3), считая у постоянным; при этом вместо постоянной интегрирования надо поставить </?(?/) — неизвестную функцию от у :
F = J (2ж + Зж2?/) dx = ж2 + ж3?/ + </>(?/).
26	§6. Уравнения в полных дифференциалах
Подставляя это выражение для F во второе из уравнений (3), найдем р(у):
(ж2 + х3у + ф(у))'у = х3 - Зу2-, (р'- Зу2-, <р(у)= - у3+ const.
Следовательно, можно взять Р(ж, у) = х2 + х3у — у3, и общее решение уравнения (2) будет иметь вид
х + х у — у = С.
2.	Интегрирующим множителем для уравнения
М(ж, у) dx +	у) dy = 0	(4)
называется такая функция т(ж, у) 0, после умножения на которую уравнение (4) превращается в уравнение в полных дифференциалах. Если функции М и N в уравнении (4) имеют непрерывные частные производные и не обращаются в нуль одновременно, то интегрирующий множитель существует. Однако нет общего метода для его отыскания (когда общее решение уравнения (4) неизвестно).
В некоторых случаях интегрирующий множитель можно найти с помощью приемов, изложенных в [1]. гл. II, § 3, п. 3 или в [4], гл. 1, § 5. Для решения некоторых уравнений можно применять метод выделения полных дифференциалов, используя известные формулы:
d(xy) = у dx + х dy,	d(y2) = 2ydy,
, (х\ ydx — xdy	4 dy
d -	=	d(ln?/) = и т. n.
\yj У2	У
Пример. Решить уравнение
у dx — (4ж2?/ + х) dy = 0.	(5)
Сначала выделяем группу членов, представляющую собой полный дифференциал. Так как ydx — xdy = —х2 d(y/x), то, деля уравнение (5) на —ж2, имеем
d Ц) + 4у dy = 0, d (5) + d(2?/2) = °-
Это — уравнение в полных дифференциалах. Интегрируя непосредственно (приводить к виду (1) не нужно), получаем решение
+ 2у2 = С.
X
§6. Уравнения в полных дифференциалах	27
Кроме того, при делении на —ж2 было потеряно решение х = 0.
Замечание. Так как после деления уравнения (5) на —ж2, т. е. умножения на — 1/ж2, получилось уравнение в полных дифференциалах, то интегрирующий множитель для уравнения (5) равен — 1/ж2.
3.	Если в уравнении (4) можно выделить полный дифференциал некоторой функции у?(ж, у), то иногда уравнение упрощается, если от переменных (ж, у) перейти к переменным (ж, z) или (у, z), где z = </?(ж, у).
Примеры. 1) Решить уравнение у Пж — (ж3?/ + ж) dy = 0.
Выделив полный дифференциал как в предыдущем примере, получим
d Q) + xydy = 0.
Перейдя к переменным z = у/х и t/, получим уравнение
Az + — Ay = О,
Z
которое легко решается.
2)	Решить уравнение (ху + t/4) dx + (ж2 — ж?/3) dy = 0.
Сгруппируем члены так, чтобы выделить полные дифференциалы
х(у dx + ж dy) + y3(ydx — ж dy) = 0, ж d(xy) + у5 d ( — ) =0. \yj
Разделив на ж и сделав замену ху = ?/, ж/у = получим уравнение и2
dzz Ч—- dt; = 0, которое легко решается. v
В задачах 186—194 проверить, что данные уравнения являются уравнениями в полных дифференциалах, и решить их.
186.	2xydx + (ж2 — у2) dy = 0.
187.	(2 — 9xy2)xdx + (4t/2 — 6х3)у dy = 0.
188.	е~у da? — (2у + хеГу) dy = 0.
189.	dx + (у3 + In ж) dy = 0.
i пл За?2 + У2 я 2ж3 + 5у ,
190.		(1ж-----------—- dy = 0.
У	У3
28	§ 6. Уравнения в полных дифференциалах
191.	2х (1 + у/х2 — yj da? — \/х2 — у dy = 0.
192.	(1 + ?/2 sin 2а?) da? — 2у cos2 a? dy = 0.
193.	За?2(1 + In?/) da? = ^2у — dy,
,	( г	1	(а?2+ 1) cos V _
194.	+ 2 da? + ------dy = 0.
\ sin у J	cos 2у — 1
Решить уравнения 195—220, найдя каким-либо способом интегрирующий множитель или сделав замену переменных.
195.	(а?2 + у2 + х) da? + у dy = 0.
196.	(а?2 + у2 + у) da? — a? dy = 0.
197.	у dy = (a? dy + у da?) ^/1 + т/2.
198.	ху2 {ху' + у) = 1.
199.	у2 da? - (ху + a?3) dy = 0.
200.	(у — — | da? + — = 0. \ х) у
201.	(а?2 -I- 3In?/)?/da? = a?dy.
202.	у2 da? + (ху + tg ху) dy = 0.
203.	у(х + у) da? + (ху + 1) dy = 0.
204.	у(у2 + 1) da? + х(у2 — а? + 1) dy = 0.
205.	(а?2 + 2а? + у) da? = (а? — За?2?/) dy.
206.	у da? — a? dy = 2а?3 tg | da?.
207.	у2 da? + (еж — у) dy = 0.
208.	ху da? = (?/3 + х2у + a?2) dy.
209.	х2у(у da? + a? dy) = 2у da? + a? dy.
210.	(а?2 — у2 + у) da? + х(2у — 1) dy = 0.
211.	(2а?2?/2 + у) da? + (а?3?/ — a?) dy = 0.
212.	(2а?2?/3 — l)?/da? + (4а?2?/3 — l)xdy = 0.
§ 7. Существование и единственность решения
29
213.	у(х + у2) <1ж + х2(у - 1) dy = 0.
214.	(ж2 — sin2 у) da? + х sin 2у dy = 0.
215.	a?(ln?/ + 2 Ina? — 1) dy = 2у da?.
216.	(а?2 + 1) (2а? da? + cos у dy) = 2а? sin у da?.
217.	(2а?3?/2 — у) da? + (2а?2?/3 — a?) dy = 0.
218.	а?2?/3 + у + (а?3?/2 — х)у' = 0.
219.	(а?2 — у) da? + а?(?/ + 1) dy = 0.
220.	y2(ydx — 2а?dy) — a?3(a?d?/ — 2?/da?).
§ 7.	СУЩЕСТВОВАНИЕ И ЕДИНСТВЕННОСТЬ РЕШЕНИЯ
1.	Теорема существования и единственности решения уравнения
?/' = /(ж, у)	(1)
с начальным условием ?/(®о) =?/о-
Пусть в замкнутой области R (|ж — жо| а, \у — ?/о| Ь) функции f и fy непрерывны1. Тогда на некотором отрезке xq — — d^x^XQ-\-d существует единственное решение уравнения (1), удовлетворяющее начальному условию у(хо) = у о.
При этом можно взять d = min {а; где а и b указаны выше, ат — любое такое, что \f | т в R.
Последовательные приближения, определяемые формулами
уо(х) = уо, ук(х) = уо + у f(s, 2/fc_i(s))ds, k = 1, 2, ...
«О
равномерно сходятся к решению на указанном отрезке.
Замечание. Для существования решения достаточно только непрерывности f (ж, у) в области R. но при этом решение может не быть единственным.
Требование непрерывности f'(y) можно заменить требованием ее ограниченности или условием Липшица: |/(ж,?/1) — /(ж,?/2)|	&|?/1 — 2/2Ь
k = const.
30
§ 7. Существование и единственность решения
2.	Система уравнений
' У1 = fi(x. У1, •••• Уп), < ................................... (2)
. Уп = fn(,X, у!, уп)
в векторных обозначениях записывается так:
у' =	у),	(3)
где у = (г/1,	у„)и/ = (Д, Д) — векторы. Непре-
рывность вектор-функции f означает непрерывность всех функций /1,	а вместо рассматривается матрица из частных
производных г, к = 1, . ..,п.
Теорема существования и единственности решения и все утверждения п. 1 остаются справедливыми и для системы, записанной в виде (3). При этом \у\ означает длину вектора у: \у\ = = д/?/1 + ••• + yl-
3.	Теорема существования и единственности решения для уравнения п-г о порядка
</(п) =/(», у, у',	?/(’’_1)).	(4)
Пусть в области D функция f и ее частные производные первого порядка по у, у', ..., у^п~^ непрерывны, и точка (жо, ?/о, Уъ. ...,	лежит внутри D. Тогда при начальных
условиях
у(х0) = уо, у'(х0) = у'о, у(п~1\х0') = у^-1'1
уравнение (4) имеет единственное решение.
Уравнение (4) можно свести к системе вида (2), если ввести новые неизвестные функции по формулам у = у±, у' = у2, у" = = ?/з, ..., у{п~1} = уп- Тогда уравнение (4) сводится к системе
У1 = Я/2, У2 = Х/3, . . . , Уп-Х = Уп. Уп = yr, . . . , Уп).
которая является частным случаем системы (2) и к которой применимы все утверждения п. 2.
4.	Продолжение решений. Во многих случаях решение уравнения (1) или системы (2) существует не только на отрезке, указанном в п. 1, но и на большем отрезке.
Если уравнение (1) или система (2) удовлетворяет условиям теоремы существования в замкнутой ограниченной области, то
§ 7.	Существование и единственность решения 31 всякое решение можно продолжить до выхода на границу этой области.
Если правая часть уравнения (1) или системы (3) в области а < х < /3, \у\ < оо (а и /3 могут быть конечными или бесконечными) непрерывна и удовлетворяет неравенству
|/(х, г/)| С a(x)|i/| + b(x),
и функции а(х) и Ь(х) непрерывны, то всякое решение можно продолжить на весь интервал а < х < (3.
221.	Построить последовательные приближения уц, у1у у% к решению данного уравнения с данными начальными условиями:
а)	у' = х - у2. у(0) = 0.
б)	у' = У2 + Зж2 - 1, у{1) = 1.
в)	У' = У + ей-1, у(0) = 1.
г)	у' = 1 + х sin 7/, т/(тг) = 2тг.
222.	Построить по два последовательных приближения (не считая исходного) к решениям следующих уравнений и систем:
а) у1 = 2х -h г, zf = у; т/(1) = 1, г(1) = 0.
б^=у> ё =ж2: ж(0) = Е у(0) = 2-
в) у" + у'2 - 2у - 0; /ДО) - 1, //(0) - 0.
г)§ = ЗЬ; »(1) = 2, ||	=-1.
at	at |t=1
223.	Указать какой-нибудь отрезок, на котором существует решение с данными начальными условиями:
а)	у' = х + у\ у(0) = 0.
б)	у' = 2у2 - х, у(1) = 1.
в)	= t + ех. ж(1) = 0.
Г) t=y2’ М =ж2’ Ж(О) = 1’ У(О) = 2‘
32	§ 7. Существование и единственность решения
224*. Для уравнения у' = х — у2 с начальным условием 2/(0) = 0 построить третье приближение к решению и оценить его ошибку при 0 х 0,5.
Указание. Оценить остаток ряда, сходимость которого доказывается в теореме существования решения, см. [1], гл. II, § 1; И, §15.
225.	Пользуясь каким-либо достаточным условием единственности, выделить области на плоскости ж, у, в которых через каждую точку проходит единственное решение уравнения
а) у' = 2ху + у2,
б) у' = 2 + itfy - 2ж,
г) у' = 1 + tgy,
е) ху' = у + д/у2 - а?2
Д) {.У-х)у' = у In ж,
226.	При каких неотрицательных а нарушается единственность решений уравнения у' = \у\а и в каких точках?
227.	С помощью необходимого и достаточного условия единственности для уравнений вида yr = f(y) (см. [1], гл. III, § 4, п. 1, мелкий шрифт или [2], § 4) исследовать написанные ниже уравнения. Выделив области, где f(y) сохраняет знак, приближенно изобразить на чертеже решения. Для уравнений д) и е) правые части при у = 0 доопределяются по непрерывности.
а) У' = '/'if-.
в) У' = (у - 1)а/у^-
д) у' = ylny,
228.	При каких начальных условиях существует единственное решение следующих уравнений и систем?
а) у” = tgy + tyx,
в) (ж - у)у'у'” = 1пжу,
б) у1 = у^у + 1, г) у' = arccos //, е) у' = у In2 у.
г) у” - уу'" = Vy' - х,
a^=y2+^.^=irx,
е) J = У3 + ln(t + 1),
§ 7. Существование и единственность решения
33
229.	Могут ли графики двух решений данного уравнения на плоскости ж, у пересекаться в некоторой точке (жо, ?/о)
а)	для уравнения у1 = х + у21
б)	для уравнения у" = х + у21
230.	Могут ли графики двух решений данного уравнения на плоскости ж, у касаться друг друга в некоторой точке (^о, Уо)
а)	для уравнения у' = х + у21
б)	для уравнения у" = х + у21
в)	для уравнения уш = х + у21
231.	Сколько существует решений уравнения у^ = х + + у2, удовлетворяющих одновременно двум условиям: y(fi) = = 1, з/(0) = 2? Рассмотреть отдельно случаи п = 1, 2, 3.
232.	Сколько решений уравнения у^ = /(ж, у) (f и fy непрерывны на всей плоскости ж, у) проходит через точку (жо, Уо) по заданному направлению, образующему угол а с осью 0x2 Рассмотреть случаи n = 1, п = 2 и 3.
233.	При каких п уравнение у^ = /(ж, у) (J и fy непрерывны) может иметь среди своих решений две функции: У1 = X, у2 = X + Ж4?
234.	При каких п уравнение = /(ж, у, у1,_, у^п~^)
с непрерывно дифференцируемой функцией f может иметь среди своих решений две функции: у± = ж, у% = sin ж?
235*. Пусть /(ж, у) непрерывна по ж, у и при каждом ж не возрастает при возрастании у. Доказать, что если два решения уравнения у1 = /(ж, у) удовлетворяют одному и тому же начальному условию ?/(жо) = у^. то они совпадают при ж xq.
236.	Сколько производных имеют решения следующих уравнений и систем в окрестности начала координат? (Теорему о гладкости решений см. [2], § 19 или [4], § 6, теорема 1.4.)
а) у' = х + у7/3,	б) у' = ж|ж| - у2,
в) у" = |ж3| + у5/3,	г) у"' = у — Ху/х,
fl) § =t + y, § =ж + £2|£|,
е)^=^ + Ж | =
34 §8. Уравнения, не разрешенные относительно производной
237*. При каких а каждое решение продолжается на бесконечный интервал — оо < х < +оо
а)	для уравнения у' =
б)	для уравнения у' = (у2 +еж)а?
в)	для уравнения у' = |т/|а-1 + |х'^Л/|2а?
г)	для системы у1 = (у2 + z2 + 2)-ы, z1 — у(1 + г2)ы?
238*. Для следующих уравнений доказать, что решение с произвольным начальным условием у(хо) = уо существует при х0 х < -hoc:
а) //' = .г3 — 7/3,	б) у' = ху + е~у.
239*. Пусть на всей плоскости х, у функции /(ж, у) и fy(x, у) непрерывны и fy(x, у) к(х), функция к(х) непрерывна. Доказать, что решение уравнения у1 = /(ж, у) с любым начальным условием у(хъ) = т/о существует при xq х < +оо.
240*. Дана система в векторной записи у1 = /(ж, т/), удовлетворяющая условиям теоремы существования в окрестности каждой точки (ж, у). Пусть в области \у\ > b при всех х
У f(x, у) &(ж)|у|2,
где у • / — скалярное произведение, а функция к(х) непрерывна. Доказать, что решение с любым начальным условием 2/(^о) = Уъ существует при ж0 х < +оо.
§ 8.	УРАВНЕНИЯ, НЕ РАЗРЕШЕННЫЕ ОТНОСИТЕЛЬНО ПРОИЗВОДНОЙ
1.	Уравнения вида F(x, у, у) = 0 можно решать следующими методами.
а)	Разрешить уравнение относительно у', т. е. из уравнения Р(ж, у, у) = 0 выразить у' через х и у. Получится одно или несколько уравнений вида у' = f(x, у). Каждое из них надо решить.
б)	Метод введения параметра1.
1 Здесь излагается простейший вариант этого метода. Более общий вариант см. [1], гл. III, § 3, п. 1.
§8. Уравнения, не разрешенные относительно производной 35
Пусть уравнение Р(ж, у, у) = 0 можно разрешить относительно у, т. е. записать в виде у = /(ж, у). Введя параметр
получим
У = f(x, р).	(2)
Взяв полный дифференциал от обеих частей равенства (2) и заменив dy через pdx (в силу (1)), получим уравнение вида
М{х, р) dx + 7У(ж, р) dp = 0.
Если решение этого уравнения найдено в виде х = ip(p), то, воспользовавшись равенством (2), получим решение исходного уравнения в параметрической записи: х = у?(р), у = f (^(р), р)-
Уравнения вида х = / (у, у) решаются тем же методом.
Пример. Решить уравнение у = х + у' — In?/. Вводим параметр р = у':
у = х + р — Inp.	(3)
Берем полный дифференциал от обеих частей равенства и заменяем dy на pdx в силу (1): dy = Пж+Пр— pdx = Пж-l-dp— Решаем полученное уравнение. Переносим члены с dx влево, с dp — вправо:
(р — 1) dx = -—-dp.	(4)
Р
а) Если р Ф 1, то сокращаем на р — 1:
1 dp _	„
dx = —, х = Inp + С.
Р
Подставляя это в (3), получаем решение в параметрической записи:
х = Inp + С, у = р + С.	(5)
В данном случае можно исключить параметр р и получить решение в явном виде. Для этого из первого из уравнений (5) выражаем р через ж, т. е. р = ех~с. Подставляя это во второе уравнение, получаем искомое решение:
У = ех~с + с.	(6)
б)	Рассмотрим случай, когда в (4) имеем р = 1. Подставляя р = 1 в (3), получаем еще решение
у = ж + 1.
(7)
36 §8. Уравнения, не разрешенные относительно производной
(Было бы ошибкой в равенстве р = 1 заменить р на у' и, проинтегрировав. получить у = х 4- С.)
2. Решение у = у>(ж) уравнения F(x, у, у) = 0 называется особым, если через каждую его точку, кроме этого решения, проходит и другое решение, имеющее в этой точке ту же касательную, что и решение у = у>(ж), но не совпадающее с ним в сколь угодно малой окрестности этой точки1.
Если функция F(x, у, у') и производные и непрерывны, то любое особое решение уравнения
у, у') = 0	(8)
удовлетворяет также уравнению
^W)=o	(9)
ду'
Поэтому, чтобы отыскать особые решения уравнения (3), надо исключить у' из уравнений (8) и (9). Полученное уравнение ^(ж, у) = = 0 называется уравнением дискриминантной кривой. Для каждой ветви дискриминантной кривой надо проверить, является ли эта ветвь решением уравнения (8), и если является, то будет ли это решение особым, т. е. касаются ли его в каждой точке другие решения.
Пример. Найти особое решение уравнения
у = х + у'-\пу’.	(10)
Дифференцируем обе части равенства по уг:
0 = 1-^.	(11)
Исключаем уг из уравнений (10) и (11). Из (11) имеем yf = 1; подставляя это в (10), получаем уравнение дискриминантной кривой
У = х + 1.	(12)
Проверим, будет ли кривая особым решением. Для этого сначала проверяем, является ли она решением уравнения (10). Подставляя (12) в (10), получаем тождество х + 1 = х + 1. Значит, кривая (12) — решение.
хЭто определение взято из [1]. Есть и другие определения, не равносильные этому.
§8. Уравнения, не разрешенные относительно производной 37
Теперь проверим, является ли это решение особым, т. е. касаются ли его в каждой точке другие решения. В п. 1 было найдено, что другие решения выражаются формулой (6). Пишем условия касания кривых у = ?/1(ж) и у = ?/2(ж) в точке с абсциссой xq:
У1(х0) = ?/2(ж0), 2/'1(жо) = Х/2(ж0).	(13)
Для решений (6) и (12) эти условия принимают вид ех°~с + С = = жо+1, еЖо-с = 1. Из второго равенства имеем С = жо; подставляя это в первое равенство, получаем 1 + хо = хо + 1. Это равенство справедливо при всех xq. Значит, при каждом хо решение (12) в точке с абсциссой хо касается одной из кривых семейства (6), а именно той кривой, для которой С = жо-
Итак, в каждой точке решение (12) касается другого решения (6), не совпадающего с ним. Значит, решение (12) — особое.
Если семейство решений записано в параметрическом виде, как в (5), то выполнение условий касания проверяется аналогично. При этом надо учесть, что у' = р.
3. Если семейство кривых Ф(ж, у, С) = 0, являющихся решениями уравнения Е(ж, ?/, у) = 0, имеет огибающую у = у>(ж), то эта огибающая является особым решением того же уравнения. Если функция Ф имеет непрерывные первые производные, то для отыскания огибающей надо исключить С из уравнений
ф(ж, у, С) = 0, аФ( д/’ С) = о (/О
и проверить, будет ли полученная кривая огибающей, т. е. касаются ли ее в каждой точке кривые семейства. Эту проверку можно провести изложенным в конце п. 2 методом, используя условия касания (13).
В задачах 241—250 найти все решения данных уравнений; выделить особые решения (если они есть); дать чертеж.
241. у'2 -у2 = 0.	242. 8у'3 = 27у.
243. (у' + 1)3 = 27(ж + у)2.
245. у2(у,2 + 1) = 1.
245. у'2 - 4л? = 0.	246. у'2 = 4г/3 (1 - у).
247. ху'2 = у.	245. уу'3 + х = 1.
249. у'3 + у2 = уу'(у' + 1).
38 §8. Уравнения, не разрешенные относительно производной
250. 4(1 - у) = (Зу - 2)2у'2.
Уравнения 251—266 разрешить относительно у', после этого общее решение искать обычными методами (§§ 2,4,5,6). Найти также особые решения.
если они есть.
251.	у'2 + ху = у2 + ху'.	252.	жу'(жу' + у) = 2у2.
253.	ху'2 — 2уу' + х = 0.	254.	жу'2 = у(2у' - 1).
255.	у'2 + х = 2у.	256.	у'3 + (ж + 2)е® = 0.
257.	у'2 - 2ху' = 8х2.	258.	(ху' + Зу)2 = 7х.
259.	у'2 - 2уу' = у2(ех - 1).	
260. у'(2у — у') = у2 sin2 х.
261.	у'4 + у2 = у4.
262.	х(у — ху')2 = ху'2 — 2уу'.
263.	у(жу' - у)2 = у - 2ху'.
264.	уу'(уу' - 2х) = х2 - 2у2.
265.	у'2 + 4жу' — у2 — 2х2у = х4 — 4ж2.
266.	у(у - 2жу')2 = 2у'.
Уравнения 267—286 решить методом введения параметра.
267. х = у'3 + у'.	268.	х(у'2 - 1) = 2у'-
269. х = у'у/у'2 + 1.	270.	у'(ж-1пу') = 1.
271. у = у'2 + 2у'3.	272.	у = 1п(1 + у'2).
273. (у' + I)3 = (у' - у)2.	274.	у = (у' - I)®2''.
275. у'4 - у'2 = у2.	276.	/2	/3	2 У -у =У-
277. у'4 = 2уу' + у2.	278.	у'2 — 2ху' = х2 — 4у.
279. 5у + у'2 = х(х + у').	280.	х2у'2 = хуу' + 1.
281. у'3 + у2 = хуу'.	282.	2ху' — у — у' In у у'.
283. у' = еху'/у.	284.	/	2 /3 у — ху — X у .
§9. Разные уравнения первого порядка
39
285.	у = 2ху' + у2у'3.	286. у(у - 2ху')3 = у'2.
Решить уравнения Лагранжа и Клеро (задачи 287—296).
287.	у = хуг - у'2.	288.	у + хуг = ^у/у7.
289.	у = 2ху' - 4у'3.	290.	у = ху' - (2 + у').
291.	у'3 = 3(ху' - у).	292.	у = ху'2 - 2у'3.
293.	ху' — у = In?/.	294.	ху'(у'	+ 2) = у.
295.	2у'2(у — xyr) = 1.	296. 2хуг — у = In?/.
297.	Найти особое решение дифференциального уравнения, если известно семейство решений этого уравнения:
а) у = Сх2 - С2.	б) Су = (х- С)2,
в) у = С(ж - С)2,	г) ху = Су- С2.
298.	Найти кривую, каждая касательная к которой образует с осями координат треугольник площади 2а2.
299.	Найти кривую, каждая касательная к которой отсекает на осях координат такие отрезки, что сумма величин, обратных квадратам длин этих отрезков, равна 1.
300.	Найти кривую, проходящую через начало координат и такую, что отрезок нормали к ней, отсекаемый сторонами первого координатного угла, имеет постоянную длину, равную 2.
§ 9. РАЗНЫЕ УРАВНЕНИЯ ПЕРВОГО ПОРЯДКА1
Решить уравнения 301—330 и построить графики их решений.
301.	ху' + х2 + ху — у = 0.	302. 2ху' + у2 = 1.
303.	(2ху2 — у) da? + х dy = 0.
304.	(xyf + у)2 = x2yr.	305. у — у' = у2 + хуг.
306.	(ж + 2у3)у' = у.	307. у'3 - у'е2х = 0.
хВсе задачи §9 решаются изложенными ранее методами.
10
§9. Разные уравнения первого порядка
308. x2yr = y(x + у).	
309.	(1 — ®2) dy + xy d;c = 0. 310.	y'2 + 2(ж - l)y' - 2y = 0. 311.	у + у'In2 у = (ж + 21ny)y'. 312.	ж2у' — 2жу = Зу. 313.	ж + уу' = у2(1 + у'2). 314.	у = (ху' + 2у)2.	315.	у' =	1 2- х-у
316. у'3 + (Зж - 6)у' = Зу.	317.	II *Ь|^Э5 1
318. 2у'3 - Зу'2 + ж = у.	319. 320.	2ж3уу' + Зж2у2 + 7 = 0. 321.	- 2ж^ dy. X	уу	J 322.	ху' = & + 2у'. 323.	2(х - у2) dy = у dx. 324.	ж2у'2 + у2 = 2ж(2 - уу'). 325.	dy + (ху — ху3) dx = 0. 326.	2х2у' = у2(2ху' — у). 327.	= 2.	(ж + у)2у' = 1.
х + УУ
328.	х(х — 1)у' + 2ху = 1.
329.	ху(ху' — у)2 + 2у' = 0.
330.	(1 — х2)у' — 2ху2 = ху.
Решить уравнения 331—420.
331.	у' + у = ху3.
332.	(ху4 - х) dx* + (у + ху) dy = 0.
333.	(sinx + у) dy + (?/cosx — х2) dx = 0.
§9. Разные уравнения первого порядка
41
334.	Зу'3 - ху' + 1 = 0.
335.	уу' + у2 ctg;c = cos ж.
336.	(о27 + 2ху) (Lr + (о27 + х)х dy = 0.
337.	ху'2 = у-у'.
338.	ж(ж + 1)(«/'-1) =у.
339.	у(у - ху') = л/х^Ту*.
340.	ху1 + у = In?/.
341.	.z'2(d?/ — dx} — (х + y}ydx.
342.	у' + xf/y = Зу.
343.	(х cos у + sin 2у}у' = 1.
344.	у'2 - уу' + еж = 0.
345.	у' = f е2ж + у.
346.	(ху' - у)3 = у'3 - 1.
347.	(4жг/ - 3)г/' + у2 = 1.
348.	у' у[х — у/у — х + у/х.
349.	ху' = 2^/t/cosj; — 2у.
350.	ЗУ4 =у' + у.
351.	у2 (у - ху') = х3у'.
352.	у' = (4ж + у-3)2.
353.	(cos х — х sin x}ydx + (х cos х — 2у} dy = 0.
354.	х2у'2 — 2хуу' = х2 + Зу2.
355.	+2ж?/1пж +1 = 0.
356.	ху' = ху/у — х2 + 2у.
357.	(1 — х2у) (кг + х2(у — х) dy = 0.
358.	(2Ж627 + у4)уг = уеУ.
42
§9. Разные уравнения первого порядка
359.	ху’^у — Inх) = у.
360.	2у' = х + In?/.
361.	(2х2у — Зу2)у’ = 6а;2 — 2ху2 + 1.
362.	уу' = 4а; + Зу — 2.
363.	у2у' + х2 sin3 х = у3 ctga;.
364.	2ху' — у = sin?/'.
365.	(х2у2 + 1)у + (ху — 1)2хуг = 0.
366.	у sin х + у' cos х = 1.
367.	xdy — ydx = ху/х2 + у2 da;.
368.	у2 + х2у'5 = ху(у'2 + у'3).
369.	у' = ^/2х^у + 2.
370.	(х — у cos	da; + х cos dy = 0.
371.	2 (х2у + д/1 + a;4?/2) da; + х3 dy = 0.
372.	(у' - Xy/у) (ж2 - 1) = ху.
373.	у'3 + (у'2 - 2у')ж = Зу' - у.
374.	(2х + Зу — 1) скс + (4ж + бу — 5) dy = 0.
375.	(2жу2 - у) dz + (у2 + х + у) dy = 0.
376.	у = у'л/1 + у'2.
377.	у2 = (хуу' + 1) In х.
378.	dy = x2 +у'2.
379.	2х dy + у da; + ху2 (х dy + у dx) = 0.
380.	х da; + (х2 ctg у — 3 cos у) dy = 0.
381.	х2у'2 — 2(ху — 2)у' + у2 = 0.
382.	ху’ + 1 = ех~у.
383.	у' = tg(?/ - 2х).
384.	За;2 — у = у'\/х2 + 1.
§9. Разные уравнения первого порядка
43
385.	уу' + ху = х3.
386.	х(х — 1)уг + у3 = ху.
387.	ху' =2у+ д/1 + у'2.
388.	(2ж + у + 5)з/' = Зж + 6.
389.	у' + tg«/ = х secy.
390.	у'4 = 4у(ху' — 2г/)2.
391.	у' =
2у(х +1)
392.	ху' = ж2е-г/ + 2.
393.	у' = Зж + д/г/ - ж2.
394.	ж dy — 2г/ <1ж + ху2 (2ж dy + у <1ж) = 0.
395.	(ж3 — 2жг/2) <1ж + Зж2г/ dy = ж dy — у <1ж.
396.	(г/г/')3 = 27ж(г/2 — 2ж2).
397.	у' - 8х^у = х — 1
398.	[2ж - 1п(г/ + 1)] аж - dy = 0.
399.	хуг = (ж2 + tg у) cos2 у.
400.	ж2(г/ - ху') = уу'2.
401. у' =	Зж2
	ж3 + у + 1
402. у' =	(1 + г/)2
	х(у + 1) — х2
	
403.	(г/ — 2жг/')2 = 4г/г/'3.
404.	6ж5у/(1ж + (?/4 In у — Зж6) dy = 0.
405.	у' = i-у/ж + ^у.
406.	2ху' + 1 = у Ч—Г.
У - 1
44	§ 10. Уравнения, допускающие понижение порядка
/2	2 \
407.	уу' + х = 1 (М^) •
409.	ух^/у1 2 + 1 + 1J (у2 + 1) da; = xydy.
410.	(а;2 + у2 + У) у у' + (а;2 + у2 — 1)а; = 0.
411.	у2(х — 1) da; = х(ху + х — 2у) dy.
412.	(ху' — у)2 = х2у2 — х4.
413.	хуу' — х2 у/у2 = (х + 1)(?/2 + 1).
414.	(а;2 — 1)уг + у2 — 2ху + 1 = 0.
415.	у' tg у + 4ж3 cos у = 2х.
416.	(Ху'-уУ = у'2- ^+1.
417.	(а; + у)(1 — ху) da; + (а; + 2у) dy = 0.
418.	(Зху + х + у)у da; + (dxy + х + 2у)х dy = 0.
419.	(а;2 — 1) da; + (х2у2 + а;3 + х) dy = 0.
420.	х(у'2 + е2?/) = -2у'.
§ 10. УРАВНЕНИЯ, ДОПУСКАЮЩИЕ ПОНИЖЕНИЕ ПОРЯДКА
1. Если в уравнение не входит искомая функция у, т. е. оно имеет вид Р(ж, у^к\ у^к+1\ ..., у^) = 0, то порядок уравнения можно понизить, взяв за новую неизвестную функцию низшую из производных, входящих в уравнение, т. е. сделав замену у^ = z.
2. Если в уравнение не входит независимое переменное ж, т. е. уравнение имеет вид F(y, у', у", ..., у^) = 0, то порядок уравне-
ния можно понизить, взяв за новое независимое переменное у, а за неизвестную функцию у' = р(у).
Пример. Решить уравнение 2уу" = у'2 + 1.
§ 10. Уравнения, допускающие понижение порядка 45
В уравнение не входит х. Полагаем уг = р(у). Тогда
//	<1(2/')	&Р(У)	Ф &У	/
У = 1— =	— = 3“ ’	= Р Р-
аж аж	ау аж
Подставляя у = р и у" = рр в уравнение, получим 2урр = р2 + 1. Порядок уравнения понижен. Решив полученное уравнение, найдем р = ±у/Су — 1. Следовательно, yf = ±у/Су — 1. Из этого уравнения получим 4(Су — 1) = С2(ж + С2).
3.	Если уравнение однородно относительно у и его производных, т. е. не меняется при одновременной замене у, у , у", ... на ky, куг, ку". ..., то порядок уравнения понижается подстановкой у' = yz, где z — новая неизвестная функция.
4.	Порядок уравнения понижается, если оно является однородным относительно ж и у в обобщенном смысле, т. е. не меняется от замены ж на кх, у на к™у (при этом у' заменяется на к™-1 у', у” — на ктп~2у,г и т. д.). Чтобы узнать, будет ли уравнение однородным, и найти число т, надо приравнять друг другу показатели степеней, в которых число к будет входить в каждый член уравнения после указанной выше замены. Например, в первый член уравнения 2хАу" — Зу2 = ж4 после этой замены число к будет входить в степени 4 + (т — 2), во второй — в степени 2m, в третий — в степени 4. Следовательно, т должно удовлетворять уравнениям
4 + (т — 2) = 2т = 4.
Отсюда т = 2. Если же полученные уравнения для т будут несовместными, то дифференциальное уравнение не является однородным в указанном смысле.
После того как число т найдено, надо сделать замену переменных ж = е\ у = zemt, где z = z(t) — новая неизвестная функция, a t — новое независимое переменное. Получим уравнение, в которое не входит независимое переменное t. Порядок такого уравнения понижается одним из ранее рассмотренных способов.
5.	Порядок уравнения легко понижается, если удается преобразовать уравнение к такому виду, чтобы обе его части являлись полными производными по ж от каких-нибудь функций. Например, пусть дано уравнение уу" = у'2. Деля обе части на уу', получим (In?/')' = (In?/)'; In?/' = In?/ + In С; у' = yC. Порядок уравнения понижен.
Решить уравнения 421—450.
421. х2у" = у'2.	422. Ъху'у" = у'2 - 1.
46
§ 10. Уравнения, допускающие понижение порядка
423. уэу" = 1.	424.	у'2 + 2уу" = 0.
425. у" = 2уу'.	426.	уу” +1 = у’2 
427. у"(ех + 1) + у' = 0.	428.	у”’ = у”2-
429. уу" = у'2 — у'3.	430.	у"' = 2(у" — 1) ctgx.
431. 2уу" = у2 +у'2.	432.	у"3 + ху" = 2у'.
433. у"2 + у' = ху".	434.	у" + у'2 = 2е~У.
435. ху'" = у" — ху".	436.	ff2	I i У =У + 1-
437. у" = eV.	438.	у" — ху"' + у'"3 = 0.
439. 2у'(у" + 2) = ху"2.	440.	у4 - у3у" = 1.
441. у'2 = (Зу - 2у')у".	442.	у"(2у' + х) = 1.
443. у"2 - 2у'у"' + 1 = 0.		
444. (1 — х2}у” + ху' = 2.		
445. уу” — 2уу' In?/ = у'2.		
446. (у' + 2у)у" = у'2.	447.	®у" = у' + х sin V-.
448. у'”у'2 = у”3.	449.	и 1	/2 УУ +у = У 
450. ху” = у' + х(у'2 + ж2).
Решить уравнения 451—454, воспользовавшись формулой, сводящей многократное интегрирование к однократному (см. [1], гл. IV, § 2, п. 1).
451. xyw = 1.	452. ху” = sin х.
453. у”' = 2ху”.	454. ху1У + у”' = еж.
Решить уравнения 455—462, преобразовав их к такому виду, чтобы обе части уравнения являлись полными производными.
455. уу”' + Зу'у” = 0.
457. уу” = у\у' + 1).
459. уу" + у'2 = 1.
456. у'у'" = 2у"2.
458. 5у"'2 - 3y"yw = 0.
460. у” = хуг + у + 1.
§ 10. Уравнения, допускающие понижение порядка 47
461. ху" = 2уу' — у'.	462. ху" — у' = х2уу'.
В задачах 463—480 понизить порядок данных уравнений, пользуясь их однородностью, и решить эти уравнения.
463. хуу" — ху'2 = уу'. 464. уу" = у'2 + \Ьу2у/х.
465. (ж2 + 1)(у'2 - уу”) = хуу'.
466. хуу" + ху'2 = 2уу'.	467. х2уу" = (у — ху')2.
468.	у”+У- + ^=У~.
X X* у
469.	у(ху" + у') = ху’2(1 - х).
470.	х2уу" + у'2 = 0.
471.	х2(у'2 - 2уу") = у2.
472.	хуу" = у'(у + у').
473.	4х2у3у" = х2 — у4.
474.	х3у" = (у — ху')(у — ху' — х).
475.	+ у’2 = 3>ху” +
X*	X
гч	5\ .	«	4 у/
2ху------) у +4у2---
X)	хл
477.	х2(2уу" - у'2) = 1 - 2хуу'.
478.	х2(уу" — у'2) + хуу' = (2ху' — Зу)у/хР.
479.	х4(у'2 — 2уу") = ^х^уу' + 1.
480.	уу' + хуу" — ху'2 = ж3.
В задачах 481—500, понизив порядок данных уравнений, свести их к уравнениям первого порядка.
481.	у"(3 + уу'2) = у'4.	482. у"2 - у'у"' = (^.
483. уу' + 2х2у" = ху'2.	484. у'2 + 2хуу" = 0.
485.	2ху2(ху" + у') + 1 = 0.
476. у" =
48
§ 10. Уравнения, допускающие понижение порядка
486.	х(у" + у'2) = у'2 + у'.
487.	у2(у'у'" - 2у"2) = у'4.
488.	у(2ху" + у') = ху'2 + 1.
489.	у" + 2уу'2 = (2х + ±) у'.
490.	у'у'" = у”2 + у'2 у".	491. уу" = у'2 + 2ху2.
492. у"4 = у'5 - уу'3у".	493. 2уу"' = у'.
494. у'"у'2 = 1.	495. у2у'" = у'3.
496.	х2уу" + 1 = (1 — у)ху'.
497.	уу'у'" + 2у'2у" = Зуу"2.
498.	(у'у'" - Зу"2)у = у'5.
499.	у2(у'у'" - 2у"2) = уу'2у" + 2у'4.
500.	х2(у2у'" - у'3) = 2у2у' - Зхуу'2.
В задачах 501—505 найти решения, удовлетворяющие заданным начальным условиям.
501.	уу" = 2ху'2; 7/(2) = 2, у'(2) = 0,5.
502.	2у'" - Зу'2 = 0; 2/(0) = -3, т/'(0) = 1, т/"(0) = -1.
503.	х2у" — Зху' =	— 4т/; т/(1) = 1, у'(1) = 4.
504.	у'" = Зуу'-, 7/(0) = -2, т/'(0) = 0, т/"(0) = 4,5.
505.	у" cos у + у'2 sin у = у'-, т/(-1) = f, у'(-1) = 2.
506.	Найти кривые, у которых в любой точке радиус кривизны вдвое больше отрезка нормали, заключенного между этой точкой кривой и осью абсцисс. Рассмотреть два случая: а) кривая обращена выпуклостью к оси абсцисс; б) вогнутостью к оси абсцисс.
507.	Найти кривые, у которых радиус кривизны обратно пропорционален косинусу угла между касательной и осью абсцисс.
508.	Определить форму равновесия нерастяжимой нити с закрепленными концами, на которую действует нагрузка
§11. Линейные уравнения с постоянными коэффициентами 49 так, что на каждую единицу длины горизонтальной проекции нагрузка одинакова (цепи цепного моста). Весом самой нити пренебречь.
509.	Найти форму равновесия однородной нерастяжимой нити (с закрепленными концами) под действием ее веса.
510*	. Доказать, что уравнение движения маятника у" + + sin?/ = 0 имеет частное решение ?/(ж), стремящееся к тг при х +ос.
§ 11.	ЛИНЕЙНЫЕ УРАВНЕНИЯ
С ПОСТОЯННЫМИ КОЭФФИЦИЕНТАМИ
1.	Чтобы решить линейное однородное уравнение с постоянными коэффициентами
еад(п) +	+ ... + ап-ху + апу = 0,	(1)
надо составить характеристическое уравнение
«оАп -I- «1ЛП + ... + ап—1Л + ап = 0	(2)
и найти все его корни Л1, ..., Лп.
Общее решение уравнения (1) есть сумма, состоящая из слагаемых вида CieXiX для каждого простого корня Л? уравнения (2) и слагаемых вида
(Ст + 1 + Ст+2Ж + Ст+ЗЖ2 + . . . + Cm+fc^^ 1)еЛж	(3)
для каждого кратного корня Л уравнения (2), где к — кратность корня. Все Ci — произвольные постоянные. Коэффициенты уравнения (1) и корни Л здесь могут быть вещественными или комплексными.
Если же все коэффициенты уравнения (1) вещественные, то решение можно написать в вещественной форме и в случае комплексных корней Л. Для каждой пары комплексных сопряженных корней Л = а ± fii в формулу общего решения включаются слагаемые
Ст+1еаж cos fix + Ст+2еаж sin/Зж,
если эти корни простые, и слагаемые
Pk-i(x)eax cos fix + Qk-i(x)efix siiafix.
50 §11. Линейные уравнения с постоянными коэффициентами
если каждый из корней а + (3i и а — (3i имеет кратность к. Здесь Pk-i и Qk-i — многочлены степени к — 1, аналогичные многочлену в (3), их коэффициенты — произвольные постоянные.
Пример. Решить уравнение yv — 2ylv — 16у' + 32?/ = 0.
Пишем характеристическое уравнение
А5 - 2А4 - 16А + 32 = 0.
Разлагая левую часть на множители, находим корни:
(А - 2)(А4 - 16) =0. (А - 2)2(А + 2)(А2 + 4) = 0,
Л1 = Л2 = 2, Аз = —2, Л4 = 2г, А5 = —2г.
По изложенным выше правилам пишем общее решение
у = (Ci + С2ж)е2ж + Сзе-2ж + С4 cos 2х + С5 sin2x*
(степень многочлена Ci + С2Ж на 1 меньше кратности корня Л = 2).
2.	Для линейных неоднородных уравнений с постоянными коэффициентами и с правой частью, состоящей из сумм и произведений функций bo + bix + ... + 6тжт, еаж, cos/Зж, sin/Зж, частное решение можно искать методом неопределенных коэффициентов.
Для уравнений с правой частью Рт(ж)е7Ж, где Рт(ж) = 6о+ -\-bix + ... + частное решение имеет вид
У1 = ж5<Эт(ж)е7Ж,	(4)
где <Эт(ж) — многочлен той же степени т. Число s = 0, если 7 — не корень характеристического уравнения (2), а если 7 — корень, то s равно кратности этого корня. Чтобы найти коэффициенты многочлена <Эт(ж), надо решение (4) подставить в дифференциальное уравнение и приравнять коэффициенты при подобных членах в левой и правой частях уравнения.
Если в правую часть уравнения входят синус и косинус, то их можно выразить через показательную функцию по формулам Эйлера
cos/Зж = ----------, sin/Зж = ----—----- (5)
и свести задачу к уже рассмотренному случаю.
Если же коэффициенты левой части уравнения вещественны, то можно обойтись без перехода к комплексным функциям (5). Для уравнения с правой частью
ео:!'(р(ж) cos Рх + <Э(ж) sinРх)
(6)
§11. Линейные уравнения с постоянными коэффициентами 51
можно искать частное решение в виде
ух = ж5еаж(Вт(ж) cos/Зж + Тт(ж) sin/Зж),	(7)
где s = 0, если а + (3i не корень характеристического уравнения, и s равно кратности корня а + Pi в противном случае, a Rm и Тт — многочлены степени т, равной наибольшей из степеней многочленов Р и Q. Чтобы найти коэффициенты многочленов Rm и Тт. надо подставить решение (7) в уравнение и приравнять коэффициенты при подобных членах.
Еще один метод отыскания частного решения уравнения с вещественными коэффициентами и правой частью вида (6) состоит в следующем. Сначала решают уравнение с правой частью Р(ж)е(о:+/Зг)ж. Вещественная часть этого решения будет решением уравнения с правой частью Р(ж)еаж cos /Зж, а мнимая — решением уравнения с правой частью Р(ж)еаж sin/Зж.
Если правая часть уравнения равна сумме нескольких функций вида Р(ж)е7Ж и вида (6), то частное решение отыскивается по следующему правилу.
Частное решение линейного уравнения с правой частью fi + + ... + fp равно сумме частных решений уравнений с той же левой частью и правыми частями Д, ..., fp.
Общее решение линейного неоднородного уравнения во всех случаях равно сумме частного решения этого уравнения и общего решения однородного уравнения с той же левой частью.
Пример. Решить уравнение
у'” — бу” + §у = же3ж + е3ж cos 2ж.	(8)
Характеристическое уравнение Л3 —6Л2+9Л = 0 имеет корень Л = 3 кратности 2 и корень Л = 0 кратности 1. Поэтому общее решение однородного уравнения имеет вид у$ = (Ci + С2ж)е3ж + + Сз-
Правая часть (8) состоит из двух слагаемых вида (6); для первого 7 = a + pi = 3, а для второго а + /3г = 3 + 2г. Так как эти числа различны, то надо искать отдельно частные решения уравнений
у'” - бу” + 9у' = же3ж,	(9)
у'” — бу” + 9у' = е3ж cos 2ж.	(10)
Число 7 = 3 является корнем кратности s = 2, поэтому частное решение уравнения (9) согласно (4) имеет вид yi = ж2(аж+ +6)е3ж. Подставив у = у± в (9), найдем а = 1/18, Ь = —1/18.
52 §11. Линейные уравнения с постоянными коэффициентами
Далее, число а + (3i = 3 + 2г не является корнем характеристического уравнения, поэтому частное решение уравнения (10) согласно (7) имеет вид у2 = e3®(ccos2a? + dsin2a?). Подставив у = у2 в (10), найдем с = —3/52, d = —1/26.
Общее решение уравнения (8) равно у = уо + yi + г/2, где г/о, г/i, у2 уже найдены.
3.	Линейное неоднородное уравнение
еад(п) +	+ ... + апу = f(x)	(11)
с любой правой частью f(x) решается методом вариации постоянных. Пусть найдено общее решение у = Ciyi + ... + Спуп линейного однородного уравнения с той же левой частью. Тогда решение уравнения (11) ищется в виде
у = С1(ж)?/1 + ... + Сп(х)уп.
Функции Ci{x) определяются из системы
С\У1 + • • • + СпУп = 0
С1У1 + • • • + С'пу'п = 0
С{у[п-2) + ...+ С^п~2) = 0 + ... + сЫ"-1’) = f(x).
4.	Уравнение Эйлера
аохпу(п>) + а1хп~1у^п~1>> + ... + an_ixy' + апу = f(x) (12) сводится к линейному уравнению с постоянными коэффициентами заменой независимого переменного х = е* при х > 0 (или х = —е* при х < 0). Для полученного уравнения с постоянными коэффициентами характеристическое уравнение имеет вид
«оА(Л — 1)(Л — 2)... (Л — п +1) + ... + ап_гЛ(Л — 1) + ап—1Л + ап = 0.
тт	If (If}
При составлении этого уравнения каждое произведение х у 1 в (12) заменяется на произведение к убывающих на 1 чисел: Л(Л — 1)(Л — 2)... (Л — fc + 1).
Пример. Решить уравнение
з ///	2 п . о о з
х у — х у + 2ху — 2у = х .
(13)
§11. Линейные уравнения с постоянными коэффициентами 53
Сразу пишем характеристическое уравнение и решаем его:
Л(Л - 1)(Л — 2) — Л(Л — 1) + 2Л — 2 = 0,	(14)
(А - 1) (А2 — ЗА + 2) = 0, Ai = А2 = 1, Аз = 2.
При таких Л общее решение однородного уравнения с постоянными коэффициентами имеет вид (согласно п. 1)
у® = (Ci + 62^)0* + Сзе2*.
Чтобы решить неоднородное уравнение (13), сначала раскроем скобки в (14): Л3 — 4Л2 + 5Л — 2 = 0. По этому характеристическому уравнению составляем левую часть дифференциального уравнения, а правую часть получаем из правой части (13) заменой х = е*:
у'" - 4у" + by't -2у = e3t.
Так как число 3 не является корнем характеристического уравнения, то частное решение ищем в виде у± = ae,3t. Подставляя в уравнение, находим а = 1/4.
Следовательно, общее решение имеет вид
У = уо + yi = (Ci + С2^)е* + Сзе2* + -е3< =
= (Ci + С2 1пж)ж + Сзх2 + |ж3 (ж > 0).
При ж < 0 получается аналогичная формула, но с 1п|ж| вместо In ж.
5.	Для решения задач 635—640 и 879 можно пользоваться следующими законами теории электрических цепей (см. также [3], §13).
Для каждого узла цепи сумма всех притекающих токов равна сумме вытекающих токов.
Алгебраическая сумма напряжений источников тока, содержащихся в любом замкнутом контуре цепи, равна алгебраической сумме падений напряжений на всех остальных участках этого контура.
Падение напряжения на сопротивлении R равно RI; падение напряжения на самоиндукции L равно падение напряжения на конденсаторе емкости С равно q/C, где q = q(t)— заряд конденсатора в момент £; при этом = I; во всех трех случаях I = 1(f) — сила тока, протекающего через рассматриваемый участок цепи в данный момент t. В этих формулах I выражается в амперах, R, — в омах, L — в генри, q — в кулонах, С — в фарадах, t — в секундах, напряжение — в вольтах.
54 §11. Линейные уравнения с постоянными коэффициентами
Пример. Последовательно включены: источник тока, напряжение которого меняется по закону Е = V sincctf. сопротивление R и емкость С. Найти силу тока в цепи при установившемся режиме1.
Решение. Сила тока I = 1(f) на любом участке цепи одна и та же (по закону о последовательном соединении). Падение напряжения на сопротивлении равно RI, а на емкости q/C. Следовательно. R.I +	= V sinojC Дифференцируя и пользуясь тем, что
dq Т С — = I, получим уравнение
7?^—h — = Vw coswt. (it О
(15)
Это — линейное уравнение с постоянными коэффициентами. Для отыскания установившегося режима найдем периодическое решение этого уравнения. Исходя из вида правой части уравнения, ищем решение в виде
I = Ai cos сЛ + Bi sinwt.	(16)
Подставляя (16) в (15) и приравнивая коэффициенты при подобных членах, получим систему двух уравнений, из которой можно найти Ai и Bi. Но в электротехнике важнее знать не коэффициенты А± и Bi, а амплитуду изменения силы тока. Поэтому выражение (16) переписывают в виде
I = Asin(ci;£ — </?).	(17)
Подставляя (17) в (15), переходя к тригонометрическим функциям углов иЛ и </?, приравнивая коэффициенты сначала при sina;£, а затем при coso;t, получим
А	А
RAw sin р + — cos p = 0, RAw cos p — — sin p = Vw.
C	C
Отсюда найдем
=____1	= V
RCw
Поясним, почему найденное периодическое решение называется установившимся режимом. Общее решение уравнения (15) равно
1 Установившимся режимом называется такой, при котором сила тока постоянна или меняется периодически.
§11. Линейные уравнения с постоянными коэффициентами 55
сумме найденного частного решения (17) и общего решения линейного однородного уравнения
(18>
Так как решение уравнения (18) I = Ко,~^яс (здесь К — произвольная постоянная) стремится к нулю при t +оо, то любое решение уравнения (15) при t +оо неограниченно приближается (и притом весьма быстро) к найденному периодическому решению (17).
Решить уравнения 511—548.
511.	у" + у'-2у = 0.	512.	у” + 4у' + Зу = 0.
513.	У" ~ 2у' = 0.	514.	2yzz-5yz + 2y = 0.
515.	у" — 4у' + Ьу = 0.	516.	у” + 2у' + 10у = 0.
517.	у" + 4т/ = 0.	518.	у'" - 8у = 0.
519.	у™ - У = 0-	520.	yIV + 4у = 0.
521.	?/VI + 64т/ = 0.	522.	у" - 2у' + у = 0.
523.	±у" + 4т/ + у = 0.	524.	yv — 6yIV + 9yzzz = 0.
525.	yN - 10?/"' + 9у‘ = 0.		
526.	у1У + 2у” + у = 0.		
527.	ут — Зу” -I- Зу1 — у = 0.		
528.	Ут -у"-у' + у = 0.		
529.	у1У — Ъу” -h 4у = 0.	530.	yv + 8y"' + 16yz = 0.
531.	у”' - Зу' + 2у = 0.	532.	yIV + 4yzz + 3y = 0.
533.	у” - 2у‘ — Зу = е4ж.	534.	у" + у = 4®еж.
535.	у” — у = 2еж — х2.		
536.	у” + Уг ~ %У = Зхех.		
537.	у” — Зу' + 2у = sin х.		
538.	у” + У = 4 sin х.		
539.	у” — Зу' + 4у = 4ж2е2ж.		
56 §11. Линейные уравнения с постоянными коэффициентами
540.	у" — Зу' + 2у = х cos х.
541.	у” + Зу' — 4у = е-4ж + хе~х.
542.	у" + 2у' -Зу = х2ех.
543.	yff — 4уг + 8у = е2ж + sin2.T.
544.	у" — 9у = е3ж cos х.
545.	у" — 2у' + у = 6жеж.
546.	у” + у = х sin х.
547.	у” + 4уг + 4у = же2ж.
548.	у" — Ьу' = Зж2 + sin5a?.
В задачах 549—574 для каждого из данных уравнений написать его частное решение с неопределенными коэффициентами (числовых значений коэффициентов не находить).
549.	у” - 2у' + 2у = еж + х cos х.
550.	у" + &уг + Ют/ = Зже-Зж — 2е3ж cos х.
551.	у” — 8уг + 20т/ = 5.те4ж sin2:c.
552.	у" + Чу1 + Ют/ = же-2ж cos Ъх.
553.	у" — 2уг + Ъу = 2жеж + еж sin2#.
554.	у” — 2у' + у = 2хех + еж sin2#.
555.	у" — Sy' + 17т/ = е4ж(ж2 — 3#sin#).
556.	у”' + у' = sin х + х cos х.
557.	у"' - 2у" + Щ -8у = е2ж sin2a; + 2х2.
558.	yff — 6уг + 8у = 5же2ж + 2е4ж sin х.
559.	у” + 2у' + у = ж(е-ж — cos ж).
560.	у'” — у" — у' + у = Зеж + Зх sin х.
561.	у" — 6уг + 13т/ = ж2е3ж — 3cos2a?.
562.	у" — 9у = е-3ж(#2 + sin3#).
563.	у1У + у" = 7х — 3cosa?.
§11. Линейные уравнения с постоянными коэффициентами 57
564.	у” +	= cos х • cos Зж.
565.	у"' — 4у" + Зуг = х2 + же2ж.
566.	у” — ^у' + Ъу = е2ж sin2 х.
567.	у” + Зу' + 2у = е-ж cos2 х.
568.	у” — 2у' + 2у = (х + еж) sinх.
569.	y1N + Ъу” + 4у = sin х • cos 2х.
570.	у" - Зу' + 2у = 2Ж.
571.	у” — у = 4sh.r.
572.	у” + 4т/ + Зу = chх.
573.	у" + 4у = sh.r • sin 2;с.
574.	у" 4- 2у' + 2у = ch х • sin х.
Решить уравнения 575—581 способом вариации постоянных.
575.	у"-2у' + у=^.
576.	у" + 3у' + 2у =
577.	у” + у =
578.	у” + 4у = 2 tg х.
579.	у" + 2у'+ у = Зе-жл/ж+Т.
580.	у" + у = 2 sec3 х.
581*. х3(у" -у) = ж2 — 2.
Найти решения уравнений 582—588, удовлетворяющие указанным начальным условиям.
582.	у" - 2у' + у = 0; у(2) = 1. у'(2) = -2.
583.	у" + у = 4еж; у(0) = 4, у'(0) = -3.
584.	у" - 2у' = 2еж; у(1) = -1, у'(1) = 0.
585.	у" + 2у' + 2у = же-*;	-</(0) = ^(0) = 0.
586.	у'" - у' = 0; у(0) = 3, у'(0) = -1, у"(0) = 1.
58 §11. Линейные уравнения с постоянными коэффициентами
587.	у"' - Зу' -2у = 9е2ж; у(б) = 0, т/'(О) = -3, у"№ = 3.
588.	у1У + у” = 2 cos х: ?/(0) = —2. т/(0) = 1. y"W = т/"(0) = 0.
В задачах 589—600 решить уравнения Эйлера
589.	х2у” — 4ху' + бу = 0.
590.	х2у” — ху' — Зу = 0.
591.	х?у'" + ху' — у = 0.
592.	х2у'" = 2у'.
593.	х2у” — ху' + у = 8ж3.
594.	х2у” + ху' + 4у = Юж.
595.	х^у" — 2ху = 6 In ж.
596.	х2у" — Зху' + бу = Зж2.
597.	х2у" — бу = 5ж3 + 8ж2.
598.	х2у" — 2у = sin In ж.
599.	(ж — 2)2у" — 3(ж — 2)у' + 4у = ж.
600.	(2ж + 3fy"' + 3(2ж + 3)у' - бу = 0.
Применяя различные методы, решить уравнения 601— 611.
601.	у” + 2у' + у = cos/ж.
602.	у” — 2у' + у = жеж sin2 /ж.
603.	у” + 2iy = 8еж sin ж.
604.	у" + 2iy' — у = 8 cos ж.
605.	у”' - 8iy = cos 2ж.
606.	у" - Ц = 31п(-ж).
X
607.	у" + 2у' + у = жеж Н—
§11. Линейные уравнения с постоянными коэффициентами 59
608.	у" + 2у' + Ьу = е ж(сов2 х + tg#).
609.	х2у" -2у=-^-.
610.	х2у" -ху' + у= 111^ + * X In®
611*. y" + y = f(x).
612*. Какие условия достаточно наложить на функцию /(#), чтобы все решения уравнения задачи 611 оставались ограниченными при х —> +оо?
В задачах 613—618 построить линейные однородные дифференциальные уравнения с постоянными коэффициентами (возможно более низкого порядка), имеющие данные частные решения.
613.	ух	= #2еж.	614.	= е2ж cos х.
615.	=	х sin#.	616.	У1	= #еж cos 2#.
617.	yi	= #еж, т/2	= е-ж.	618.	у\	= #, У2 — sin#.
619.	При каких а и b все решения уравнения у” + ау' + + Ъу = 0 ограничены на всей числовой оси —оо < # < +оо?
620.	При каких а и b все решения уравнения у” + ау' + + Ъу = 0 стремятся к нулю при # —> +оо?
621.	При каких а и b уравнение у" + ay' + by = 0 имеет хотя бы одно решение у(х)ф 0. стремящееся к нулю при # +ос?
622.	При каких а и b каждое решение уравнения у" + + ау' + Ьу = 0, кроме решения т/(#) = 0, монотонно возрастает по абсолютной величине, начиная с некоторого #?
623.	При каких а и b каждое решение уравнения у” + + ау' + Ьу = 0 обращается в нуль на бесконечном множестве точек #?
624*. При каких а и b все решения уравнения у" + + ay' + by = 0 удовлетворяют соотношению у = о(е-ж) при # +ос?
625*. Для заданного b > U подобрать такое а, при котором решение уравнения у" + ay' + by = 0 с начальными условиями
60 §11. Линейные уравнения с постоянными коэффициентами
3/(0) = 1, т/(0) = 0 возможно быстрее стремится к нулю при х -л +оо.
626.	При каких к и ш уравнение у” + к2у = sinojf имеет хотя бы одно периодическое решение?
627.	Найти периодическое решение уравнения х + ах + +Ьх = sinojf и нарисовать график зависимости его амплитуды от величины и.
628.	Найти периодическое решение уравнения х + х + + 4ж = и на комплексной плоскости начертить кривую, которую пробегает амплитудный множитель этого решения при изменении о; от 0 до +оо.
629*. Дано уравнение у" + ay' + by = f(x), причем |/(ж)| т (—оо < х < оо), а корни характеристического уравнения А2 < Ai < 0. Найти решение, ограниченное при —оо < х < оо. Показать, что а) все остальные решения неограниченно приближаются к этому решению при х +оо, б) если f(x) периодическая, то это решение тоже периодическое.
Указание. Применить метод вариации постоянных. Нижние пределы полученных интегралов взять бесконечными такого знака, чтобы интегралы сходились.
В задачах 630—632 принять, что при отклонении груза от положения равновесия на расстояние х пружина действует на него с силой кх. направленной к положению равновесия.
630.	Найти период свободных колебаний массы т, подвешенной к пружине, если движение происходит без сопротивления.
631.	Один конец пружины закреплен неподвижно, а к другому прикреплен груз массы т. При движении груза со скоростью v сила сопротивления равна hv. При t = 0 грузу, находившемуся в положении равновесия, сообщена скорость го-Исследовать движение груза в случаях h2 < 4кт и h2 > 4кт.
632.	Решить предыдущую задачу при дополнительном условии, что к грузу приложена еще периодическая внешняя сила / = 6sina;f. Показать, что при любых начальных условиях движение груза будет приближаться к периодическому и найти это периодическое движение (вынужденные колебания).
§11. Линейные уравнения с постоянными коэффициентами 61
633.	На конце упругого стержня укреплена масса т. Другой конец стержня вибрирует так, что его смещение в момент t равно В sincjt. Упругая сила, возникающая в стержне, пропорциональна разности смещений его концов. Найти амплитуду А вынужденных колебаний массы т. Может ли быть А > В? (Массой стержня и трением пренебречь.)
634.	Частица массы т движется по оси О.т, отталкиваясь от точки х = 0 с силой Зтго и притягиваясь к точке х = 1 с силой 4mri, где го и г± — расстояния до этих точек. Определить движение частицы с начальными условиями
ж(0) = 2, ±(0) = 0.
635.	Электрическая цепь состоит из последовательно включенных источника постоянного тока, дающего напряжение V, сопротивления В, самоиндукции L и выключателя, который включается при t = 0. Найти зависимость силы тока от времени (при t > 0).
636.	Решить предыдущую задачу, заменив самоиндукцию L конденсатором емкости С. Конденсатор до замыкания цепи не заряжен.
637.	Последовательно включены сопротивление R и конденсатор емкости С, заряд которого при t = 0 равен q. Цепь замыкается при t = 0. Найти силу тока в цепи при t > 0.
638.	Последовательно включены самоиндукция £, сопротивление R и конденсатор емкости С, заряд которого при t = 0 равен q. Цепь замыкается при t = 0. Найти силу тока в цепи и частоту колебаний в том случае, когда разряд носит колебательный характер.
639.	Последовательно включены источник тока, напряжение которого меняется по закону B=Vsina;t, сопротивление R и самоиндукция L. Найти силу тока в цепи (установившийся режим).
640.	Последовательно включены источник тока, напряжение которого меняется по закону Е = Vsincjt, сопротивление В, самоиндукция L и емкость С. Найти силу тока в цепи (установившийся режим). При какой частоте ш сила тока наибольшая?
62 § 12. Линейные уравнения с переменными коэффициентами
§ 12.	ЛИНЕЙНЫЕ УРАВНЕНИЯ
С ПЕРЕМЕННЫМИ КОЭФФИЦИЕНТАМИ
1.	Большинство задач этого параграфа решается с помощью методов общей теории линейных дифференциальных уравнений (см. [1], гл. V, § 2, § 3 или [4], гл. 2, § 3, § 5) и методов качественного исследования линейных уравнений второго порядка (см. [1], гл. VI, § 2, п. 1, п. 3). К остальным задачам даны указания или ссылки на литературу.
2.	Если известно частное решение линейного однородного уравнения n-го порядка, то порядок уравнения можно понизить, сохраняя линейность уравнения. Для этого в уравнение надо подставить у = yiz и затем понизить порядок заменой z = и.
Чтобы найти общее решение линейного однородного уравнения второго порядка а^х)у” + а\(х)у' + а?(х)у = 0, у которого известно одно частное решение ?/i, можно понизить порядок уравнения указанным выше способом. Однако удобнее воспользоваться формулой Остроградского — Лиувилля:
?/i У2 -JW)<b	_ а1(ж)
/	/	— ut	, fj I./, I —	\,
У1 У2	ao(x)
где yi и y2 — любые два решения данного уравнения.
Пример. Пусть известно частное решение yi = х уравнения (ж2 + 1)у" - 2ху' + 2у = 0.	(1)
По формуле Остроградского — Лиувилля получим
У1 У2
У1 У2
= Се Лж2+17	;
У1У2 - У1У2 = С(х2 + 1).
Так как функция у± известна, то мы получили линейное уравнение первого порядка относительно у2- Проще всего оно решается следующим способом. Разделив обе части уравнения на yl, получим слева производную от дроби уъ/уъ
У2\' _ У1У2 - У1У2 _ С(х2 + 1)
19	9
yi J У1	У1
Так как у\ = ж, то
У2	Г	ж2 + 1 ,	(	1\
— = / С •---------— (1ж + С2 = С I х---) + С2;
yi J X2	\ х)
У2 = С(ж2 - 1) + С2Х.
§ 12. Линейные уравнения с переменными коэффициентами 63
Это — общее решение уравнения (1).
3.	Общего метода для отыскания частного решения линейного уравнения второго порядка не существует. В некоторых случаях решение удается найти путем подбора.
Пример. Найти частное решение уравнения
(1-2Ж2)г/" + 22/' + 42/ = 0,	(2)
являющееся алгебраическим многочленом (если такое решение существует).
Сначала найдем степень многочлена. Подставляя у = хп + ... в уравнение (2) и выписывая только члены с самой старшей степенью буквы ж, получим: —2ж2 • п(п — 1)хп~2 + ... + +4хп + ... =0. Приравнивая нулю коэффициент при старшей степени ж, получим: —2п(п — 1) +4 = 0; п2 — п — 2 = 0. Отсюда ni = 2; корень П2 = —1 не годен (степень многочлена — целое положительное число). Итак, многочлен может быть только второй степени. Ищем его в виде у = ж2 + ах + Ь. Подставляя в уравнение (2), получим (4а + 4)ж + + 2 + 2а + 4Ь = 0. Следовательно, 4а + 4 = 0, 2 + 2а + 4Ь = 0. Отсюда а = — 1, b = 0. Итак, многочлен у = ж2 — ж является частным решением.
4.	При решении задач 738—750 воспользоваться следующими утверждениями, вытекающими, например, из § 7 гл. V книги [5].
Пусть |f(t)|	при to t < оо; с, а = const > 0. Тогда
1) уравнение и" + (1 + f(t))u = 0 имеет два таких линейно независимых решения, что при t +оо
ui(t) = cost + О • u^(t) = sint + О ^^7^ :
2) уравнение и" — (1 — f (t))u = 0 имеет два таких линейно независимых решения, что при t +оо
«1(0 =е< (1 +о > «г(0 =е_< (l + O QQ) •
В задачах 641—662 исследовать, являются ли данные функции линейно зависимыми. В каждой задаче функции рассматриваются в той области, в которой они все определены.
641. х + 2, х - 2.	642. 6ж + 9, 8х + 12.
643. sin ж, cos ж.	644. 1 , X, X2.
645.	4 - х, 2х + 3, 6ж + 8.
64 § 12. Линейные уравнения с переменными коэффициентами
646.	х2 + 2, Зж2 - 1, х + 4.
647.	х2 — х + 3. 2х2 + х, 2х — 4.
648.	еж, е2ж, е3ж.	649. ж, еж, хех.
650.	1, sin2 х, cos2#. 651. sh#, chx, 2 + еж.
652.	1п(.т2), 1пЗж, 7.	653. x, О, еж.
654.	sh;r, ch x. 2еж - 1. Зеж + 5.
655.	2Ж, Зж, 6Ж.	656. sin ж, cos ж, sin2#.
657.	sinх, sin(# + 2), cos(# — 5).
658.	y/x, y/x + 1, y/x + 2.
659.	arctga?. arcctga?. 1.
660.	x2. ж|ж|.	661. ж, |ж|, 2x + л/4;г2.
662.	ж, ж3, |ж3|.
663.	а) Являются ли линейно зависимыми на отрезке [а, Ь] функции, графики которых изображены на рис. 1? б) Тот же вопрос для рис. 2.
664.	Известно, что для функций т/i, .... уп детерминант Вронского в точке х^ равен нулю, а в точке х^ не равен нулю. Можно ли что-нибудь сказать о линейной зависимости (или независимости) этих функций на отрезке [жо, #1]?
665.	Детерминант Вронского для функций ..., уп равен нулю при всех х. Могут ли быть эти функции линейно зависимыми? Линейно независимыми?
§ 12. Линейные уравнения с переменными коэффициентами 65
666.	Что можно сказать о детерминанте Вронского функций т/I, ..., уп, если только известно, а) что они линейно зависимы? б) что они линейно независимы?
667.	Функции ух = х, у2 = ж5, ?/з = |ж5| удовлетворяют уравнению х2у" — Ъху' + Ъу = 0. Являются ли они линейно зависимыми на интервале (—1, 1)? Объяснить ответ.
668.	Доказать, что два решения уравнения у"+ +р(ж)2/' + + q(x)y = 0 (с непрерывными коэффициентами), имеющие максимум при одном и том же значении ж, линейно зависимы.
669.	Даны 4 решения уравнения у'" + ху = 0, графики которых касаются друг друга в одной точке. Сколько линейно независимых имеется среди этих решений?
670.	Пользуясь известным утверждением об интервале существования решения линейного уравнения ([1], гл. V, конец § 1), определить, на каком интервале существует решение данного уравнения с указанными начальными условиями (не решая уравнения): а) (ж + 1)у” — 2у = 0, //(0) = 0, 2/(0) = 2; 6) у" + у tgx = 0, ?/(5) = 1, ?/'(5) = 0.
671.	Могут ли графики двух решений уравнения у^ + + Р1(х)у{п~^ + ... + Рп(х)у = 0 (с непрерывными коэффициентами) на плоскости ж. у а) пересекаться, б) касаться друг друга?
672.	При каких п уравнение задачи 671 может иметь частное решение у = ж3?
673.	Линейное однородное уравнение какого порядка на интервале (0, 1) может иметь такие четыре частных решения: Ух = ж2 - 2ж + 2, 2/2 = (х - 2)2, у3 = х2 + х - 1, 2/4 = 1 - хЧ
В каждой из задач 674—680 составить линейное однородное дифференциальное уравнение (возможно меньшего порядка), имеющее данные частные решения.
674.	1, cos х.	675. х, ех.
676.	Зж, х - 2. ех + 1.
677.	ж2 - Зж, 2ж2 + 9, 2ж + 3.
678.	еж, sha?, сЬж.	679. ж, ж2, ех.
680.	ж, ж3, |ж3|.
66 § 12. Линейные уравнения с переменными коэффициентами
В задачах 681—701 найти общие решения данных уравнений, зная их частные решения. В тех задачах, где частное решение не дано, можно искать его путем подбора, например, в виде показательной функции у± = еаж или алгебраического многочлена у\ = хп + (ixn~r + Ьхп~2 + . ..
681.	(2,т + 1)у” + 4ху' — 4т/ = 0.
682.	х2(х + 1)у" - 2у = 0: У1 = 1 + 1.
683.	ху” — (2,т + 1)у' + (ж + 1)у = 0.
684.	ху” + 2у'-ху = ^ У1 = ^.
685.	у” — 2(1 + tg2 х)у = 0: у± = tga?.
686.	х(х — 1)у” — ху' + у = 0.
687.	(еж + 1)у” - 2у' - еху = 0; yr = ех - 1.
688.	х?у" In х — ху' + у = 0.
689.	у” — у' tg ж + 2у = 0; у± = sin х.
690.	(ж2 — 1)у” + (х — 3)у' — у = 0.
691.	ху” — (ж + 1)у' — 2(х — 1)у = 0.
692.	у” + 4ху' + (4ж2 + 2)у = 0; у± = еаж\
693.	ху” - (2х + 1)у' + 2у = 0.
694.	х(2х + 1)у” + 2(х + 1)у' — 2у = 0.
695.	х(х + 4)у” — (2х + 4)?/ + 2у = 0.
696.	х(х2 + 6)у" — 4(х2 + 3)у' + бху = 0.
697.	(х2 + 1)у” - 2у = 0.
698.	2х(х + 2)у" + (2 - х)у' + у = 0.
699.	ху'" - у” - ху' + у = 0; уА = ж, у2 = еж.
700.	х2(2х — 1)у"' + (4х — 3)ху" — 2ху' + 2у = 0;
У1 =х. у2 = 1/х.
701.	(х2 — 2х + 3)у'” — (х2 + 1)у” + 2ху' — 2у = 0;
Ух =х. у2= ъх.
§ 12. Линейные уравнения с переменными коэффициентами 67
В задачах 702, 703 найти общее решение линейного неоднородного уравнения, если известно, что частное решение соответствующего однородного уравнения является многочленом.
702.	(ж + 1)ху" + (ж + 2)у' - у = ж +
703.	(2ж + 1)у" + (2ж - 1)у' - 2у = ж2 + ж.
В задачах 704, 705, зная два частных решения линейного неоднородного уравнения второго порядка, найти его общее решение.
704.	(ж2 - 1)у” + 4ху' + 2у = 6.т: уг = х. у2 =	•
705.	(Зж3 + хУу" + 2у' — бху = 4 — 12ж2; у± = 2ж, у2 = (х + I)2.
В уравнениях 706—710 линейной заменой искомой функции у = a(x)z уничтожить член с первой производной.
706.	х2у” — 2xyf -I- (х2 + 2)у = 0.
707.	х2у” — 4хуг + (6 — х2)у = 0.
708.	(1 + х2Уу" + ^ху' + 2у = 0.
709.	х2у" + 2х2у' + (ж2 — 2\у = 0.
710.	ху” + у' + ху = 0.
В уравнениях 711—715 заменой независимого переменного t = уничтожить член с первой производной.
711.	ху” — у’ — 4х3у = 0.
712.	(1 + х2)у” + ху' + у = 0.
713.	ж2(1 — х2)у” + 2(.т — х3)у' — 2у = 0.
714.	у” -у’ + е4ху = 0.
715.	2ху” + у’ + ху = 0.
716.	Зная три частных решения у\ = 1, у2 = ж, у% = х2 линейного неоднородного уравнения второго порядка, написать его общее решение.
68 § 12. Линейные уравнения с переменными коэффициентами
717.	Что можно сказать о функции р(ж), если известно, что все решения уравнения у" +р(х)у' + q(x)y = 0 при х —> оо стремятся к нулю вместе со своими первыми производными?
Указание. Воспользоваться формулой Лиувилля.
718.	Доказать, что в случае q(x} < 0 решения уравнения у" -|- р(х)у' + q(x)y = 0 не могут иметь положительных максимумов.
719.	Где могут лежать точки перегиба графиков решений уравнения у" + q(x)y = О?
720.	Могут ли графики двух решений уравнения у” + +	— 0 (функция q(x) непрерывна) располагаться так,
как на рис. 3,а? рис. 3,tf? рис. 3,в? рис. 3,г?
Рис. 3
721.	Доказать, что отношение двух любых линейно независимых решений уравнения у" +р(х)у' + q(x)y = 0 (с непрерывными коэффициентами) не может иметь точек локального максимума.
722.	Доказать, что в случае q(x) > 0 для любого решения уравнения у" + q(x)y = 0 отношение у'(х)1у(х) убывает при возрастании х на интервале, где у(х} 0.
§ 12. Линейные уравнения с переменными коэффициентами 69
723.	Доказать, что в случае q(x) 0 все решения уравнения у" + q(x)y = 0 с положительными начальными условиями у(х$) > 0, у'(хо) > 0 остаются положительными при всех X > Xq.
724.	Доказать, что решение уравнения у" — х2у = 0 с начальными условиями 2/(0) = 1, //'(0) = 0 есть четная функция, всюду положительная.
725*. Доказать, что в случае q(x) 0 краевая задача
y” + q(x)y = O, у(х1) = а, у(х2) = Ъ
при любых а, b и Xi / ж2 имеет единственное решение. Доказать, что это решение — монотонная функция, если b = 0.
726.	Найти расстояние между двумя соседними нулями любого (не тождественно равного нулю) решения уравнения у” + ту = 0, где т = const > 0. Сколько нулей может содержаться на отрезке а х bl
В задачах 727—730, используя результат предыдущей задачи и теорему сравнения (см. [1], гл. VI, § 2, п. 3), оценить сверху и снизу расстояние между двумя соседними нулями любого (не тождественно равного нулю) решения следующих уравнений на заданном отрезке.
727.	у” + 2ху = 0,	20 х 45.
728.	ху” + у = 0,	25 х 100.
729.	у" - 2хуг + (ж + I)2?/ = 0.	4 х 19.
730.	у” - 2еху' + е2ху = 0,	2 х 6.
731*	. Доказать, что любое решение уравнения у" + ху = 0 на отрезке —25 х 25 имеет не менее 15 нулей.
732.	Пусть a?i, х2. ... — расположенные в порядке возрастания последовательные нули решения уравнения у" + +	— 0, гДе ^(ж) > 0; при х < оо функция q(x)
непрерывна и возрастает. Доказать, что — хп < хп — хп_-± (т. е. расстояние между соседними нулями убывает).
733.	В предыдущей задаче обозначим через с конечный или бесконечный предел функции q(x) при х —> оо. Доказать, что lim (ж„+1 - хп) = -к/у/с.
п—>оо
70 § 12. Линейные уравнения с переменными коэффициентами
734*. Пусть у и z — решения уравнений у” + q(x)y = = 0 и г" + Q(x)z = 0 с совпадающими начальными условиями ?/(#o) = г(ж0), у'(хо) = г'(жо) и на интервале (жо, х1) имеем Q(x) > q(x), у(х) > 0, z(x) > 0. Доказать, что на этом интервале отношение z(x)/y(x) убывает.
735*. Пусть выполнены условия задачи 732 и пусть bn= max |?/(ж)|. Доказать, что 61 > 62 >	> • • •
Хп^Х^Хп + А
736*. Пусть в задаче 733 предел с конечный. Доказать, что Ъп В > 0 при п —> оо (в обозначениях задачи 735).
737*. Заменой независимого переменного t = <р(ж) при-вести уравнение ± щ%))4 = 0 к виду + b(t)± у = О, затем избавиться от первой производной заменой у = a(t)u. (Это преобразование называется преобразованием Лиувилля. Во многих случаях оно позволяет привести уравнение у” + + ч{х)У = 0 к уравнению аналогичного вида, но с «почти постоянным» (слабо меняющимися на интервале (^ оо)) коэффициентом при у. Это облегчает исследование асимптотического поведения решения при х -л оо.)
В задачах 738—748 исследовать асимптотическое поведение при х +оо решений данных уравнений, пользуясь преобразованием Лиувилля (см. задачу 737) и утверждениями п. 4 (стр. 77).
738.	у” + х4у = 0.	739.	у"	— х2у = 0.
740.	у" + х2у = 0.	741.	у"	+ е2ху = 0.
742.	ху” ~ У — 0-	743.	у"	— ху = 0.
744.	ху” + 2у' + у = 0.			
745.	у” - 2(ж - 1)у' + х2у =	0.		
746*. у” + (ж4 + 1)у = 0.
747*. (ж2 + 1)у” - у = 0.
748*. х2у” -I- у In2 х = 0.
В задачах 749—750 получить более точное асимптотическое представление решений данных уравнений, применяя два раза преобразование Лиувилля.
§ 13. Краевые задачи
71
749*. у" — 4;с2г/ = 0.
750*. ху" + у = 0.
§ 13. КРАЕВЫЕ ЗАДАЧИ
1. Для отыскания решения краевой задачи
ао(х)у" + ai(x)y' + а2{х}у = f (ж), ж0 х жх,	(1)
ау(хо) + Ру{хо) = 0, уу' (ж1) + ду^хг) = 0	(2)
надо подставить общее решение уравнения (1) в краевые условия (2) и из этих условий определить (если это возможно) значения произвольных постоянных, входящих в формулу общего решения. В отличие от задачи с начальными условиями (задачи Коши), краевая задача не всегда имеет решение.
2. Функцией Грина краевой задачи (1), (2) называется функция G(a\ s), определенная при хо х Ж1, жо < s < Ж1, и при каждом фиксированном s из отрезка [жо, Ж1] обладающая свойствами (как функция от ж):
1)	при ж s она удовлетворяет уравнению
а0(х)у" + а1(ж)?/' + а2(х)у = 0:	(3)
2)	при ж = жо и ж = Ж1 она удовлетворяет заданным краевым условиям (2);
3)	при ж = s она непрерывна по ж, а ее производная по ж имеет скачок, равный 1/«о(з), т. е.
G(s + 0, s) = G(s — 0, s), G'J =Gd +4"V <4) l®=s-|-0	1^=5 — 0 tto(^)
Чтобы найти функцию Грина краевой задачи (1), (2), надо найти два решения ?/1(ж) и у2(х) (отличных от у(х) = 0) уравнения (3), удовлетворяющие соответственно первому и второму из краевых условий (2). Если ?/1(ж) не удовлетворяет сразу обоим краевым условиям, то функция Грина существует и ее можно искать в виде
{ш/1(ж) (жо ж < s),
(5) Ьу2(х) (в Ж Ж1).
Функции а и b зависят от з и определяются из требования, чтобы функция (5) удовлетворяла условиям (4), т. е.
byz(s) = ayi(s), by2(s) = ay'^s) + —J—.
ao(s)
72
§ 13. Краевые задачи
3.	Если функция Грина (7(ж, s) существует, то решение краевой задачи (1), (2) выражается формулой
?/(«) = У G(x, s)f(s)ds.
«О
4.	Собственным значением задачи
а0(х)у" + а1(я)2/' + az(x)y = Ху,	(6)
ау'(хо) + (Зу(хо) = 0, 72/'(ж i) +	= 0	(7)
называется такое число Л, при котором уравнение (6) имеет решение у(х) ^0, удовлетворяющее краевым условиям (7). Это решение у(х) называется собственной функцией.
Найти решения уравнений 751—762, удовлетворяющие указанным краевым условиям.
751.	у"-у = 2х- т/(0) = 0, 7/(1) = -1.
752.	у” + у’ = 1; т/'(0) = 0, т/(1) = 1.
753.	у" - у’ = 0; у(0) = -1, т/'(1) - т/(1) = 2.
754.	у" + у = 1; 7/(0) = 0. у (f) = 0.
755.	у" + у = 1; 7/(0) = 0, т/(тг) = 0.
756.	у" + у = 2х - тг; т/(0) = 0, т/(тг) = 0.
757.	у" -у1 -2у = 0; т/'(0) = 2, т/(+оо) = 0.
758.	у" — у = 1; 7/(0) = 0, у(х) ограничено при х -л +ос.
759.	у” - 2iy = 0; ?/(0) = -1, </(+оо) = 0.
760.	х2у” — бу = 0; ?/(0) ограничено, у(1) = 2.
761.	х2у” — 2ху' + 2у = 0; у(х) = о(х) при ж —> 0, у(1) = 3.
762.	х2у” + Ъху' + Зу = 0: у'(Х) = 3, у(х) = О(ж-2) при х +ос.
763*. При каких а краевая задача у” + ay = 1, 3/(0) = 0, у(1) = 0 не имеет решений?
§ 13. Краевые задачи
73
Для каждой из краевых задач 764—779 построить функцию Грина.
764.	у" = f(x\, т/(0) = 0. т/(1) = 0.
765.	у" + у = f(x); т/(0) = 0, 2/(тг) = 0.
766.	у" + у' = РхУ у(0) = 0, т/(1) = 0.
767.	у"-у = f(x); т/(0) = 0, т/'(2) + 7/(2) = 0.
768*. у" + у = /(ж); 7/(0) = т/(7г), т/(0) = у'^У
769.	х2у" + 2ху' = f(x); 3/(1) = 0, у'(3) = 0.
770.	ху" -у' = f(xy т/(1) = 0, 7/(2) = 0.
771.	х2у" -2у = f(x)-, 7/(1) = 0. 7/(2) + 2т/(2) = 0.
772.	у" = f(x); 7/(0) = 0, т/(ж) ограничено при х —> +оо.
773.	у" + у' = f(x); т/(0) = 0. т/(+оо) = 0.
774.	ху" + у' = f(x); т/(1) = 0, у(х) ограничено при х —> +оо.
775.	у" + 4т/ + Зт/ = /(ж); т/(0) = 0, т/(ж) = О(е“2ж) при х —> +оо.
776.	х2у" + ху' — у = /(ж); т/(1) = 0. т/(ж) ограничено при ж —> +оо.
777.	х2у" + 2ху' — 2у = f(x); т/(0) ограничено, 7/(1) = 0.
778.	у" — у = /(ж), т/(ж) ограничено при ж —> ±оо.
779.	х2у” — 2у = /(ж), у(х) ограничено при х —> О и при х +ос.
780.	При каких а существует функция Грина краевой задачи у" + ау = /(ж), у(0) = 0, у(1) = О?
781*	. Оценить сверху и снизу решение задачи х2у" + + Zxy1 — 2у = f(x), у(х) ограничено при х Ч 0 и ,т ч +оо, и его первую производную, если известно, что 0 f(x) т.
Указание. Записать решение с помощью функции Грина.
74 § 14. Линейные системы с постоянными коэффициентами
В задачах 782—785 найти собственные значения и собственные функции.
782.	у”	= Ау;	у(0) = 0, y(Z)	= 0.
783.	у"	= Ау;	у'(0) = 0, у'(/)	= 0.
784.	у”	= Ау;	у(0) = 0, у'(/)	= 0.
785.	Х2у"	= Ау;	у(1) = 0, у(а)	= 0 (а > 1)
§ 14. ЛИНЕЙНЫЕ СИСТЕМЫ С ПОСТОЯННЫМИ КОЭФФИЦИЕНТАМИ
1. Путем исключения неизвестных систему, вообще говоря, можно свести к уравнению более высокого порядка с одной неизвестной функцией (см. [1], гл. VII, § 1, п. 2 или [4], гл. 3, § 2). Этот способ удобен для решения лишь несложных систем.
Пример. Решить систему х = у + 1, у = 2е* — х. Исключаем у. Из первого уравнения имеем у = х — 1. Подставляя во второе уравнение, получаем х = 2е* — х. Решив это уравнение второго порядка (методами § 11), найдем х = Ci cost + С2 sint + е*. Значит. у = х — 1 = — Ci sint + С2 cost + е* — 1.
2. Для решения системы (где х означает
Ж1 — «цЖ1 -Р ... -Р ttinXn,
хп — ап±х± -|- ... -|- аппхп^
или, в векторной записи, х = Аж, где х — вектор, А — матрица:
надо найти корни характеристического уравнения
«и — л	«12	«1п
«21	«22 — А	«2п
«п1	«п2	• •	 •	«пн А
(2)
§ 14. Линейные системы с постоянными коэффициентами 75
Каждому простому корню А? характеристического уравнения соответствует решение CiVleXit, где Ci — произвольная постоянная, v1 — собственный вектор матрицы А, соответствующий этому Xi.
Если для кратного корня А имеется столько линейно незави-1	г.
симых собственных векторов v , ..., v , какова его кратность, то ему соответствует решение Civ1^ + ... + CkVke,xt.
Если для корня Л кратности к имеется только т линейно независимых собственных векторов, и т < А;, то решение, соответствующее этому Л, можно искать в виде произведения многочлена степени к — т на еЛ*, т. е. в виде1
Х1 = (а + ^+ ... +dtk~™}ex\
(3)
хп = (р + qt + ... + stk m) eXt.
Чтобы найти коэффициенты а, 6, ..., s, надо подставить решение (3) в систему (1). Приравняв коэффициенты подобных членов в левой и правой частях уравнений, получим систему линейных алгебраических уравнений относительно а, 6, ..., s. Надо найти общее решение этой системы. Коэффициенты а, 6, ..., s должны зависеть от к произвольных постоянных, где к — кратность корня Л.
Найдя для каждого Л решения указанного вида и сложив их. получим общее решение системы (1).
Пример. Решить систему
х = 2х + у + z, у = —2.7; — z, z = 2х + у + 2z.	(4)
Составляем и решаем характеристическое уравнение
2 -Л 1	1
—2 -А -1
2	1	2-А
А3 - 4А2 + 5А - 2 = О,
Ai = 2, А2 = Аз = 1.
(5)
Для простого корня Ai=2 находим собственный вектор (а, /3, 7). решая систему
/^ + 7 = 0,
—2а - 2/3 - 7 = 0,	(6)
2q + (3 = 0
ХВ случае к 3 число к — т нельзя уменьшить, а в случае к 4 иногда можно, если известна жорданова форма матрицы А.
76 § 14. Линейные системы с постоянными коэффициентами
(коэффициенты этой системы равны элементам детерминанта (5) при Л = 2). Из (6) находим 2а = —р = 7. Значит, вектор (1, —2, 2) — собственный, и
х = е2<. у = -2е2‘. z = 2e.2t	(7)
— частное решение системы (4).
Для кратного корня Л = 1 сначала определим число линейно независимых собственных векторов. При Л = 1 из (5) получаем матрицу
(1	1	1	\
—2	-1	-1	I	.
2	1	1	/
Ее порядок п = 3, ранг г = 2. Число линейно независимых собственных векторов равно т = п — г = 1. Корень А = 1 имеет кратность к = 2. Так как к > т, то решение надо искать в виде произведения многочлена степени к — тп = 1 на ext. т. е. в виде
х = (а + Ы)е , у = (с + dt)e , z = (f + gt)e .	(8)
Чтобы найти коэффициенты а, 6, ..., подставляем (8) в систему (4) и приравниваем коэффициенты при подобных членах. Получаем систему
6 + d + g=0, 6 = a + c+f,
—26 — d — g = О, d = —2a — c—f,	(9)
26 + d + g = 0, g = 2a + c + f.
Найдем общее решение этой системы. Из двух левых уравнений имеем 6 = 0, g = —d. Подставляя это в остальные уравнения, получаем
0 = а -I- с -I- /. d = —2а — с — f	(10)
(остальные уравнения будут следствиями написанных). Решаем систему (10), например, относительно а и /:
а = —d, f = d — с.
Таким образом, все неизвестные выражены через с и d. Положив с = Ci, d = С2, имеем а = —С2, 6 = 0, / = С2 — Ci, g = —С2. Общее решение системы (9) найдено.
Подставив найденные значения а, 6, ... в (8) и прибавив частное решение (7), умноженное на Сз, получим общее решение системы (4):
х = —С2^ + Сзе2*, у = (Ci + С*2^) е* — 2Сзе2*,
Z = (С2 - С! - c2t) е + 2С3е2*.
§ 14. Линейные системы с постоянными коэффициентами 77
3.	Другой способ решения системы (1). Для любой матрицы существует базис, в котором матрица имеет жорданову форму. Каждой клетке порядка р 1 жордановой формы соответствует серия hi, 7/2,	, hp векторов базиса, удовлетворяющих уравнениям
Ahi = Xhi,hi ф О, Ah>2 = Л/^2 Ч- hi, Ahs = ХЬз +
(11)
Ahp — Xhp —|— hp—j.
Вектор hi называется собственным, a Т^з, • • •, hp — присоединенными. Каждой серии hi, h>2, ..., hp соответствует p линейно независимых решений х1, х2, ..., хр системы х = Ах (верхний индекс указывает номер решения):
= е hi,

/ у.2	»
х3 = I — hi + —Л2 +
(12)
Р_ xtf t’’-1
& ((Р-1)!
tp~	t
... +лЛР-1
Общее число всех таких решений равно сумме порядков всех клеток жордановой формы, т. е. порядку матрицы. Они составляют фундаментальную систему решений системы х = Ах.
Правило для запоминания формул (12). Собственному вектору hi, соответствует решение ж1 = exthi. Если везде отбросить ext, то каждая строка правой части (12) получится интегрированием по t предыдущей строки, причем постоянную интегрирования надо взять равной следующему по порядку вектору серии.
4.	В случае, когда имеются комплексные корни Л, изложенные способы дают выражение решения через комплексные функции. Если при этом коэффициенты системы (1) вещественны, то можно выразить решение только через вещественные функции. Для этого надо воспользоваться тем, что вещественная и мнимая части комплексного решения, соответствующего корню Л = а + fli (/3^0), являются линейно независимыми решениями.
78 § 14. Линейные системы с постоянными коэффициентами
Пример. Решить систему х = 4ж — ?/, у = 5ж + 2у.
Составляем и решаем характеристическое уравнение
4ТА „-1. =0. А2 — 6А + 13 = 0, А = 3 ± 2i. о 2 А
Для корня А = 3 + 2г находим собственный вектор (а, 6):
( (1 — 2г)а — 6=0,
( 5а - (1 + 2г)6 = 0.
Можно взять а = 1, 6 = 1 — 2г. Имеем частное решение х = е(3+2г)*5 у = (1 - 2i)e(3+2i)*.
Так как данная система с вещественными коэффициентами, то решение, соответствующее корню А = 3 — 2г, можно не искать, оно будет комплексно сопряженным с найденным решением. Чтобы получить два вещественных решения, надо взять вещественную и мнимую части найденного комплексного решения. Так как е(з+2«)г _ езг (CQS 2^ _|_ j sin 2f), то
( xi = Ree^3+2*^ = e3t cos2t,
( yi = Re(l — 2г)е^3+2г^ = e3?/(cos2t + 2sin2t),
( X2 = Ime^3+2^f = e,3t sin2t,
( У2 = Im(l — 2г)е^3+2г^ = e3*(sin2t — 2cos2t).
Общее решение выражается через два найденных линейно независимых решения:
х = Cixi + С2Х2 = Ci e3t cos 2t + С2 е3* sin2t,
у = Ciyi + С2у2 = Ci e3t (cos 2t + 2 sin 2t) + C2 e3* (sin 2t — 2 cos 2t).
5.	Чтобы решить систему
r аюж(т) + ацж(т-1) + ... + aimx +
+ 6ю?/(п) + 6цг/(п-1) + ... + biny = 0, а20ж(р) + а21Ж(р-1) + • • • + а,2Рх +
k + 620?/(</) + 62i?/^-1) + ... + b2qy = 0,
не приведенную к нормальному виду, надо составить характеристическое уравнение
аюАт + ацЛт + ... + aim 6юАп + 6цАп + ... + 6in  агоА29 + а21Ар 1 + ... + a2f?	62qAq + 62iAq 1 + ... + 62g
§ 14. Линейные системы с постоянными коэффициентами 79
и найти его корни. После этого решение отыскивается тем же способом, как в п. 2.
Аналогично решаются системы трех и более уравнений.
6.	Частное решение линейной неоднородной системы с постоянными коэффициентами
Xi = anXi + ... + ainxn + Л(£), i = 1, ... - п (13) можно искать методом неопределенных коэффициентов в том случае, когда функции fi(t) состоят из сумм и произведений функций bo + bit + ... + brnt'm, eat, cos/3t, sin/3t. Это делается по тем же правилам, что для одного линейного уравнения с постоянными коэффициентами, см. п. 2 § 11, со следующим изменением. Если fi(t) = Pmi(£)e7\ где Pmi(t) — многочлен степени m?-. то частное решение системы (13) ищется не в виде	, а в виде
Xi =	е1*, г = 1,	(14)
где Qm-^-s(t) — многочлены степени т + s с неизвестными коэффициентами, т = тахт», s = 0, если 7 — не корень характеристического уравнения (2), а если 7 — корень, то s можно взять равным кратности этого корня (или, точнее, s на 1 больше наибольшей из степеней многочленов, на которые умножается е7* в общем решении однородной системы). Неизвестные коэффициенты многочленов определяются путем подстановки выражений (14) в данную систему (13) и сравнения коэффициентов подобных членов.
Аналогично определяются степени многочленов и в случае, когда fi(t) содержат eat cospt и eat sin/3t, а число 7 = a + pi является корнем характеристического уравнения.
Пример. Решить систему
{х = 4ж — у + e3t (t + sint), у = х + 2у + t e3t cos t.
Сначала для однородной системы х = 4ж — у. у = х + 2у находим корни Ai = Л2 = 3 и как в п. 2 отыскиваем общее решение
хо = (Cit + С2) е3*, уо = (Cit + С2 — Ci) e3t.
В системе (15) для функций £е3*, e3*sint, te3tcost числа а + pi соответственно равны 3, 3 + г, 3 + i. Поэтому надо отдельно найти частные решения систем
х = 4ж — у +1 е3\ у = х + 2?/,	(16)
х = 4ж — у + e3t sint, у = х + 2у + t e3t cos t. (17)
80 § 14. Линейные системы с постоянными коэффициентами
Для системы (16) a + fii = 3 = Ai = А2, s = 2, т = 1. Согласно (14), частное решение можно искать в виде
Х1 = (at3 + bt2 + ct + d) e3<, yi = (ft3 + gt2 + ht + j) e3t.
Для системы (17) a + (3i = 3 + i Ф Ai,2, s = 0, m = 1. Частное решение имеет вид
Х2 = (kt + Z) е3* sint + (mt + n) e3t cost.
?/2 = (pt + q) e3t sin t + (rt + s) e3t cos t.
Отыскав значения коэффициентов a. b....общее решение систе-
мы (15) напишем в виде
X = Хо + Ж1 + Ж2, У = Уо + У1 + У2>
7.	Решение неоднородной системы
Xi = ац(к)х! + .. . + Uin(t)xn + fi(t). i = 1.п
можно найти методом вариации постоянных, если известно общее решение однородной системы с теми же коэффициентами a^(t). Для этого в формуле общего решения однородной системы надо заменить произвольные постоянные Ci на неизвестные функции Ci(C). Полученные выражения для Xi надо подставить в данную неоднородную систему, и из этой системы найти C«(t).
8.	Показательной функцией еЛ матрицы А называется сумма ряда
А _ А А2 А3
е -•Б+^ + -^- + ^-+...,	(18)
где Е — единичная матрица. Ряд сходится для любой матрицы А.
Свойства еА:
а)	если А = СМС~\ то еА = С ем С-1;
б)	если АВ = В А. то еА+в = еА -ев = ев • ел:
в)	матрица X(t) = etA удовлетворяет уравнению = АХ; Х(0) = Е.
Методы отыскания еА:
1) Путем решения системы дифференциальных уравнений. В силу свойства в) i-Й столбец матрицы etA есть решение системы уравнений (в векторной записи) х = Ах с начальными условиями ж«(0) = 1, ж/е(0) = 0 при к ф i (xi — i-я координата вектора ж).
§ 14. Линейные системы с постоянными коэффициентами 81
2) Путем приведения матрицы к жордановой форме. Пусть известна такая матрица С, что матрица С-1 АС = М имеет жорда-нову форму, т. е. состоит из клеток Ki. Каждая жорданова клетка имеет вид К = AE+F, у матрицы F все элементы нули, кроме 1-го косого ряда над диагональю. Поэтому Fm = 0, где т — порядок матрицы F, и eF легко найти с помощью ряда (18). Так как еще еХЕ = еЛ Е, то
К XE+F ХЕ F X тр F X F ! = е = е • е = е Е • е =ее
Составив из клеток еКг матрицу ем, найдем еА с помощью свойства а). Доказательства и пример см. в [5], гл. 1, §§ 12-14.
В задачах 786—812 решить данные системы уравнений (х означает и т. д.; для облегчения работы в некоторых задачах указаны корни характеристического уравнения).
786.	| х = 2х + у, 1 у = Зх + ^у.	787.	] х = х-у, [ У = У - 4®.
	х + х — Зу = 0,		| X = х + у,
788. <		789.	
	у - х - у = 0.		[у = 3у -2х.
	| х = х — Зу,		| х + х + Ьу = 0,
790.		791.	
	[ у = Зх + у.		[ у - х - у = 0.
	х = 2х + у,		| х = Зх — у,
792. <		793.	
	у = 4у-х.		[ у = 4ж - у.
			
	х = 2у — Зх,		| х — 5х — Зу = 0,
794. <		795.	
	у = у — 2х.		1 у + Зх + у = 0.
	г х = х + г — у.		f х = х — 2у — г,
796. <	у = х + у - г,	797. {	у = у - х + г,
	г = 2х — у		z = х — г
х = 2х — у + г, у = х + 2у — г, 799. z = х — у + 2г
798.
х = Зх — у + г,
У = x + y + z, z = 4,т — у + 4г (Ат = 1, Л2 = 2, Аз = 3). (Ат = 1, А2 = 2, А3 = 5).
82 § 14. Линейные системы с постоянными коэффициентами
	f х = 4у — 2z — Зх.	Г X — х — у — Z.
800. <	у = z + X. z = §х — &у + bz	801. у = х + у. z = Зх + z
(Ai	= 1, А2 = 2, Аз = -1). г х = 2х + //,	(Ai = l, А2.3 = 1±2г). г х = 2х + 2г — //,
802. <	у = х + Зу — г, z = 2у + 3z — х	803. < у = х + 2г, г = у — 2х — г
	1 = 2, А2,з = 3 ± г). Т х = 4х — у — г,	(Ai = 1, А2?з = ±г). г х = 2х — у — г,
804. <	у = х + 2у — г, z = х — у + 2z	805. < у = Зх — 2у - Зг, г = 2г — х + у
	1 = 2. А2 = Аз = 3). f х — у — 2х — 2z.	(Ai = 0. А2 = A3 = 1). г х = Зх — 2у — г,
806. <	у = х — 2у + 2z. z = Зх — Зу + bz	807. < у = Зх — 4у - 3z, z = 2х — 4у
(А1 :	= 3, А2 = Аз = —1). f х = х — у + г,	(Ат = А2 = 2, Аз = -5) г х = у — 2г — ж,
808. <	у = X + у - Z, z — 2z — у	809. < у = 4х + //, г = 2х + у — г
	1 = А2 = 1, Аз = 2). г х = 2х + у.	(Ai = 1, А2 = Аз = —1) г х = 2х — у — г,
810. <	у = 2у + 4г, Z — х — Z	811. < у = 2х — у — 2г, г = 2г — х + у
	1 = А2 = 0, Аз =3). ( х = 4х — у,	(Ai = А2 = Аз = 1).
812.	у = Зж + у - Z, (Л 1 Z = X + Z	1 = А2 = Аз = 2).
В задачах 813—825 решить системы, не приведенные к
нормальному виду.
813.
х = 2х — Зу, у = х - 2у.
х = Зх + 4//, у = -х - у.
§ 14. Линейные системы с постоянными коэффициентами 83
815.
817.
ж = 2у, У = —2х.
2х — Ъу = 4у — х, Зх — 4у = 2х — у.
816.
818.
819.
х - 2у + у + х - Зу = 0, 4г/ — 2х — х — 2х + Ъу = 0.
г х = Зх — у — z.
у = —х + Зу — г,
z = —х — у + 3z.
х + х + у — 2?/ = О, <
х — у + х = 0.
820.
821.
х — х + 2у — 2у = О, х — х + у + у = ft.
х — 2у + 2х = 0.
Зх + у - Sy = 0.
822.
х + Зу — х = 0.
х + Зу — 2у = 0.
823.	1 х + Ъх + 2у + у = 0, 1 Зх + Ъх + у + Зу = 0.
824.	[ х + 4,т — 2х — 2у — у = 0. 1 х — 4± — у + 2у + 2у = 0.
825.	[ 2'х + 2х + х + Зу + у + у = 0, 1 х + 4,т — х + Зу + 2у — у = 0.
В задачах 826—845 решить линейные неоднородные сис-
темы.	| х = у + 2 е*,	\ х = у — Ъ cos t.
826.	827. < . [ у = X + г.	[ у = 2х + у. ( х = Зх + 2у + 4 e5t.	f х = 2х — 4у + 4 e-2t,
828.	829. [ у = х + 2у.	[у = 2х -2у.
830.	[ х = 4ж + у — e2t,	831 Г х = 2у — х + 1, [ у = у - 2х.	^у = 3у -2х.
832.	f х = Ъх — Зу + 2 e3t,	( х = 2х + у -к е\ к	,	833. \ 1?/ = ж + ?/ + 5 е-г.	1 у = — 2х + 2t.
84 § 14. Линейные системы с постоянными коэффициентами
	х = х + 2у,	[ х = 2х — 4у,
834. <	835. <	
	у = х — 5 sint.	[	= х - Зу + 3 ef.
	х = 2х — у.	f х = ж+2?/-|-16£ ef,
836. <	837.	
	1 у = у — 2х + 18t.	[ у = 2х - 2у.
	f х = 2х + 4у — 8,	х = 2х — Зу,
838. <	839. <	
	[ £ = Зх + бу.	у = х—2у+2 sint.
	I х = х — у + 2 sin t.	Г х = 2х — у,
840.	841. <	
	[ у = 2х - у.	= х + 2 ef.
	I х = 4ж—Зт/+ sint,	Г х = 2х + у + 2ef,
842.	843. <	
	1 у = 2х—у—2 cost.	[ у = X + 2у - 3 e4t.
	I х = х — у + 8t,	I х = 2х — у,
844.	845. <	
	1 у = 5ж - у.	\у = 2у — х — Ье1 sin t.
В задачах 846—850 данные системы решить методом вариации постоянных.
846.
х = 2/+ tg2 t—1, у = -ж +tgt.
847.
х = 2у — х, e3t
у = 4у-3х+-^-^.
848.
849.
х = — 4,т — 2у +
У = 6ж + Зу -
2 ef —1 ’ 3
ef —1
1 х = х-у+----,
cos г,
у = 2х-у.
850.
х = Зх — 2у,
Решить системы 851—866, записанные в векторной форме: х = Ах, где х — вектор, А — данная матрица.
851. х = Ах, А =
852. х = Ах, А =
3
О
1
2
0\
Зу 1\
Оу
§ 14. Линейные системы с постоянными коэффициентами 85
853. х = Ах, А =	\2 /3 —2\
854. х = Ах. А =	
	\4 ~Ч
	/2—1—1
855. х = Ах, А =	1	0-1
	\ 3 -1 —2
	/1—2	2
856. х = Ах, А =	1	4-2
	\ 1	5-3
	/ -1 —2 2
857. х = Ах, А =	1—2—12
	\ -3 —2 3
	/ -3	2 2
858. х = Ах, А =	(—3—11
	\ -1	2 0
	/3-31
859. х = Ах, А =	3-2 2
	\ -1	2 0
	/21-1
860. х = Ах, А =	-10	1
	\ 1 1	0
	/ ° 1 1 \
861. х = Ах, А =	1 ° 1 .
	\ 2 2 1 /
	/ ° 1 1 \
862. х = Ах, А =	110
	\ —1 ° 1 /
	/ —2 1 2 \
863. х = Ах, А =	-i ° 2
	\ —2 0 3 /
	/ ° i -1 \
864. х = Ах, А =	1 ° -i
	\ 2 2 -3 /
	/ 4 2 —2 \
865. х = Ах, А =	1 3 -1
	\ 3 3 -1 /
86 § 14. Линейные системы с постоянными коэффициентами
(2 О -1 \ 1-1 О . 3-1-17
В задачах 867—873 найти
показательную
функцию еА
данной матрицы А.
867-Л=(о -“)
”»• 4=(о
/ -2 —4 \
871. А = I х 2 J.
(2 1 0 \ 0 2 1.
О 0 2 7
868. А =
870. А =
872. А =
0 1
-1 О J’
3 -1 \
2 О J’ О 1 0 \
ООО
0027
В задачах 874 и ЧУ ел.
875 найти dete"4, не вычисляя матри-
/ 1 о
874. А =	-1	2
\ 0 1
3 \	/ 1 4
О . 875. А =	3 1
-1 7	\ 2 1
876.	Тело массы ш движется на плоскости ж, у, притягиваясь к точке (0, 0) с силой а2тг, где г — расстояние до этой точки. Найти движение тела при начальных условиях ж(0) = d, ?/(0) = 0, ±(0) = 0, ?/(0) = v и траекторию этого движения.
877.	Один конец пружины закреплен неподвижно в точке 0, а к другому прикреплен груз массы 3m, соединенный другой пружиной с грузом массы 2т. Оба груза двигаются без трения по одной прямой, проходящей через точку 0. Каждая из пружин растягивается на величину х под действием силы а2тх. Найти возможные периодические движения сис-
темы.
878.	На концах вала закреплены два шкива, моменты инерции которых и 12. При повороте одного шкива относительно другого на любой угол вследствие деформации вала
§ 15. Устойчивость
87
возникают упругие силы с крутящим моментом Kip. Найти частоту крутильных колебаний вала при отсутствии внешних
сил.
879. К источнику тока с напряжением Е = V sincjf последовательно присоединено сопротивление R. Далее цепь разветвляется на две ветви, в одной из которых включена самоиндукция L, а в другой — емкость С (рис. 4). Найти силу тока в цепи (установившийся режим), проходящего через сопротивление R. При какой частоте ш сила тока наибольшая? Наименьшая?
E=Vsina^
Рис. 4
Указание. О составлении дифференциальных уравнений в задачах об электрических цепях см. п. 5 § 11.
880*. Какое условие достаточно наложить на собственные значения матрицы А, чтобы система уравнений (в векторной записи) х = Ах + /(£) имела периодическое решение при всякой непрерывной вектор-функции /(t) периода о;?
Указание. Применив метод вариации постоянных в векторной форме, выразить общее решение через фундаментальную матрицу е*л, функцию f(t) и начальные условия. Воспользоваться условием периодичности.
§15. УСТОЙЧИВОСТЬ
1.	Рассмотрим систему уравнений
— fi(t) • • • •) i — 1, • • • ,	(1)
или, в векторной записи
= f(t, х), х = (®1, ..., хп).	(2)
ГТ	г dfi	.
Пусть все R и —— непрерывны при to t < оо. их к
Решение х = p(t) системы (2) называется устойчивым по Ляпунову, если для любого е > 0 существует такое 6 > 0, что для
88
§ 15. Устойчивость
всякого решения x(t) той же системы, начальное значение которого удовлетворяет неравенству
|«(to) — ¥?(to)| < <5,	(з)
при всех t to выполняется неравенство
|ж(г) - y>(t)| < е.
Если же для некоторого е > 0 такого 6 не существует, то решение ip(t) называется неустойчивым.
Решение называется асимптотически устойчивым, если оно устойчиво по Ляпунову и, кроме того, все решения с достаточно близкими начальными условиями неограниченно приближаются к ip(t) при t +оо, т. е. если из неравенства (3) следует x(t) — <p(t) -4 0 (t +оо).
Наличие или отсутствие устойчивости не зависит от выбора to.
Вопрос об устойчивости данного решения х = ip(t) системы (2) сводится к вопросу об устойчивости нулевого решения y(t) = О другой системы, получаемой из (2) заменой искомой функции х — - sp(O = у-
2.	Исследование на устойчивость по первому приближению. Пусть Xi(t) = 0 (г = 1, ..., п) — решение системы (1). Чтобы его исследовать на устойчивость, надо выделить из функций fi линейную часть вблизи точки х± = ... = хп = О, например, по формуле Тейлора. Полученную систему часто можно исследовать с помощью следующей теоремы.
Теорема Ляпунова. Рассмотрим систему
= anxi + ... + ainxn +	Ж1, ..., жп), i = 1, ..., п, (4)
at
где aik — постоянные, a ipi — бесконечно малые выше первого порядка. точнее, при |ж| < во
Ш ?(ж)|ж|, i = 1, • • •, п, у(х) 0 при |ж|	0,	(5)
где |ж| = д/|ж112 + ... + |яп|2.
Тогда если все собственные значения матрицы (aik), i, k = = 1, ..., п, имеют отрицательные вещественные части, то нулевое решение системы (4) асимптотически устойчиво; если же хоть одно собственное значение имеет положительную вещественную часть, то нулевое решение неустойчиво.
§ 15. Устойчивость
89
Пример. Исследовать на устойчивость нулевое решение системы
х= y/4 + 4^-2ex+v,
у = sin аж + 1п(1 — 4?/), а = const.
Выделяя линейную часть функций по формуле Тейлора, получаем
х = —2ж - у + ^>1(ж, у),
у = ах - 4?/ + ^2(ж, у),
где функции и ^2 равны О(ж2 + у2) и, значит, удовлетворяют условию (5). Находим собственные значения матрицы коэффициентов
-2-Л -1
а —4 — Л
— О, Л + 6Л + 8 + а — О, Л1? 2 — —3 i у/1 — а.
При а > 1 корни комплексные, ReAi,2 = —3 < 0, а при —8 < а 1 корни вещественные отрицательные, значит, в этих случаях нулевое решение асимптотически устойчиво.
При а < — 8 один корень положителен, значит, нулевое решение неустойчиво.
При а = —8 имеем Л1 = О, Л2 = —6 и вопрос об устойчивости не решается с помощью изложенной теоремы.
3.	Исследование на устойчивость с помощью функции Ляпунова. Производной от функции v(t^ Ж1, ... ..., хп) в силу системы (1) называется функция
dt (1) dt dxi	дхп?п’
где fi, ..., fn — правые части системы (1).
Теорема Ляпунова. Если существует дифференцируемая функция г;(ж1, ..., хп), удовлетворяющая в области |ж| < во усло
виям
1) v > 0 при х ф 0. г;(0) = 0.
to,
dv
} dt (1) то нулевое решение системы (1) устойчиво по Ляпунову.
Если вместо условия 2) выполнено более сильное условие
dv dt
— w(x) < 0 при 0 < |ж| < во, t > £0,
а функция w(x) непрерывна при |ж| < во? тпо нулевое решение системы (1) асимптотически устойчиво.
90
§ 15. Устойчивость
Теорема Четаева. Пусть система (1) обладает нулевым решением. Пусть в некоторой области V пространства Ж1, .... хп существует дифференцируемая функция v(xi,....., хп)ч причем
1)	точка х = 0 принадлежит границе области V,
2)	v = 0 на границе области V при |ж| < во,
3)	в области V при t > to имеем v > 0.	w(x) > 0,
функция w(x) непрерывна.
Тогда нулевое решение системы (1) неустойчиво.
Не существует общего метода построения функции Ляпунова v (когда решение системы (1) неизвестно). В ряде случаев функцию Ляпунова удается построить в виде квадратичной формы v = bijXiXj или в виде суммы квадратичной формы и интегралов от нелинейных функций, входящих в правую часть данной системы.
4.	Условия отрицательности всех вещественных частей корней уравнения
аоАп + aiA™ + ... + an—iA + ап = 0, ао > 0,	(6)
с вещественными коэффициентами.
а)	Необходимое условие: все щ > 0. В случае п 2 это условие является и достаточным.
б)	Условие Рауса — Гурвица: необходимо и достаточно, чтобы были положительными все главные диагональные миноры матрицы Гурвица
/ «1	«0	0	0	0	0	..	0 \
«3	а2	«1	«о	0	0	..	0
«5	«4	«3	а2	«1	ао	..	0
\ 0	0	0	0	0	0	ап /
На главной диагонали этой матрицы стоят числа сц, «2, ••• ..., ап. В каждой строке индекс каждого числа на 1 меньше индекса предыдущего числа. Числа ai с индексами г > п или i < 0 заменяются нулями.
Главные диагональные миноры матрицы Гурвица:
Ai = «1, Д2 =
«1 «о
«з &2
Дз =
«1	«о	0
аз	CL2	СИ
«5	«4	«з
(7)
§ 15. Устойчивость
91
в)	Условия Льенара—Шипара. Необходимо и достаточно, чтобы все (ii >0 и чтобы An-i > 0. Ап_з > 0. Ап-5 >0, ..., где А?; те же, что в (7).
Эти условия равносильны условиям Рауса - Гурвица, но удобнее, так как содержат меньше детерминантов.
Пример. При каких а и b корни уравнения Л4 + 2Л3 + аЛ2 + +ЗА + b = 0 имеют отрицательные вещественные части?
Пишем условия Льенара—Шипара:
2
а >
1 0
а 2 b 3
0.
b > 0.
А3= 3
= 6а — 46 — 9 > 0.
Ai = 2 > 0.
0
Отсюда получаем условия b > 0, 6а > 46 + 9.
г)	Критерий Михайлова. Необходимо и достаточно, чтобы на комплексной плоскости точка где f(X) — левая часть (6), при изменении а? от 0 до +оо не проходила через начало координат и сделала поворот вокруг него на угол пъ/Ъ в положительном направлении.
Другая (эквивалентная) формулировка критерия Михайлова: Необходимо и достаточно, чтобы anan_i > 0 и чтобы корни многочленов
р(£) — ап ап—2^ Ч- а/;—4^	...,
</(//) = а„_1 - ап-зг] + ап_5т/2 - . - -были все положительными, различными и чередующимися, начиная с корня £i, т. е.
0 < £1 < 7/1 < £2 < Т]2 < ...
(Заметим, что многочлен (6) при Л = iw равен р(и>2) + га;д(а;2).)
Пример. f(A) = Л5+2Л4+7Л3+8Л2 + 10Л+6. Здесь ап = 6 > 0, an-i = 10 > 0, а многочлены р(£) = 6 — 8£ + 2£2, q(t/) = 10 — 7т/ + т/2 имеют корни £i = 1, £2 = 3, т/i = 2, т/2 = 5. Значит, 0 < £i < т/i < < £2 < т/2. По критерию Михайлова все корни многочлена f(A) имеют отрицательные вещественные части.
5. Условия устойчивости нулевого решения линейной системы с периодическими коэффициентами см. в [5], гл. III, § 16.
Задачи 881—898 решаются с помощью определения устойчивости.
881. Пользуясь определением устойчивости по Ляпунову, выяснить, устойчивы ли решения данных уравнений с указан-
92
§ 15. Устойчивость
ними начальными условиями
a) 3(t — 1)ж = х. ж(2) =0. б) х = 4,т — t2x. ж(0) = 0.
в) х = t — ж, ж(0) = 1.	г) 2tx = х — х3, ж(1) = 0.
В задачах 882—888 начертить на плоскости ж, у траектории данных систем вблизи точки (0, 0) и по чертежу выяснить, устойчиво ли нулевое решение.
882. х = —х, у = —2у.
884.	х = —ж, у = у.
886. х = у, у = — sin х.
883. х = ж, у = 2у.
885.	х = — у. у = 2ж3.
887.	х = у, у = х3(1 + у2
У
Рис. 5
888.	х = — у cos х. у = sin х.
889.	Траектории системы урав-нений	ы = Q(x> у),
где функции Р, Р'х, Р'у, Q, Q'x, Q'y непрерывны, изображены на фазовой плоскости (рис. 5). Что можно сказать о поведении решений при t —> +оо? Является ли нулевое решение асимптотически устойчивым? Является ли оно устойчивым по Ляпунову?
В задачах 890—892 выяснить, является ли устойчивым нулевое решение системы, если известно, что общее решение этой системы имеет указанный вид.
890.	х = Ci cos2 t — С2 е“* у = Cit4 e-t +2С2.
891.	х =	у = (Crf3 + С2) е“*.
1 “h t
892.	х = (Сх - C2t) у =	+ C2.
mi v I- Za \
893.	Доказать, что для устойчивости по Ляпунову нулевого решения уравнения = a(t)x (где функция a(t) непре
93
§ 15. Устойчивость
рывна) необходимо и достаточно, чтобы
t
lim / a(s) ds t->+oo J
0
+oc.
894.	Доказать, что если какое-нибудь одно решение линейной системы дифференциальных уравнений устойчиво по Ляпунову, то устойчивы все решения этой системы.
895.	Доказать, что если каждое решение линейной однородной системы остается ограниченным при t +оо. то нулевое решение устойчиво по Ляпунову.
896.	Доказать, что если каждое решение линейной однородной системы стремится к нулю при t +оо, то нулевое решение асимптотически устойчиво.
897.	Доказать, что если линейная однородная система имеет хотя бы одно неограниченное при t —> +оо решение, то нулевое решение неустойчиво.
898.	Устойчиво ли нулевое решение системы х± = = an(t)xi -I- ai2(t)a?2, х% = a2i(t)xi -h 0.22 (fix?, если известно, что an(t) + «22^) —> 6 > 0 при t -> +оо?
В задачах 899—906 с помощью теоремы Ляпунова об устойчивости по первому приближению исследовать на устойчивость нулевое решение
899.
900.
х = 2ху - х + ?/, у = 5ж4 + у3 + 2х — Зу.
х = х2 + у2 — 2ж, у = Зж2 — х + Зу.
х = еж+2?/ — cos3#,
901.	<	,_____
[ у = д/4 + 8,т — 2 еу .
( х = 1п(4?/ + е-3ж), 902.	____
[ у = 2у — 1 +	— 6ж.
{х = Ы^Зе27 —2 cos х).
у = 2ех-^/8 + 12у.
94
§ 15. Устойчивость
	X = tg(y - ж),
904. <	у = 2У — 2 cos — ж) . ' х = tg(z - у) - 2ж,
905. <	у = >/9 + 12ж-Зе«, „ z = ~iy- ' х = ех — e~3z,
906. <	у = 4z — Ззш(ж + у), z = ln(l + z — Зж).
В задачах 907—912 исследовать, при каких значениях параметров а и b асимптотически устойчиво нулевое решение.
907.	[ х = ах — 2у + х2.	( х = ах + у + х2. У	908.	У	’ [у = х + у + ху.	[ у = х + ау + у .
909. <	X = X + ау + у2.	Г ж = 7/+ sin ж, у = Ьх — Зу — х2.	[ у = ах + by.
911.	1 ж = 2e~x-y/4+(vy,	Г ж = 1п(е+аж)—е®. 1 у = 1п(1 + ж + ay).	\y=bx + tgy.
913. Исследовать, устойчиво ли решение х = — t2. у = t системы
х = у2 — 2ty — 2у — ж, у = 2х + 2t2 + e2t 2у .
914. Исследовать, устойчиво ли решение х = cost, у = = 2sint системы
х = In I х + 2 sin2 - I-,
\	2/2
у = (4 — ж2) cos t — 2х sin2 t — cos3 t.
В задачах 915—922 для данных систем найти все положения равновесия и исследовать их на устойчивость.
х = у — х2 — х,	( х = (х - 1)(у — 1),
915. У 9	916. <! .	М '
у = Зх — х — у.	( у = ху — 2.
§ 15. Устойчивость
95
917.
я = У,
у = 8ш(ж + у).
918.
X = 1п( — X + у2), У = х - у - 1.
ото / * = 3 “ \/4 + ж2 +у,
( У = 1п(ж -з).
( х = еу—ех,
920.	.-----
[ у = Зх + у2 — 2.
{х = 1п(1 + у + sin ж), у = 2 + у/3 sin ж — 8.
{х = — sin?/, _____________________-____ у = 2х + у 1 — Зх — sin у.
В задачах 923—931 исследовать устойчивость нулевого решения, построив функцию Ляпунова и применив теоремы Ляпунова или Четаева.
\ х = х3 — у,	f х = у — х + ху,
923.	,	924. <!	2 \
I у = х + у .	I у = х—у—х —у .
925.
927.
929.
х = 2у3 — х5, У = -х- у3 +у5.
х = у — Зж — ж3.
у = 6х — 2у.
х = —х — ху, у = у3 -х3.
926.
928.
930.
х = ху — х3 + у3, у = х2 - у3.
х = 2у — х — у3, у = х-2у.
X = х — у — ху2, у = 2х-у-у3.
931*
Ч У = /з(®) - А(?/).
где sgn/Дг) = sgnz, i = 1, 2, 3, 4.
В задачах 932—948 исследовать устойчивость нулевого решения, пользуясь известными условиями отрицательности вещественных частей всех корней многочлена, например, условиями Рауса—Гурвица или критерием Михайлова.
96
§ 15. Устойчивость
932.	у"' + у" + у' + 2у = 0.
933.	у'" + 2у” + 2у' + Зу = 0.
934.	yIV + 2у'" + 4у" + Зу' + 2у = 0.
935.	yIV + 2у'" + Зу” + Чу' + 2у = 0.
936.	y1N + 2у"' + бу" + Ьу' + бу = 0.
937.	yIV + 8у'" + 14у" + Збу' + 45у = 0.
938.	yIV + 13у'" + 16у" + 55у' + 76у = 0.
939.	yIV + Зу"' + 26у" + 74у' + 85у = 0.
940.	yIV + 3,1у'" + 5,2у" + 9,8у' + 5,8у = 0.
941.	yv	+ 2yIV	+ 4у'" + бу" + 5у'	+	4у = 0.
942.	yv	+ 2yIV	+ 5у'" + бу" + 5у'	+	2у = 0.
943.	yv	+ 3yIV	+ бу"' + 7у" + 4у'	+	4у = 0.
944.	yv	+ 4yIV	+ 9у'" + 16у" + 19у'	+ 13у = 0.
945.	yv + 4yIV + 1бу'" + 25у" + 13у' + 9у = 0.
946.	yv + 3yIV + 10у'" + 22у" + 23у' + 12у = 0.
947.	yv + 5yIV + 15у'" + 48у" + 44у' + 74у = 0.
948.	yv + 2yIV + 14у'" + Збу" + 23у' + 68у = 0.
В задачах 949—958 исследовать, при каких значениях параметров а и b нулевое решение асимптотически устойчиво.
949.	у"' + ау” + Ьу' + 2у = 0.
950.	у"' + Зу” + ау' + Ьу = 0.
951.	yIV + 2у'" + Зу" + 2у' + ау = 0.
952.	yIV + ау'” + у" + 2у' + у = 0.
953.	ayIV + у'" + у" + у' + by = 0.
954.	yIV + у'" + ау” + у' + by = Q.
955.	yIV + ay'” + 4у" + 2у' + by = 0.
956.	yIV + 2у"' + ay” + by' + у = 0.
97
§ 16. Особые точки
957.	у1У + ау'” + dy” + by' + у = 0.
958.	yw + 2у"' + 4у" + ay1 + by = 0.
Для исследования устойчивости уравнений с периодическими коэффициентами в задачах 959 и 960 надо найти матрицу монодромии и вычислить мультипликаторы, см. [5], гл. III, § 15, § 16.
959.	Исследовать на устойчивость нулевое решение уравнения х + p(t)x = 0, p(t) = а2 (0 < t < тг), p(t) = b2 (тг < t < 2тг), р (t + 2тг) = /)(£), при следующих значениях параметров:
а) а =	0,5,	b	=	0;	б)	а	=	0,5,	5=1;
в) а =	0.5.	b	=	1.5:	г)	а	=	0.75.	5 = 0:
д) а =	1,	5	=	0:	е)	а	=	1,	5 = 1,5.
960.	Исследовать, при каких а и 5 устойчиво нулевое решение системы с периодическими коэффициентами
= A(t)
A(t + 2) = 4(t),
0 J при 0 < t < 1, A(t) = К Q j при 1 < t < 2.
§ 16. ОСОБЫЕ ТОЧКИ
1. Особой точкой системы				
dx (1/	= Р(Ж,	у)	’ = -1'1	(1)
или уравнения	dy dx	=	Q&, у) Р(ж. у)’	(2)
где функции Р и Q непрерывно дифференцируемы, называется такая точка, в которой Р(х, у) = 0, Q(x, у) = 0.
2. Для исследования особой точки системы
dx	_ dy	j
—— = ах + by. —— = ex + dy dt	dt	y
(3)
98
§ 16. Особые точки
или уравнения
dy _ сх + dy
da? ах + by
надо найти корни характеристического уравнения
а — Л Ъ с d — Л
= 0.
(4)
(5)
Если корни вещественные, различные и одного знака, то особая точка — узел (рис. 6,а), если разных знаков — седло (рис. 6,б), если корни комплексные с вещественной частью, отличной от нуля, то особая точка — фокус (рис. 6,в), если чисто мнимые, — центр (рис. 6,г); если корни равные и ненулевые (т. е. Л1 = = Аг -/- 0), то особая точка может быть вырожденным узлом (рис. 6,д) или дикритическим узлом (рис. 6,е), причем дикрити-ческий узел имеет место только в случае системы = аж; = = ау (или уравнения = ^), а во всех остальных случаях при Ai = Д2 ф 0 особая точка является вырожденным узлом.
Если же один или оба корня уравнения (5) равны нулю, то
= 0 и, следовательно, дробь в правой части уравнения (4)
сокращается. Уравнение принимает вид = fc, и решения на плоскости ж, у изображаются параллельными прямыми.
а Ъ с d
Рис. 6
Чтобы начертить траектории (кривые, изображающие решения на плоскости ж, у) в случае узла, седла и вырожденного уз-
§ 16. Особые точки
99
ла, надо прежде всего найти те решения, которые изображаются прямыми, проходящими через особую точку. Эти прямые всегда „	f а b\
направлены вдоль собственных векторов матрицы I 1, состав-
ленной из коэффициентов данной системы (3). В случае узла кривые касаются той прямой, которая направлена вдоль собственного вектора, соответствующего меньшему по абсолютной величине значению Л.
В случае особой точки типа фокус надо определить направление закручивания траекторий. Для этого надо, во-первых, исследовать устойчивость этой точки по знаку Re Л и, во-вторых, определить, в каком направлении вокруг особой точки происходит движение по траекториям. Для этого достаточно построить в какой-нибудь точке (ж, у) вектор скорости (^|, ^), определяемый по формулам (3).
Аналогично исследуется направление движения в случае вырожденного узла.
Пример 1. Исследовать особую точку х = 0, у = 0 системы
х = 2ж, у = х + у.
Составляем и решаем характеристическое уравнение
°	-О, (2 —А)(1 —А) = 0, Л1 = 1, Л2 = 2.
1	1 — л
Корни вещественные, различные и одного знака. Следовательно, особая точка — узел (того же типа, что на рис. 6,а). Для Ai = 1 находим собственный вектор (0, 1), а для А2 = 2 — вектор (1, 1). На плоскости ж, у строим прямые, направленные вдоль этих векторов, а затем кривые, касающиеся в начале координат первой из этих прямых, так как |Л1| < |Л2|, см. рис. 7.
Другой способ построения интегральных кривых. Разделив одно из уравнений (6) на другое, получим уравнение вида (4)
<1г/ ж + у ( Дж 2ж
-г^= о '	( или —=----------
Дж 2ж у ау х + у
100
§ 16. Особые точки
Прямые, проходящие через особую точку, ищем в виде у = кх (а также х = 0). Подставляя в написанные уравнения, находим к = 1. Значит, ?/ = ж и ж = 0 — искомые прямые. Остальные интегральные кривые строятся с помощью изоклин (рис. 7).
Пример 2. Исследовать особую точку уравнения
dy _ 4ж - Зу (1ж х — 2у ’
Находим корни характеристического уравнения
1 ~ А ~2 . = 0; А2 + 2А + 5 = 0; А = -1 ± 2г. 4	—о — Л
Особая точка — фокус. Переходим от уравнения (7) к системе ^=«-2»,	(8)
at	at
Строим в точке (1. 0) вектор скорости (^|,	. В силу (8) он ра-
вен (ж — 2у< 4ж — 3?/). В точке ж = 1. у = 0 получаем вектор (1. 4) (рис. 8,а). Следовательно, возрастанию t соответствует движение по траекториям против часовой стрелки. Так как вещественная часть корней Л равна —1 < 0, то особая точка асимптотически устойчива, следовательно, при возрастании t решения неограниченно приближаются к особой точке. Итак, при движении против часовой стрелки интегральные кривые приближаются к началу координат (рис. 8,б).
Ук	У
Рис. 8
3. Для исследования особой точки более общей системы (1) или уравнения (2) надо перенести начало координат в исследуемую особую точку и разложить функции Р и Q в окрестности этой точки по формуле Тейлора, ограничиваясь членами первого порядка. Тогда система (1) примет вид
Пж1	_ z ч dt/i	_	., ч
—— = аЖ1 + Ьуу + у>(ж1, ?/1),	-г- = СЖ1 + dyr + ^(^i, ?/i),	(9)
at	at
§ 16. Особые точки
101
где Ж1, yi — новые координаты (после переноса), а, 6, с, d — постоянные. Предположим, что для некоторого е > 0
при Ж1^01?л^01
где г = д/ж1 + yf. Очевидно, это условие выполняется (при любом £ < 1), если функции Р и Q в исследуемой точке дважды дифференцируемы. Предположим еще, что вещественные части всех корней характеристического уравнения (5) отличны от нуля. Тогда особая точка Xi = 0, yi = 0 системы (9) будет того же типа, что особая точка системы (3), получаемой отбрасыванием функций и *ф. Далее, угловые коэффициенты направлений, по которым траектории входят в особую точку, для систем (3) и (9) одни и те же (однако прямым у = кх для системы (3) могут соответствовать кривые для системы (9)), а в случае фокуса — направление закручивания траекторий одно и то же.
В том случае, когда для системы (3) особая точка — центр, для системы (9) она может быть фокусом или центром. Для наличия центра достаточно (но не необходимо), чтобы траектории системы (9) имели ось симметрии, проходящую через исследуемую точку. Ось симметрии, очевидно, существует, если уравнение вида (2), к которому можно привести систему (9), не меняется от замены х на —х (или у на — у). Для наличия фокуса необходимо и достаточно, чтобы нулевое решение системы (9) было асимптотически устойчиво при t —> +оо или при t —оо. Исследование на устойчивость можно провести с помощью функции Ляпунова. Это сделать нелегко, так как в рассматриваемом случае функцию Ляпунова часто приходится брать в виде суммы членов второй, третьей и четвертой степеней относительно ж, у.
В задачах 961—978 исследовать особые точки написанных ниже уравнений и систем. Дать чертеж расположения интегральных кривых на плоскости (ж, у).
961.	у' =	2х + у Зх + 4?/ ’	962.	у' =	х — 4у 2у — Зх
963.	у' =	У - У '	964.	у' =	х + 4у 2х + 3?/
965.	у' =	Зх — 4?/ ’	966.	у' =	2ж - gz Х-у ’
967.	у' =	у — 2ж 2у — Зх'	968.	у' =	4?/ — 2х х + у
102
§ 16. Особые точки
969. у' = У-.	970. у' =
{х = Зж.	( х = 2х — у.
972. <!
у = 2ж + у.	I у = X.
{х = х + 3?/,	f х = х.
974. <!
у =	—б.т — Ьу.	[	у = 2х — у.
{х =	— 2х — Ьу.	(	х = Зх + у.
976.
у =	2ж + 2у.	I	у = у - х.
{х =	Зх — 2у.	(	х = у — 2ж,
у = Фу — б.т.	[ у = 2у — 4ж.
В задачах 979—992 найти и исследовать особые точки данных уравнений и систем.
979. у' =	.	980. у' -- у	Зя + 6	у 981. у' = 9 42/2 ~ж2 й. 982. у' у	2ху — Фу — 8	у 983. у' = д2 +г/2 ~2.	984. у' *	х — у	* I X = х2 — у. 985. < у = 1п(1 — х + ж2) — 1пЗ. ( х = 1п(2 — у2).	( х 986. <!	V У 7	987. <! [ у = еж - еу .	1 у о««	= х/ж2-У + 2-2, [ у = arctg(# + ху). 1 X = х2 — у. 989.	9 ( у = X2 - (у - 2)2. .	. у2 - У +1 990. Ь' ~'"	3	’ ^у = х2 - у2.	2х + у х — 2у — 5’ = 2-У х2-у2-1' _ 7/ + у/1 + 2х2 X + у + 1  = (2ж-у)(я-2), ' = ху — 2.
§ 16. Особые точки
103
Г Ж = 1п(1 — 7/+ З/2),
991.	t-------
у = 3 — у ж2 + Ъу.
( х = л/(® - у)2 + з - 2,
992.	2
( у = еУ ~х—е.
Для уравнений 993—997 дать чертеж расположения интегральных кривых в окрестности начала координат.
Указание. В задачах 993—997 особые точки не принадлежат к рассмотренным в начале § 16 типам. Для их исследования можно построить несколько изоклин. Затем надо выяснить, с каких сторон интегральные кривые входят в особую точку.
2	2
993*. у' =	994*. у' =
у х + у	у х2 + у
995*. у' =	996*. у' =
у + х	у — X
997*. у' =
у+ х2
998. Доказать, что если особая точка уравнения
(ах + by) Дж + (тх + пу) dy = 0
является центром, то это уравнение является уравнением в полных дифференциалах. Обратное неверно.
999*. Доказать, что если уравнение предыдущей задачи не является уравнением в полных дифференциалах, но имеет интегрирующий множитель, непрерывный в окрестности начала координат, то особая точка — седло (если ап ф Ьт).
1000*. Пусть в уравнении
ах + by + р(ж, у) сх + dy + #(ж, у)
(1)
функции р и q определены и непрерывно дифференцируемы в некоторой окрестности точки (0,0), а в самой точке
104
§17. Фазовая плоскость
(0,0) р = рх = р'у = q = qx = q’y = 0. Доказать, что если уравнение (1) не меняется от замены у на — у. а корни характеристического уравнения
с — A d а b — А
= 0
чисто мнимы, то особая точка (0.0) — центр.
§ 17.	ФАЗОВАЯ ПЛОСКОСТЬ
1.	О понятиях фазового пространства, фазовой плоскости, автономной системы, траектории см. [1], гл. VII, § 1, п. 4, или [3], § 15, или [4], гл. 3, § 1.
2.	Чтобы построить траектории системы
X=fl(x, у), у = fi(x, у)	(1)
на фазовой плоскости ж, ?/, можно или исследовать непосредственно эту систему, или, разделив одно уравнение на другое, свести ее к уравнению первого порядка
= fajx, У)	/2ч
d.-r /1(ж, у)'
Траектории системы (1) будут интегральными кривыми уравнения (2). Их можно построить или решив уравнение (2) (часто оно решается проще, чем система (1)), или с помощью метода изоклин (§ 1), при этом необходимо исследовать особые точки системы (методами § 16).
Для построения траекторий уравнения х = /(ж, ж) на фазовой плоскости надо от этого уравнения перейти к системе ж = ?/, у = = /(ж, ?/), которая исследуется так же, как система (1).
3.	Предельным циклом называется замкнутая траектория, у которой существует окрестность, целиком заполненная траекториями, неограниченно приближающимися к этой замкнутой траектории при t +оо или при t —оо. Предельный цикл называется устойчивым, если траектории приближаются к нему только при t +оо, неустойчивым — если только при t —оо, полуустойчи-вым — если с одной стороны цикла траектории приближаются к нему при t +оо, а с другой стороны при t —оо. О предельных циклах см. [3], § 28, [2], § 25.
§17. Фазовая плоскость
105
В задачах 1001—1020 для данных уравнений начертить траектории на фазовой плоскости. По чертежу сделать выводы о поведении решений при t +оо.
1001. ж + 4ж = 0.	1002.	ж - х = 0.
1003. х - х + ж2 = 0.	1004.	X - Зж2 = 0.
1005. ж + 2ж3 = 0.	1006.	ж + 2ж3 — 2ж = 0.
1007. ж + еж —1 = 0.	1008.	ж — 2Ж + ж + 1 = 0.
1009. х — sin х = 0.	1010.	х + 2 cos х — 1 = 0.
1011. х - 4ж + Зх = 0. 1012. х + 2х + Ъх = 0.
1013. х — х — 2х = 0.	1014. х + 2х + х2 + х = 0.
1015.	х + х + 2х — х2 =0.
1016.	х + х2 — х2 + 1 = 0.
1017.	х + 2* - х2 = 0.	1018. х + у/х2 + х2 -1 = 0.
1019. ж + 5ж — 41п^ = 0.
1020. х + х + arctg(#2 — 2,т) = 0.
В задачах 1021—1034 начертить на фазовой плоскости траектории данных систем и исследовать особые точки.
1021.	Г х = 2х + у2 - 1,	X = у2 - 4ж2, 1022.	. [ у = 6ж - у2 + 1.	у = ку - 8.
1023.	Г х = 4 — 4,т — 2у.	х = 1 — х2 — у2. 1024. 1 у = ху.	у = 2х.
1025.	\х = 2 + у — х2.	\ х = ху — 4, 1026. < [ у = 2х(х — у).	[у = (х-4)(у-х).
1027.	II II 1 1 N5 1 К5
1028.	[ж = 2(ж - 1)(у- 2), 1 У = У2 - х2.
106
§17. Фазовая плоскость
1029.
1031.
х = (х+у)2 — 1.
У = -у2-х+1.
х = (2х — у)2 — 9, у = (х- 2у)2 - 9.
1030.
х= (2х- у)2 - 9, у = 9 - (ж - 2у}\
1032.
1033.
х = х2 + у2 — б.т — 8?/, у = х(2у — х + 5).
х = х2 - у.
У =	- У)(х - у + 2).
х = х2 + у2 — 5,
У = (ж- 1)(ж + Зт/ — 5).
1035.	Вывести уравнение движения маятника без сопротивления. Для случая, когда все постоянные, входящие в уравнение, равны 1, начертить траектории на фазовой плоскости. Дать физическое истолкование траекториям различных типов.
1036.	Вывести уравнение движения маятника с сопротивлением, пропорциональным квадрату скорости. Дать чертеж траекторий на фазовой плоскости.
Указание. Воспользоваться чертежом, построенным для задачи 1035.
1037.	Вывести уравнение движения маятника, на который действует постоянная сила, равная половине веса маятника и направленная всегда в одну сторону по касательной к дуге окружности, по которой движется маятник.
Приняв постоянные I и g равными 1, нарисовать траектории полученного уравнения на фазовой плоскости. Какие движения маятника изображаются траекториями различных типов?
1038.	Груз массы т прикреплен к пружине. При отклонении груза на расстояние х пружина действует на него с силой кх, направленной к положению равновесия. Сила трения равна / = const и направлена в сторону, противоположную скорости
§17. Фазовая плоскость
107
груза. При t = 0 груз находится на расстоянии h от положения равновесия и имеет нулевую скорость.
Вывести уравнение движения груза. Приняв т = 2, к = 2, f = 1? h = 5, изобразить движение груза на фазовой плоскости.
1039.	Изобразить на фазовой плоскости малые колебания маятника переменной длины, считая, что при движении маятника вверх его длина равна Z, а при движении вниз равна L > I. Во сколько раз увеличится амплитуда за одно полное колебание? (Пример: раскачка качелей.)
Указание. При малых колебаниях считать sin ж « х. Изменение длины маятника происходит мгновенно (скачком), при этом угол отклонения маятника и его момент количества движения относительно оси не испытывают скачков.
Начертить на фазовой плоскости траектории систем 1040—1046, записанных в полярных координатах, и исследовать, имеются ли предельные циклы.
1040.	dr dt	= r(l — г2),	dy = I dt
1041.	dr dt	= r(r — l)(r — 2),	dy = J dt
1042.	dr dt	= r(l — r)2.	dy = J dt
1043.	dr dt	— sinr.	dy = J dt
1044.	dr dt	= r(|r — 1| — |r — 2|	-2r + 3),
1045.	dr	= r sin i	dy = i
	dt	r	dt
1046.	dr	= r(l — r) sin —,	dy = J
	dt	V	/	1 - r’	dt
1047*	. При каких условиях система		
= 1 dt
Ф: = /(г)	= 1,
dt J{ h dt
108	§ 18. Зависимость решения от начальных условий
где функция /(г) непрерывна, имеет предельный цикл? При каких условиях этот цикл устойчив? Неустойчив? Полуустой-чив?
1048*. При каких значениях постоянной а система do?	dr	о
= 1, тт = (Г - 1)(а + sin у>) at	at
имеет устойчивый предельный цикл? Неустойчивый?
Для уравнений 1049—1052 с помощью изоклин построить траектории на фазовой плоскости и исследовать особые точки. По чертежу сделать заключение о поведении решений при t +оо и о возможности существования замкнутых траекторий.
1049.	х + х3 — х + х = 0.
1050.	х + (х2 - 1)ж + х = 0.
1051.	х + х — 2 arctg# + х = 0.
1052.	х + 2* -х + х = Ъ.
1053*. Для уравнения х + 2ах — 6sgn.T + х = 0 (О < а < 1, b > 0) построить траектории на фазовой плоскости и найти точки, в которых предельный цикл пересекает ось Ох.
Указание. Найти зависимость между абсциссами двух последовательных пересечений траектории с осью Ох.
1054. Показать, что уравнение x+F(x)+x = 0, где функция F непрерывна и F(y) >0 при у > 0. F(y) <0 при у < 0. не может иметь предельных циклов на фазовой плоскости.
Указание. Исследовать знак полной производной ^(ж2 + ?/2).
1055*. Пусть /(ж, у) и /' непрерывны, /(0, 0) < 0, а при х2 + у2 > Ъ2 имеем /(ж, у) > 0. Доказать, что уравнение х + f(x. х)х + х = 0 имеет периодическое решение x(t) 0.
Указание. Перейти на фазовую плоскость и исследовать знак полной производной ^(ж2 + у2). Построить кольцо, из которого не может выйти ни одна траектория. Применить теорему 21 из [3].
§ 18. Зависимость решения от начальных условий 109
§ 18.	ЗАВИСИМОСТЬ РЕШЕНИЯ ОТ НАЧАЛЬНЫХ УСЛОВИЙ И ПАРАМЕТРОВ.
ПРИБЛИЖЕННОЕ РЕШЕНИЕ ДИФФЕРЕНЦИАЛЬНЫХ УРАВНЕНИЙ
1.	Рассмотрим систему в векторной записи
(1)
где х = (ж1, ..., жп), f = (fi, ..., fn). Пусть в рассматриваемой области вектор-функция f непрерывна по t. х и удовлетворяет условию Липшица1 по х
Н№ ?/) - № ®)Н ОН?/- ®||-	(2)
Через || || обозначается любая из обычно применяемых норм вектора:
IM = Vh»il2 4- - - - + Ы2,
||ж|| = |Ж11 + • • • + |жп|
ИЛИ ||ж|| = max |ж*|. г = 1, . . . ,хп
Пусть x(t) — решение системы (1), a y(t) — вектор-функция, удовлетворяющая неравенствам
О- Н?/(о) — ®(о)|| s.
Тогда имеет место оценка
1140 -y(t)|| С <5еад +| (евд -1) .	(3)
Это неравенство можно применять для грубой оценки ошибки приближенного решения y(t) системы (1), а также для оценки сверху разности решения x(t) системы (1) и решения y(t) системы = g(t, у), если ||g(t, у) - f(t, ?/)||	Г).
2.	Если в системе уравнений
= fi (/, ®1, ..., хп, ц), i = 1, ..., п	(4)
хЕсли в выпуклой по х области имеем |^^| a (ii j' = 15 • • , п), то в этой области выполнено условие Липшица с к = па.
110	§ 18. Зависимость решения от начальных условий
с начальными условиями
ж«(0) = ««(//), г = 1............п
(5)
р является параметром, функции fi и ai (г = 1.п) непрерывны
и имеют непрерывные производные по ®i, ..., жп, /л то решение имеет непрерывную производную по параметру р. Производные тр- = щ, i = 1, ..., п удовлетворяют линейной системе уравнений
dm	•	-1
—-— = > ——Uj + ——,	г = 1, ..., п.
dt dxj dp
(6)
и начальным условиям tti(O) = аЦ/z), i = 1,	Значения про-
изводных и в формуле (6) берутся при a?i = a?i(t), ..., хп = = xn(t), где Ж1(£), ..., xn(t) — решение системы (4) с начальными условиями (5).
В частности, если положить аь{р) = /z, a«(/z) = const при i к и считать, что все функции Д,	не зависят от //, то из пре-
дыдущего утверждения будет следовать, что для системы (4) с начальными условиями Жг(0) = г = 1, ..., п производные = = щ (г = 1, ..., п) от компонент решения Ж1, ..., хп по начальному условию ak существуют и удовлетворяют системе уравнений
dm _ у-л dfj dt dxj Uj ’
J=i
i = 1, • • - , n,
и начальным условиям г^(0) = 0 при г fc, ttfc(O) = 1.
3.	Если в (4) и (5) функции fi и ai имеют непрерывные производные по ®i, ..., жп, р (вблизи значения р = 0) до порядка т включительно, то решение тоже имеет непрерывные производные по р до порядка тп и, следовательно, разлагается по степеням параметра р по формуле Тейлора:
x(t) = vo(t) + pvi(t) + р2 v2(t) + ... + pmvm(t) + o(pm). (7)
Здесь x и Vi — тг-мерные вектор-функции. Чтобы найти функции Vi(t), можно разложить правые части в (4) и (5) по степеням р, подставить туда разложение (7) и приравнять коэффициенты при одинаковых степенях р. Получим систему дифференциальных уравнений, из которой последовательно определяются vo(t), ...
В случае, когда ft — аналитические функции от Ж1, ... ..., жп, р. решение x(t) разлагается в сходящийся при малых р степенной ряд по р (в силу теоремы об аналитической зависимости
§ 18. Зависимость решения от начальных условий
111
решения от параметра, см. [4], гл. 1, § 6). Коэффициенты этого ряда совпадают с коэффициентами разложения (7).
Изложенный метод можно использовать для отыскания решения дифференциального уравнения при малых р в тех случаях, когда при р = 0 уравнение решается известными методами.
Пример. Разложить по степеням параметра р решение задачи
х = ж2 + 2/zf-1, ж(1) = —1.	(8)
Ищем решение в виде x(t) =	+ pvi(t) + p2V2(t) + ...
Подставляя это в (8) и приравнивая коэффициенты при одинаковых степенях //, получаем систему
Vo = Vq,	-Uo(l) = -1,
vi = 2г?о^1 + 2£-1, ^i(l) = 0,
V2 = 2t?o^2 + ^1,	^2(1) = 0,
Из первого уравнения и начального условия находим vo(t) = —t~\ Подставляя это во второе уравнение, получаем
vi = — 2£-1t;i + 2£-1, vi (1) = 0.
Отсюда
vi(t) = 1 — t~2.
Подставляя найденные и vi в третье уравнение, получаем
V2 = — 2t~1V2 + (1 — £ 2)2, V2 (1) = 0.
Решив это линейное уравнение и воспользовавшись начальным условием, найдем V2(t) = | — | +	~ Следовательно, реше-
ние задачи (8) имеет вид
/хЧ 1 Л 1 А 2 ft 2	8	1 \	( 2ч
t	t2 J p \3 t 3t2 t3)	7
Это разложение можно продолжить дальше тем же способом.
Аналогичным методом можно получать разложения по степеням параметра периодических решений нелинейных уравнений, в частности, уравнений вида
х + а2х = pf(t. х. х. р),	(9)
где функция f периодическая по t. Переходить от уравнения 2-го порядка к системе при этом не нужно. Произвольные постоянные,
112	§ 18. Зависимость решения от начальных условий
возникающие при отыскании vo(t),	., определяются уже не
из начальных условий, а из условий периодичности (см. [4], гл. 2. §8).
В случае, когда правая часть (9) не зависит от £, период решения x(t) заранее не известен. Тогда в уравнении (9) надо перейти от t к новому независимому переменному т = t(1 + bipL + Ьэц2 + ...) и искать решения ж(т) периода 2тг/а. Коэффициент bi обычно определяется из условия существования периодического решения для г?1(т), и т. д. (см. [4], гл. 2, §8).
4.	Если функция /(ж, у) в окрестности точки (жо, уо) аналитическая, т. е. разлагается в ряд по степеням (ж — жо) и (у — у о), то решение уравнения у' = /(ж, у) с начальным условием ?/(жо) = = уо тоже является аналитической функцией, т. е. разлагается в степенной ряд в окрестности точки жо (см. [2], § 18 и [1], гл. II, § 1, п. 6). Аналогичное утверждение справедливо для уравнения ^(п) _ у у\ yin-i)^ с начальными условиями ?/(жо) = уо, у'{хо) = у'о, ..., ?/(”_1)(жо) =
Пример. Найти в виде ряда решение уравнения у” = ху2 — у' с начальными условиями у(0) = 2, yf(0) = 1.
Ищем решение в виде ряда
У = «о + CL1X + О2Х2 + ... = 2 + Ж + О2Х2 +	+ . . . ,	(10)
так как из начальных условий следует, что «о = 2, сц = 1. Подставляя ряд в дифференциальное уравнение, получаем
2а2+6азж+12а4Ж2+ ... =ж(2+ж+а2Ж2 + ... )2 — 1 — 2а2Х—Зазж2 — ...
Представляя правую часть в виде степенного ряда и приравнивая коэффициенты при одинаковых степенях ж в обеих частях уравнения, получаем 2аг = —1, баз = 4 — 2аг, 12а4 = 4 — Заз, ... Отсюда находим
1	5	1
а2 = -_. аз = - а4 =
Следовательно,
О I	1	2 . 5	3	1	4
у = 2 + х — —х +-ж +-ж + ...
Z О о
5.	Для уравнения
М®)»1"’ + Р1(ж)г/(”-1) + • • • +Рп(х)у = 0,	(11)
у которого все р«(ж) аналитические в окрестности точки ж = 0 и Ро(жо) = 0, т. е. коэффициент при старшей производной обращается в нуль в точке жо, решений в виде степенного ряда может не
§ 18. Зависимость решения от начальных условий 113
существовать. В этом случае могут существовать решения в виде обобщенных степенных рядов
а0(х - х0)г + «1(ж - жо)г+1 +	- жо)г+2 + ...	(12)
где число г не обязательно целое (см. [1], гл. VI, § 2, п. 2, или [4], гл. 2, § 7). Чтобы их найти, надо подставить ряд (12) в уравнение (11) и, приравняв коэффициенты при наименьшей степени (ж — жо), найти возможные значения показателя г, а затем для каждого из этих значений г определить коэффициенты а;.
1056.	Оценить, на сколько может измениться при О х 1 решение уравнения у' = х + sin у с начальным условием ?/(0) = ?/о=0, если число уо изменить меньше, чем на 0,01.
1057.	Оценить, на сколько может измениться при 0 t Т решение уравнения маятника ж + sin ж = 0 с начальными условиями ж(0) = 0, ж(0) = 0, если в правую часть уравнения добавить такую функцию </?(£), что |<р(£)|	0,1 (т. е.
если приложить некоторую внешнюю силу).
1058.	Чтобы приближенно найти решение уравнения ж + sin ж = 0, его заменили уравнением ж + ж = 0. Оценить при 0 t 2 возникающую от этого ошибку в решении с начальными условиями ж(0) = 0,25, ж(0) = 0, если известно, что |ж — sin.T| < 0,003 при |ж|	0,25.
В задачах 1059—1063 оценить ошибку приближенного решения на указанном отрезке.
1059.	у' = I -	?/(0) = 1; у = 1 - f, |Ж|	|.
1060.	ж = ж — 2/, у = tx, ж(0) = 1, 2/(0) = 0;
Ж = 1 -h t + у, у = у, \t\ < 0,1.
1061.	у” — х2у = 0, 2/(0) = 1, 2/(0) = 0: у = е®4/12, |ж|	0.5.
1062.	у' = + х, у (0) = 1; у = 1 + х, 0 ж
1063.	у' = 2ху2 + 1, у(0) = 1; у =	|ж|
Указание. Сначала выделить ограниченную область, в которой содержится приближенное решение у и, предположительно, точное решение у. Для этой области оценить постоянную в условии
114	§ 18. Зависимость решения от начальных условий
Липшица, затем оценить \у — у\. С помощью этой оценки проверить, содержится ли у в выделенной области.
В задачах 1064—1073 найти производные по параметру или по начальным условиям от решений данных уравнений и систем.
1064.	у' = у +	+ у2), у(0) = 1;	найти	Ё2/ d/z	1 /1=0
1065.	у' = 2х + цу2, у(0) = н - 1;	найти	d/z	1 /1=0
1066.	у' = У + У2 + ху3, у(2) = у0;	найти	ду дуо	Lo=o
1067.	t = т+^е-ж,Ж(1) = 1;	найти	дх дц	
1068.	= x2+p,tx3, ®(0) = 1+ju;	найти	дх дц	l/z=0
1069.	( х = 4ty2,	ж(0) = 0, 2/ = 1 + 5/хж, 2/(0) = 0;	найти	дх I дц |	/1=0
1070.	J x = xy + t2. ж(1) = Х*о, ( 2у = -у2,	у(1) = у0;	найти	дх дуо	ж0=3 3/о=2
1071.	( х = х + 2/,	ж(0) = 1+/1, ( У = 2ж+/ху2, у(0) = —2;	найти	ду дц	। /1=0
1072.	х — х = (ж+1)2— рьх2: ,т(0) =	ж(0)	= —	1:
найти	дх I °" 1д=1 ‘			
1073.	х = | - |, ж(1) = 1, ж(1) = Ь- найти ff|b=r Указание. При b = 1 решением служит функция х = t.
В задачах 1074—1078 найти 2-3 члена разложения решения по степеням малого параметра ц.
1074.	у' = 4/хж — у2. ?/(1) = 1.
1075.	у' = | - 5/гж, у(1) = 2.
1076.	ху1 = цх2 + In ?/, 2/(1) = 1-
§ 18. Зависимость решения от начальных условий 115
1077.	у' =	- у\ 2/(1) = 1 + 3/z.
1078.	у' = еу~х +цу, ?/(0) = -//.
Для уравнений 1079—1085 с помощью метода малого параметра (см. [4], гл. 2, § 8) найти приближенно периодические решения с периодом, равным периоду правой части уравнения; // — малый параметр.
1079.	х + Зх = 2sint + рьх2.
1080.	х + Ъх = cos 2t + pLX2.
1081.	х + Зх + #3 = 2/z cos t.
1082.	х + х2 = 1 + //sint.
1083.	x + sin x = [i sin 2t.
1084*. x + x = sin3t — sin2t + /z#2: найти лишь нулевое приближение.
1085*. х + х = 6/zsint — х3.
В задачах 1086—1090 с помощью метода малого параметра (см. [4], гл. 2, § 8, п. 4) приближенно найти периодические решения данных уравнений.
1086. х + х — х2 = 0.	1087. х + х + #3 =0.
1088. х + sin# = 0.	1089. х + х = //(1 — #2)#.
1090. х + х = //(# — #3).
В каждой из задач 1091—1097 найти в виде степенного ряда решение, удовлетворяющее данным начальным условиям. Вычислить несколько первых коэффициентов ряда (до коэффициента при #4 включительно).
1091.	у'	= У2 -х;	2/(0) = 1.
1092.	у'		2/(0) = 1.
1093.	у'	= у + хеу;	у(0) = о.
1094.	у'	= 2# + cos у;	у(0) = 0.
1095.	у'	= х2 + ?/3;	2/(1) = 1-
116	§ 18. Зависимость решения от начальных условий
1096. у" = ху’ - у2;	у(0) = 1, </'(0) = 2.
1097. у" = у'2 + ху,	у(0) = 4. у'(0) = -2.
1098*. Построив мажорирующее уравнение (см. [2], § 18), оценить снизу радиус сходимости степенного ряда, представ-ляющего решение уравнения у — у —х с начальным условием 2/(0) = 1.
1099*. Оценить, с какой точностью можно получить при |ж|	0,2 решение уравнения у1 = еу —х2у с начальным услови-
ем ?/(0) = 0, если в степенном ряде, представляющем решение, взять только четыре члена (до щж4 включительно).
В задачах 1100—1109 найти линейно независимые решения каждого из данных уравнений в виде степенных рядов. В тех случаях, когда это легко сделать, сумму полученного ряда выразить с помощью элементарных функций.
1100.	у” — х2у = 0.
1101.	у" - ху' -2у = 0.
1102.	(1 - х2)у" - 4ху' - 2у = 0.
1103.	(х2 + 1)у" + 5xyf + Зу = 0.
1104.	(1 - х)у" - 2у‘ + у = 0.
1105.	(ж2 — х + 1)?/" + (4ж — 2)уг + 2у = 0.
1106.	у" — ху' + ху = 0.
1107.	у” + у sin х = 0.
1108.	ху" + ?/1п(1 — х) = 0.
1109.	у1" - ху" + (х — 2)у' + у = 0.
В задачах 1110—1116 найти те решения данных уравнений, которые выражаются степенными (или обобщенными степенными) рядами.
1110.	ху" + 2у' + ху = 0.
1111.	2х2у" + (Зх — 2х2)у' — (х + 1)у = 0.
§ 18. Зависимость решения от начальных условий 117
1112.	9х2у” - (ж2 - 2)у = 0.
1113.	х2у” — х2у' + (х — 2)у = 0.
1114.	х2у” + 2ху’ — (х2 + 2х + 2}у = 0.
1115.	ху” — xyf — у = 0.
1116.	ху” + у' — ху = 0.
1117*. Найти с точностью до О(ж5) при х —> 0 решение уравнения ху” + у' — ху = 0, линейно независимое с решением, указанным в ответе задачи 1116.
В задачах 1118—1120 указать, имеют ли данные уравнения решение в виде степенного ряда (или обобщенного степенного ряда).
1118.	х2у” + ху' — (ж + 2}у = 0.
1119.	х2у” + ху' + (1 - х)у = 0.
1120.	х2у” + (Зх — l)yf + у = 0.
В задачах 1121—1125 найти в виде тригонометрических рядов (см. [1], гл. VI, § 1, п. 3 или [4], гл. 2, § 7) периодические решения данных уравнений.
1121.	у” - Зу = /(ж),	/(ж) = |ж| при |ж|	7Г,
/(ж + 2тг) = /(ж).
1122.	у” + у' + у = | sinх\.
Указание. Разложение в ряд Фурье правой части уравнения оо
1123 имеет вид ^2 2 п sinn.T.
п = 1
1124. у" — тг2у = f(x), f(x) = ж(1 — х) при 0 х 1, f(x + 1) = /(ж).
1125. у"+9у= £ к=1 к
В задачах 1126—1129 с помощью метода ломаных Эйлера (с итерациями или без них, см. [4], гл. 1, § 6, § 7) найти
118	§ 18. Зависимость решения от начальных условий
приближенно на указанном отрезке решения данных уравнений с указанными начальными условиями. Вычисления вести с двумя или тремя десятичными знаками после запятой с шагом h = 0,2 или h = 0,1.
1126.	у' = у2 + х,
1127.	у1 = 1 + х. у
1128.	у’ =
1129. у1 =
- - у. У
х2
0	х	1;
0	х	1;
0	х	1;
1	х	2:
2/(0) = 0,3.
7/(0) = 1.
7/(0) = 1.
7/(1) = 0.
В задачах ИЗО—1135 с помощью метода Адамса или Штермера (см. [4], гл. 1, § 7) вычислить приближенно решения написанных ниже уравнений на указанном отрезке. Вычисления вести с тремя знаками после запятой. Значения решения
в начальных точках вычислить с помощью степенного ряда.
ИЗО. 7/' = 7/.
1131.	7/' = у2 — X,
1132.	у' = i - х,
1133.	у' = х2 — т/2,
0	х	1:
0	х	1;
7/(0) = 1.
2/(0) = 0,5.
0^х^1; 7/(0) = 1.
1 х 2;
2/(1) = I-
1134.	у” = хт/, 0 х 1;
2/(0) = 1,	т/'(0) = 0.
1135. ху" + т/' + хт/ = 0, 0 х 1; 2/(0) = 1, у'(0) = 0.
Задачи 1136—1140 можно решить, сравнивая наклон поля направлений (определяемого уравнением у' = /(х, т/)) в точках некоторых кривых у = ipi(x) с наклоном этих кривых.
1136*. Оценить сверху и снизу решение уравнения у1 = = 2 + sinx — т/2, 0 х < +оо, 7/(0) = 1. (На плоскости х, у построить полосу а у /3, из которой не может выйти это решение.)
1137*. Оценить сверху и снизу решение уравнения у' = = ^ + 2х, 0 х < +оо, 7/(0) = 1.
1138*. Доказать, что решение уравнения т/'=х — у2 с начальным условием 7/(4) = 2 удовлетворяет неравенствам д/х — 0,07 < т/(х) < д/х при 4 < х < оо.
§ 19. Нелинейные системы
119
1139*. Доказать, что для решения у(х) уравнения у' = = х — у2 с начальным условием у(хо) = Уо, где х()	0, уо О,
имеем
у(х) — \fx 0 при х 4-ос.
1140*. Оценить сверху и снизу то периодическое решение уравнения у' — 2у2 — cos2 5ж, которое лежит в области у < 0.
§ 19. НЕЛИНЕЙНЫЕ системы
1. Систему дифференциальных уравнений можно свести путем исключения неизвестных к одному уравнению (иногда к нескольким уравнениям с одной неизвестной функцией в каждом). Подробнее см. [1], гл. VII, § 1, п. 2, или [4], гл. 3, § 2.
Пример 1. Решить систему уравнений
Решение. Исключаем z из данных уравнений. Из первого уравнения имеем z = ху . Подставляя во второе уравнение, получаем после упрощений
3 //	/	/\2
X у = (у-ху) .
Данная система уравнений (1) приведена к одному уравнению второго порядка. Это уравнение может быть решено методами, изложенными в § 10 (путем понижения порядка). После того как из этого уравнения будет найдено у, следует найти z, пользуясь равенством Z = ху'.
2. При решении системы уравнений путем исключения неизвестных обычно получается уравнение более высокого порядка, поэтому во многих случаях удобнее решать систему путем отыскания интегрируемых комбинаций (см. [1], гл. VII, § 5, п. 2).
Пример 2. Решить систему1
dx _ dy _ dz
xz yz —xy
(2)
1 Система (2) записана в симметрической форме. О симметрической форме системы дифференциальных уравнений см. [1], гл. VII, § 5, п. 1, или [4], гл. 3, § 3.
120
§ 19. Нелинейные системы
Первые две дроби образуют интегрируемую комбинацию. Cocl гр	d?/	X
кращая равенство —— = —- на — и интегрируя, получаем первый
XZ	у Z	Z
интеграл1
- = Сг.	(3)
У
Чтобы найти вторую интегрируемую комбинацию, воспользуемся следующим свойством равных дробей: если	= ... = ^ =
= £, то при любых Ач, &2,	, кп имеем
feim + къаъ + ... + кпап kibi + к2Ь2 + • • • + кпЬп
Пользуясь этим свойством, получаем из (2)
у • Пж + х • dy _ dz d(xy) _ dz y-xz + x-yz —xy 2xyz —xy
d(xy) = —2z dz.
Следовательно,
xy + z2 = C2.
(4)
Очевидно, первый интеграл (3) и первый интеграл (4) независимы. Система решена.
Вместо того чтобы искать вторую интегрируемую комбинацию, можно, воспользовавшись знанием первого интеграла (3), исключить из системы (2) одно из неизвестных, например, х. Из (3) имеем х = С^у. Подставляя во второе из уравнений (2), получаем
&У yz
dz
-Chy2
. Отсюда —Ciydy = zdz; z2 = — Chy2 + C2. Подстав
ляя сюда выражение для Ci из формулы (3), находим еще один первый интеграл: z2 + ху = С2.
В задачах 1141—1160 решить данные системы уравне
ний.
1141. у' = z' =
и Z '	у
1142.
z' = у + 1.
1143. у' = z'
& х
z(y+2z-l)
1144. у' = y2z, z1 = | — yz2.
1145. 2zy' = у2 — z2 + 1, z1 = z + у.
TO первых интегралах см. [1], гл. VII, § 4 или [3], § 23.
§ 19. Нелинейные системы
121
1146 d,T = d?/ = d* Ъу-z У Z
4447 da: _ d?/ _ dz у x z ’
1148. ^- = ^- = y+z	x + z	x + y
1149.	J12L = _2Ц_ =	_d^.
y—X	x+y+z	x — y
1150	—_______— —___—
Z	U X	у '
4454 da; __ dy __ dz __ dtt
y—u z—x	u — y	x — z'
1152 —   —   — z	xz	у
1153.	(1.Т		 z2-y2 ~	dy _ z	dz У ’
1154.	da; 	 dy х ~ у		 dz ~ xy+z	•
1155.	da:   dy xz	yz		 	dz xyy/z	
1156.	da; x+y2+z2	— dy _ У	dz z
1157.	da; 	 x(y+z) ~	dy z(z-y)		 dz ~ y{y-zY
1158.		da;   dy _ x2	xy—2z2 ~		_ dz xz ’
1159.	da; 	 a;(2-?/) —	dy У(у-х)	_ dz y2—xz'
1160.	da: x(y2-z2)	~ y(z	dy		 dz 2+x2)	z(x2 + y2)
В задачах 1161—1163 для данных систем дифференциальных уравнений и данных функций проверить, являются ли соотношения ср = С первыми интегралами этих систем.
1161.	^ = ^, ^ = -ж; <p1=t2+2xy, dt	у ' dt	' г -L	'
9?2 = Ж2 - fy.
1162.	х = ху, у = х2 + у2',	= ж1пу — х2у,
^2 = ^2 “ 2 In X.
1163.	= -ty =	= -d«; m = yz- их.
У	X и	Z '	‘	&
122 § 20. Уравнения в частных производных первого порядка
1164.	Проверить, являются ли независимыми первые интегралы = Ci, = С2 системы
da?	dy	dz
х	у	z
1165*. Доказать, что в области, содержащей особую точку типа узла или фокуса, для системы
= Р(ж. у),	= Q(x. у)
не может существовать первого интеграла вида у) = С с непрерывной функцией </?, ср /е const в сколь угодно малой окрестности особой точки.
1166.	Пусть ж, у} —	^2 (С х, у) = С2 — первые интегралы системы = /i(t, ж, у).	= = /2(С ж, у),
функции /1, /2 и их первые производные по ж, у непрерывны. Пусть в пространстве t, ж, у поверхности ж, у) = 1. </?2(t, ж, у) = 2 имеют только одну общую линию (т. е. пересекаются или касаются друг друга по этой линии). Доказать, что эта линия является интегральной кривой данной системы.
§ 20. УРАВНЕНИЯ В ЧАСТНЫХ ПРОИЗВОДНЫХ ПЕРВОГО ПОРЯДКА
1.	Чтобы решить уравнение в частных производных dz	dz	.	z ч
«177--h ...	+ ап—— — о,	(1)
UX1	dxn
где «1, ..., atl. b зависят от ®i, ..., жп, z, надо написать систему обыкновенных дифференциальных уравнений
da?i _	_ dxn _ dz	, .
a i	ап Ъ
и найти п независимых первых интегралов этой системы
9?1(ж1, .... жп. z) = Ci, 1 .................... >	(3)
, хп, z) = Сп. J
§ 20. Уравнения в частных производных первого порядка 123
Общее решение уравнения (1) в неявном виде записывается так:
Г(</?1, ..., (рп) = 0,	(4)
где F — произвольная дифференцируемая функция.
В частности, если z входит только в один из первых интегралов (3), например в последний, то общее решение можно написать и так:
9?П(Ж1, ..., жп, z) = f ..., 9?п-1),	(5)
где f — произвольная дифференцируемая функция. Разрешив равенство (5) относительно z, получим общее решение уравнения (1) в явном виде.
2.	Чтобы найти поверхность z = z(x, у), удовлетворяющую дифференциальному уравнению
dz	dz
ai(x, у, z)— + а2(х, у, z) — = b(x, у, z)	(6)
и проходящую через данную линию
Ж = w(0, У = ”(*), z =	(7)
надо найти два независимых первых интеграла системы
dr = dy = d?	. .
«1	«2 Ь
В эти первые интегралы
9?1(ж, у. z) = Cl. у?2(ж, у, z) = Съ	(9)
надо подставить вместо ж, г/, z их выражения (7) через параметр t. Получатся два уравнения вида
Ф1(^) = С1, Ф2(0 = С2.	(10)
Исключив из них получим соотношение F(Ci, С2) = 0. Подставив сюда вместо Ci и С2 левые части первых интегралов (9), получим искомое решение.
В том случае, когда в оба уравнения (10) не входит £, тогда линия (7) является интегральной кривой системы (8), т. е. характеристикой уравнения (6), и задача Коши имеет бесконечно много решений (см. [1], гл. VIII, § 3, п. 4).
Пример. Найти общее решение уравнения
XZd~X+yZd-y = -Xy'	(11)
124 § 20. Уравнения в частных производных первого порядка
а также интегральную поверхность, проходящую через кривую
у = ж2, z = х3.	(12)
Решение. Составляем систему уравнений
dx _ dy _ dz
xz yz —xy
и находим ее первые интегралы (см. § 19, пример 2)
- = C1, z2+xy = c2.	(13)
У
Следовательно, общее решение уравнения (11) можно написать в неявном виде
F z2 + ху ) = 0, \2/	7
где F — произвольная функция. Так как z входит только в один из первых интегралов (13), то общее решение можно написать и в явном виде. Мы получим
2 ।	р(х\	(х\
z + ху = J - ; z = ±tf - ] - ху, \yj у
где f — произвольная функция.
Чтобы найти интегральную поверхность, проходящую через линию (12), запишем эту линию в параметрическом виде, например, взяв х в качестве параметра:
X = ж, у = ж2, Z = ж3.
Подставив эти выражения в (13), получим
i = Ci, х6 + х3 = С2. X
Исключив ж, получим
— + — = С 2
£6 cf 2'
Подставив вместо Ci и Съ левые части первых интегралов (13), найдем искомое решение
й)“ + й)1 = ?+^
§ 20. Уравнения в частных производных первого порядка 125
3.	О решении системы двух уравнений в частных производных первого порядка и о решении уравнения Пфаффа см. [1], гл. IX. § 1 и § 2, пп. 1, 2, 3.
Для каждого из уравнений 1167—1188 найти общее решение.
И67.
1168.	(ж + 2у)|| -yff =0.
1169.	x^x+y^y+z^=Q.
1170.	(ж -	+ (у - z)^ + 2z^ = 0.
1171.	у^ + х^- = х — у. и дх ду	&
1172.	еж F +?/2^ = уеж .
дх ' & ду &
1173.	2ж|^ + (у - ж)^ - ж2 = 0.
ох '	' ду
1174.	ху^- — х2	= yz.
& дх	ду &
И75. жЦ+2yff =ж2у + г.
1176.	(ж2 + y2)ff + 2жу^ + z2 = 0.
1177.	2у4^ - жу^ = ж^П.
1178.	ж2^ + y2z^ =х + у.
1179.	yz^-xz^ = ez.
& дх ду
1180.	(г - y)2f^ + xz^ = ху.
1181.	жу|^ + (ж - 2z)f^ = yz.
1182.	y^+z^ = *.
& дх ду х
1183.	sin2 ж|^ + tg z|| = cos2 z.
1184.	(ж + г)|| + (у + г)^ =ж + у.
1185.	(жг + y)f* + (ж + yz)f^ = 1 - z2.
1186.	(у + ^)^ + (г + ж)^ + (ж + у)^=и.
126 § 20. Уравнения в частных производных первого порядка
И87. х^+у^ + (г + и)^ =ху.
1188. (u-X)^ + (u-y)^-z^=x + y.
Найти	решения уравнений 1189—1193. удовлетворяю		
щие указанным условиям.			
1189.	Хуг -Утг = 0; дх и оу	z = 2х	при у = 1.
1190.	^ + (2еж-у)|2 =0: дх	' оу	z = y	при х = 0.
1191.	2/^ -у¥ = 0; v дх & ду	’	z = y2	при х = 1.
1192.	+	+2§“ =0; дх	ду	dz	’	и = yz	при X = 1.
1193.	х^+у^+ху^=Ъ: дх & ду	dz	о и = X	+ у2 при z = 0.
В задачах 1194—1210 найти		поверхность, удовлетворя	
ющую данному уравнению и проходящую через данную ли			
НИЮ.
И94- У2В + хУ^ = х’ х = 0, z = y2.
1195.	ж^| — 2у^| = ж2 + у2; у = 1, z = ж2.
1196.	ж^| + у|| = z — ху, ж = 2. z = у2 + 1.
1197.	tgzf* +y^j = z; у = х, z = x3.
И08- a‘fi “	= z2(x ~ 3У); х = 1, yz + 1 = 0.
1199.	х^-+у^ = z—x2—y2-, у = —2, z = х — х2. 1200. yz^ + xz^ = ху: х = а. у2 + z2 = а2.
1201.	“ ХУ^ = 2жг! х + У = 2’ Vz = 1-
1202.	z|| + (г2 - ж2)|| + х = 0; у = ж2, z = 2ж. 1203. (у -	+ (z - ж)|| = х - у; z = у = -х.
1204.	ж|| + (жг + у)^ = z; x + y = 2z, xz = 1.
1205.	y2^+yz^+z2 = 0; ж - у = 0, ж - yz = 1. 1206. ж|* + z^ = у; у = 2z, х + 2у = z.
§ 20. Уравнения в частных производных первого порядка 127
1207.	(у + 2г2)- 2ж2г|| = ж2; х = z, у = ж2.
1208.	(x-z)^+(y-z)^ = 2z; х-у = 2, г+2ж = 1.
1209.	жу3|| + ж2г2^ = у3г; ж = -г3, у = z2.
1210*. ж|^ + у^ = 2ху; у = ж, z = ж2.
1211.	Найти общее уравнение поверхностей, пересекающих под прямым углом поверхности семейства
г2 = Сху.
1212.	Найти поверхность, проходящую через прямую
у = х. z = 1
и ортогональную к поверхностям
х2 + у2 + z2 = Сх.
1213.	Написать уравнение в частных производных, которому удовлетворяют цилиндрические поверхности с образующими, параллельными вектору (а, 6, с). Найти общее решение этого уравнения.
1214.	Пользуясь результатом предыдущей задачи, найти уравнение цилиндрической поверхности с образующими, параллельными вектору (1,—1,1), и направляющей
х + у + Z = 0. х2 + ху + у2 = 1.
1215.	Написать уравнение в частных производных, которому удовлетворяют все конические поверхности с вершиной в данной точке (а, 6, с), и решить его.
1216.	Найти поверхности, у которых любая касательная плоскость пересекает ось Ох в точке с абсциссой, вдвое меньшей абсциссы точки касания.
В задачах 1217—1219 решить данные системы уравнений
dz
1217.	<
dz k ду
Z
dz d~x=y-Z’ dz
= xz.
dy
1218.
2г
У
128 § 20. Уравнения в частных производных первого порядка
{dz	2
d^ = 2yz~z ’
OZ
—- = XZ.
ду
В задачах 1220—1223 найти поверхности, удовлетворяющие данным уравнениям Пфаффа.
1220.	(ж — у} da? + z dy — х dz = 0.
1221.	3yz da; + 2xz dy + xydz = 0.
1222.	(г + xy) dx — (z + y2 ) dy + у dz = 0.
1223.	(2yz + 3a?) dx + xzdy + xy dz = 0.
ДОБАВЛЕНИЕ
Задачи, предлагавшиеся на письменных экзаменах
В §§ 21—27 содержатся задачи, предлагавшиеся на письменных экзаменах и коллоквиумах на 2-м курсе механикоматематического факультета МГУ в 1992—1996 годах, а также небольшое число задач, дававшихся для подготовки к экзаменам. Исключены самые трудные задачи. Сокращено число задач на решение уравнений стандартными методами (подобные задачи содержатся в предыдущих параграфах этого сборника).
Ниже приводятся для примера три экзаменационные письменные работы (указаны номера задач из §§ 21—27).
Работа 15.05.94 г. состояла из задач
17, 63, 81, 98, 170, 198.
Работа 4.06.94 г. состояла из задач
28, 51, 69, 122, 127, 148, 190.
Работа 18.05.95 г. состояла из задач
22, 56. 70. 129. 135. 194. 216.
На выполнение работы студентам давалось 3 часа. Для получения оценки «отлично» требовалось решить 5-6 задач.
§21. Существование и единственность решения 129
§21. СУЩЕСТВОВАНИЕ И ЕДИНСТВЕННОСТЬ РЕШЕНИЯ
1.	Теоретические вопросы
Вопросы 1—5 рассчитаны на лиц, изучавших доказательство существования решения дифференциального уравнения, основанное на переходе к интегральному уравнению и построении последовательных приближений [1], [2].
1.	Обосновать связь условия Липшица и дифференцируемости.
2.	Изложить общий план доказательства теоремы существования и единственности.
3.	Сформулировать и доказать утверждение о переходе от дифференциального уравнения к интегральному.
4.	Доказать, что последовательные приближения сходятся к непрерывной функции.
5.	Доказать, что предел последовательных приближений есть решение интегрального уравнения.
6.	Сформулировать и доказать утверждение о единственности решения.
7.	Сформулировать теорему о существовании и единственности решения дифференциального уравнения порядка п.
8.	Сформулировать и доказать лемму об интегральном неравенстве.
2.	Существование решения и последовательные приближения
9.	Перейти от уравнения еж у"' + ху = 2уу' к системе нормального вида и при начальных условиях ?/(0) = 1, ?/(0) = 1, ^"(0) = 0 построить два последовательные приближения к решению.
10.	Построить три последовательные приближения у$, у^ У2 к решению задачи
y' = 2t + y2, у(0) = 1.
1 при у < 0
— 1 при у 0?
130	§21. Существование и единственность решения
11.	а) Задачу у' = у2 + ж, т/(1) = 0 свести к интегральному уравнению и построить последовательные приближения ?/0, УЪ У2-
б) Указать какой-либо отрезок, на котором сходятся последовательные приближения, и доказать их равномерную сходимость.
12.	Существует ли решение задачи
у' = f(.y), 3/(0) = о, где /(у) =
Обосновать ответ.
13.	а) Свести задачу
= у е* sins	=	=	= 2
(У - Х)а
к задаче для системы нормального вида.
б)	При каких а существование и единственность решения гарантируется теоремой?
14.	а) Указать все значения параметров а, а, А, при которых теорема существования и единственности гарантирует однозначную разрешимость задачи
3 +1
(at + а)у"' + 2ау" - (а - l)t2ytgt = In -— о С
у(а) = 1, у'(а) = А, у”(а) = а.
б)	На какой максимальный интервал можно продолжить решение этой задачи в случае а = —1, а = —2, А = —3?
15.	Задачу ху' =	у(—2) = 4 свести к интеграль-
ному уравнению, построить последовательные приближения и найти их предел.
16.	При каких начальных условиях существование единственного решения уравнения у'” sin ж + ж ln?/ + tga? = 1 гарантируется теоремой?
§21. Существование и единственность решения 131
3. Применение теоремы единственности
17.	Для уравнения у" =	---1 известны два решения:
/	\2 У
?/1 = 1 + sin ж, у2 = (	+ 11 , проходящие через точку (0, 1).
Как это согласуется с теоремой единственности?
В задачах 18—22 требуется выяснить, при каких п наличие указанных решений у написанных уравнений не противоречит теореме единственности.
18.	у"' = /(t, //, //', //"). / Е С1, решения у± = 1 +1 + nt2. y2 = l/(l-t) (-1 < f < 1/2).
19.	y(n) = f (ж, у, у',	y(n-1)) , / EC1, решения уг =
= 2 cos x, y2 = 2 — x2.
20.	y^ + «1(^)3/™ ^ + ... + an(x)y = 0, все аДж) непрерывны, решение у± = х(ех —1).
21.	Уравнение то же, что в задаче 20, график решения у^ указан на рис. 9
22.	Уравнение то же, что в задаче 20, график решения у± указан на рис. 10.
23.	Сколько решений имеет задача
(а3 - 4а)у"' + (а2 + 2а)у" +у' -2у = х + а, у(1) = 0, у'(1) = 1
в зависимости от значений параметра al 24. Тот же вопрос для задачи
(1 - а2)(ау"' - у") = ау' + у2, у(0) = 2, у'(0) = 4.
132	§21. Существование и единственность решения
25.	Сколько решений имеет задача
= X + у2, у(-1) = а, у'(-1) = О в зависимости от а и п!
26.	Тот же вопрос для задачи
у(”) = 2у2 - а2х, у(1) = 1, у'(1) = а.
27.	Тот же вопрос для задачи
?/(n) = X + 2у' + у2, у(-1) = ln(4 + а),	= 1.
4. Продолжение решений
28.	Существует ли при —оо < х < оо решение задачи у' = е~у sin(e2/), ?/(0) = О?
29.	Для задачи (2 — х2)у' — ху2 = 0, у(хо) = уо, где xq = = у/2 + Зе”1, у0 = —2,
а)	определить максимальный интервал существования решения;
б)	нарисовать график решения.
30.	а) Найти все решения уравнения
у' = ху2 / (к2 — х2}.
б)	Найти непродолжаемое решение этого уравнения с начальным условием у(—д/3) = 1/(1пл/тг2—3—1) и нарисовать его график.
31.	Доказать, что решение задачи у' = х — у2, у(1) = О может быть продолжено на полуинтервал 1 х < оо.
32.	Имеет ли система dx/dt = sin?/, dy/dt = x3 решение, которое нельзя продолжить на интервал —оо < t < оо?
33*. Доказать, что решение задачи у1 = х + у2, т/(0)=0 не продолжается на полуинтервал 0^ж<оо.
34*	. На каком интервале можно гарантировать существование решения задачи
= /(£, х) (t е R, х е Rn. f G С1), ж(0) = I : I , если
|/(t, ж)|	|ж|2? Дать неулучшаемую оценку интервала, общую
для всех таких /(£, ж), и подтвердить неулучшаемость примером.
§22. Общая теория линейных уравнений и систем 133
§ 22. ОБЩАЯ ТЕОРИЯ ЛИНЕЙНЫХ УРАВНЕНИЙ И СИСТЕМ
1.	Теоретические вопросы
35.	Сформулировать теорему существования и единственности решения линейного уравнения порядка п на заданном интервале.
36.	Сформулировать и доказать теорему об общем решении линейной однородной системы.
37.	Дать определение фундаментальной системы решений для линейной системы уравнений и доказать ее существование.
38.	а) Что называется общим решением линейного неоднородного уравнения?
б) Сформулировать теорему об этом решении.
39.	а) Сформулировать основные свойства детерминанта Вронского.
б) Пусть	— детерминант Вронского для скаляр-
ных функций ?/1 (t), ..yn(t) класса Сп. Если PE(t) =0 при а t 6, то можно ли сделать вывод о линейной зависимости данных функций на отрезке [а. Ь]? Обосновать ответ.
40.	а) Дать определение фундаментальной матрицы.
б) Написать фундаментальную матрицу для системы х = у. у = 0.
41.	Как из одной фундаментальной матрицы можно получить другие?
42.	Сформулировать и доказать теорему об оценке решений системы
х = A(t)x (х е Rn).
43.	Сформулировать и доказать теорему существования периодического решения линейного уравнения первого порядка с периодическими коэффициентами.
(Задачи 42 и 43 только для студентов, которым читались эти теоремы.)
134	§22. Общая теория линейных уравнений и систем
2.	Линейные однородные уравнения
44.	а) Написать общий вид линейного однородного уравнения порядка п с переменными коэффициентами. При каких требованиях на коэффициенты это уравнение имеет единственное решение, удовлетворяющее заданным начальным условиям?
б) Пусть эти требования выполнены и известно, что уравнение имеет частное решение yi = хк. Каким может быть порядок уравнения?
45.	а) Сформулировать теорему существования и единственности решения уравнения ут + ai(x)y" + d2(x)yf + аз(х)у — = 0 с начальными условиями.
б) Для какого наибольшего натурального числа т наличие у этого уравнения решения у = (еж — 1)ш не противоречит сформулированной теореме?
46.	Найти два линейно независимых решения уравнения х2у" — 2ху' + 2у = 0 и их детерминант Вронского. Принимает ли он нулевое значение? Как это согласуется с известными свойствами детерминанта Вронского?
47.	Пусть yi(ж), у2(ж) — решения уравнения (х + 2)у" — — Зу' + уу/1 — х = 0 с начальными условиями у± (0) = 1, 2/1(0) = 0, 2/2(0) = 3, 2/1(0) = 2.
а)	Указать интервал, на который их можно продолжить.
б)	Составляют ли они фундаментальную систему?
в)	Чему равен детерминант Вронского этих решений при х = —1?
48.	Пусть	^2(^)5 993 (t) — решения уравнения
— (t + 1)у"' — 2у" + 2t2ytgt = 0 с начальными условиями при t = 1:
= 2,	= 0,	= 0;
v>2 = -1, v>2 = 1, v>2 = -i;
923 =0,	993 = 0, 993 = —2.
а)	Указать интервал, на который можно продолжить эти решения по известной теореме.
б)	Составляют ли они фундаментальную систему?
в)	Найти явное выражение для их детерминанта Вронского на этом интервале.
§22. Общая теория линейных уравнений и систем 135
г)	Решение y(t) с начальными условиями т/(1) = а, у'(1) = 6, y"(V) = с выразить через	^з(^)-
49.	Существует ли такое значение параметра а, при котором детерминант любой фундаментальной матрицы системы
.	.о .	I
— = Ах, X е R , А = I 3
2 \
0
3 /
1
2 О
остается постоянным при изменении t?
50.	Сколько линейно независимых решений, определенных при —оо < t < 00, имеет уравнение t2x = 90ж? Обосновать ответ.
51*. Тот же вопрос для системы tx = 2х, ty = Зу.
52.	Построить линейное однородное уравнение возможно низшего порядка, имеющее на интервале (0,1) такие четыре решения:
7/1 = 1 — у2 = (х — 2)2, у3 = х2 + х — 1, т/4 = ж2 — 2ж + 2.
53.	Известны два решения линейного однородного уравнения 2-го порядка: т/i = х, у2 = х2 — 1. Найти решение с начальными условиями т/(2) = 4, у'(2) = —3.
54.	Известны два частных решения тд = х2 — 2х + 3, у2 = хех +2 линейного однородного уравнения 3-го порядка. Достаточно ли этого для отыскания решения с начальными условиями 7/(0) = 5, т/'(0) = —8, у”(0) = 2? Обосновать ответ.
55.	Для уравнения х\х — 1)у''' + ж2(5 — Зх)у” + х(6х — — 12)т/' + (12 — 6ж)т/ = 0 известны два частных решения: т/i = х, у2 = х3. Найти общее решение.
56.	Для линейного однородного уравнения 3-го порядка известны два частных решения т/i и у2. Описать способ отыскания общего решения.
3. Линейные неоднородные уравнения
57.	Известны два частных решения линейного неоднородного уравнения первого порядка: тд = х, у2 = ех . Найти решение с начальным условием т/(1) = —1.
136	§22. Общая теория линейных уравнений и систем
58.	Известны три частных решения линейного неоднородного уравнения 2-го порядка: у± = ж + 1, У2 = ж —1,2/з = 1 — х2. Найти общее решение этого уравнения.
59.	Известны три частных решения линейного неоднородного уравнения 2-го порядка: у^ = х2, у? = 1 — х, уз = 1 — Зж. Найти решение с начальными условиями у(0) = 2, у'(0) = 0.
60.	Даны три функции: у\ = х + 1, у% = 1 — 2ж, уз = = ж2 — 3. Составить линейное неоднородное уравнение 2-го порядка, которому они удовлетворяют.
61.	Известны два частных решения у± = х — 1 и у% = = (ж2 — ж + 1)/ж уравнения (ж2 — 2х)у” + 4(ж — 1)2/'+ +2у = = 6ж — 6. Найти общее решение.
62.	Известны два частных решения т/i = жеж, 2/2 = = (ж - 2) ех уравнения ху” — (ж + 1)2/' + у = (ж — 1) ех. Найти общее решение.
4.	Краевые задачи
63.	Пусть известно, что уравнение у” + р(х)у' + q(x)y = = 0 с непрерывными на [а, Ь] функциями р(ж) и q(x) не имеет решений 2/(ж) ^0, для которых у(а) = у(Ь) = 0. Доказать, что для любых чисел с, d существует единственное решение, для которого у(а) = с, y(b) = d.
64*. Найти наименьшее положительное число Т такое, что для уравнения ж — 2ж = 8 sin2 t разрешима краевая задача с условиями ж(0) = —1, ж(Т) = —1.
65.	Известно, что при некоторой непрерывной функции У(ж) краевая задача
у" - 2у' + 2у = f(x), у(0) = 2, г/(7г) = —2
имеет решение. Единственно ли это решение?
66.	Найти наименьшее положительное р, при котором краевая задача
у" + РУ = о, 2/(0) = 1, у(1) = 2
не имеет решений.
§ 23. Линейные уравнения и системы	137
67.	Найти наибольшее из таких чисел а, что при каждом р е (1, а) краевая задача
у” + 2у' +ру = 0, 7/(0) = 2, 7/(тг) = 3
имеет решение.
§ 23. ЛИНЕЙНЫЕ УРАВНЕНИЯ И СИСТЕМЫ С ПОСТОЯННЫМИ КОЭФФИЦИЕНТАМИ
1. Отыскание решений
Найти все вещественные решения уравнений 68—71.
68.	х — 2х + х= е* + sint.
69.	х + 4#=(e2t +2) sin 2t.
70.	у" + у = 4# cos x.
71.	у” + у = 5#е-2ж +4sin#.
Указать вид общего решения (в задачах 72 и 73 общего вещественного решения) с неопределенными коэффициентами. Не находить числовых значений коэффициентов.
72.	у'" — 2у” + у' = tef(l + cost) + t.
73.	у" — 4yf + 4у = е2ж(# + sin х).
74.	у" — 2iy = 8еж cos х.
75.	у" — 2iyf — у = 4sin#.
76.	у" + ^iy' — 5у = ех cos 2#.
77.	у'" + Siy = sin#cos#.
2.	Периодические и ограниченные решения
Имеют ли уравнения 78—80 периодические решения?
78.	у"' + у = cost.
138	§ 23. Линейные уравнения и системы
79.	х + х = (sin |)4 .
80.	х — 2х = 8 sin2 t.
81.	При каких ш Е R существует периодическое решение уравнения 'х + 4х = 2 cos o;t?
82.	При каких целых b и с уравнение у'" + b2y' = sinx + + с sin2 х не имеет периодических решений?
83.	а) При каких ojER уравнение у^ + 4?/" + 4?/ = coscjt не имеет периодических решений?
б) Найти все периодические решения в случае о; = 3.
84.	Найти периодическое решение уравнения х+£+25х = = sinojf. Нарисовать график его амплитуды как функцию от
85.	При каких целых а уравнение у" + а2у = sin4xcos2x а) не имеет решений с периодом тг?
б)* имеет только одно решение с периодом тг?
86*. Те же вопросы для уравнения
у" + (а — 1)(а — 2)у' + а2 у = sin2x.
Для каждого из уравнений 87 и 88 выяснить, при каких а Е R все решения этого уравнения не ограничены при — 00 < t < 00.
87.	х + ах = sin2 t.	88. 'х + х = cos at.
89.	При каких а Е R хотя бы одно решение уравнения у"' + у” ~ 2у' = eat + sin 2at
ограничено при t О?
90.	Тот же вопрос для уравнения
у'” + a2 yr = cos at cos 2t.
91.	Найти все значения а, а и /3, при которых задача
х — 2х + Ъх = a е* cos 2t — 17 sin2t. x(0) = q, x(0) = /3
имеет решение, ограниченное при t 0.
§ 23. Линейные уравнения и системы
139
92.	Пусть х = </?(£) и х =	— решения уравне-
ния х — х + 4± — 4,т = 0 с начальными условиями </?(0) = а, </?'(0) = 6, ^"(0) = с; V’(7r) =	— Д = 7- Указать
какие-нибудь числовые значения а, 6, с, а, /3, у так, чтобы </?(£) и были периодическими и линейно независимыми.
3.	Системы уравнений
Решить системы 93—95.
93.
95.
х = у + х — 4, у = 2>у - х.
X = z — х — 7/, у = х - у - г, z = -у.
94.
Ai = 0, ^2,3 = — 1-
96.	При каких матрицах А все вещественные решения системы х = Ах выражаются только через синусы, косинусы и константы?
97.	Для одного частного решения системы х = Ах известна только первая координата: х± = t2 + tsint. Каким может быть порядок матрицы А?
98.	Найти фундаментальную матрицу системы х = Ах, где А = I о о о J , нормированную при t = 0.
99.	Доказать, что для системы х = Ах с вещественной кососимметрической матрицей А нормированная при t = 0 фундаментальная матрица при каждом t является ортогональной.
100.	Найти все вещественные периодические решения системы
х = 2у — х + 2 cos t, у = 4у — 2х + cos t.
101.	Найти решение с периодом тг системы
х = х — у, у = 2х — у + 6 sin2 t.
140	§ 23. Линейные уравнения и системы
102.	а) Найти все вещественные периодические решения системы
х = х — у + 3 sin 2t, у = 2х — у.
б) Найти все решения с периодом тг.
103.	При каких а система
х = у + sin 2t, у = —4,т + a cos 2t
имеет периодическое решение?
104.	Для каких вещественных чисел а и b все решения системы
х — 2у — 4,т + а. у — 2х — у + b
ограничены при t О?
105.	Для каких матриц А каждое решение системы х = Ах ограничено при —оо < t < оо.
4.	Показательная функция матрицы
106. Сформулировать свойства показательной функции матрицы.
В задачах 107—110 найти etA .
107. А=(~2	108. Д=(А
(О 0 1\	/ 2 0 —1 \
0 О О ].	110. А =	0 2	0	.
10 0/	\ 0 о	2 /
111. Найти вектор е%АЬ, если
В задачах 112—114
а) не вычисляя матрицу etA, найти ее детерминант и собственные значения;
§ 23. Линейные уравнения и системы
141
б) найти etA .
( 2
112. А =	“
\ 4
114. А = | ?
113. А = ( J
/1 О О\	2
115.	А = I О О 11. Найти det f etA dt.
\O 1 О/	0
116.	При каких матрицах А имеем ем —> О при t —> +оо?
117.	Найти фундаментальную матрицу системы х = = t~1Ax.
118.	Если А — такая матрица, что еА = Е, то обязательно ли А = О?
119*. Что можно сказать о жордановой форме матрицы А, если
5 etA = к\ к=0
120*. Если при всех t матрица etA симметрическая, то обязательно ли матрица А симметрическая?
121*. Если otA etB = е*(л+в), то обязательно ли АВ = В А?
122*. Если матрица etA ортогональная при каждом tER, то обязательно ли А* = —А?
5. Линейные системы с периодическими коэффициентами
123.	Что называется мультипликатором системы х = = A(t)x с периодической матрицей A(t)?
124.	Какому условию должны удовлетворять мультипликаторы линейной системы для того, чтобы все ее решения стремились к нулю при t —> +оо?
125.	Найти мультипликатор для уравнения х = (а + + sin2 t)x.
142
§ 24. Устойчивость
126*. При каких значениях параметра а Е R уравнение х = (a + sin2t)# + l имеет ровно одно периодическое решение?
127*. Пусть матрица A(t) имеет период Z, и ||A(t)|| а при всех t. Доказать, что для системы х = A(t)x модули мультипликаторов не превосходят еаТ.
§24. УСТОЙЧИВОСТЬ
1.	Теоретические вопросы
128.	Дать определение устойчивости по Ляпунову.
129.	Сформулировать и доказать теорему об устойчивости при наличии функции Ляпунова v(x).
130.	Сформулировать теорему об устойчивости по первому приближению.
131.	Сформулировать необходимые и достаточные условия устойчивости по Ляпунову нулевого решения системы х = Ах (х Е Rn, матрица А постоянная).
132.	Доказать, что если одно решение линейной системы устойчиво, то устойчиво каждое решение этой системы.
133.	Какому необходимому и достаточному условию должна удовлетворять матрица А, чтобы для любой непрерывной функции h(t) каждое решение системы х = Ах + h(t) было устойчивым по Ляпунову?
134.	а) При каких матрицах А система х = Ах имеет более одного положения равновесия?
б) При каких дополнительных предположениях все эти положения равновесия устойчивы?
135.	Система х = Ах, где х Е R3, А — постоянная матрица, имеет частное решение, у которого известна только первая координата: х± = e-t -hcost. Устойчиво ли нулевое решение?
136.	Система х = Ах (х Е R4) имеет частное решение, у которого известны только две координаты: х± = sint + 2cost, x2 = cos 2t. Устойчиво ли нулевое решение?
137.	Если для системы х = Ах (х Е Rn) нулевое решение неустойчиво, то обязательно ли оно неустойчиво для каждой системы вида х = Ах + <р(ж), где Е С1, (р(х) = о(|ж|) при х О?
§ 24. Устойчивость
143
138*. Пусть /(t, х) Е С1, xERn и пусть разность каждых двух решений уравнения х = /(£, х) стремится к нулю при t -hoc. Следует ли отсюда при каком-либо п, что всякое решение этого уравнения асимптотически устойчиво?
2.	Исследование устойчивости конкретных систем
Для уравнений 139—144 и систем 145—147 найти положения равновесия и исследовать их на устойчивость.
139.	х	=	—х2.	140.	х	=	sin х — х.
141.	х	=	—х sin2 х.	142. х =	—х sin2 t.
143.	x	=	x sin3 t.	144.	x	=	^-j-.
145.	x	=	у, у = —x3.	146.	x	=	у, у = Зж2 — 2х.
147.	х = у — х + (у — ж)2, у = 0.
В задачах 148—155 выяснить, при каких значениях параметра а нулевое решение является
а)	асимптотически устойчивым;
б)	устойчивым, но не асимптотически;
в)	неустойчивым.
х = у.
148.
•	Ч 2
у = —ау — х — а х,
149.
х = ах + у + (а + 1)ж2, у = х + ау.
150.
х = ах + a sin у,
•	Ч 2
у = ах — а у.
151.
х = у.
у = — ж(1+ж4)—ау.
152.
153.
х = Зу2 - ау, у = 2х + (2 - а)у.
154.
' .	2	2
х = у — ах — у , <
У = -(а + 1)а? - ау.
144
§25. Фазовая плоскость
J х = -ах + (а - 1)у,
155.	<
[у = х + ау .
156.	а) При каких « G R существуют ограниченные при —оо < t < оо решения системы
х = 2у — 4,т + 1. у = 2.т — у + а.
Найти все такие решения.
б)	Устойчивы ли они?
157.	Устойчиво ли решение системы
х = х — у, у = 2х — у + 6 sin2 t, имеющее период тг?
В задачах 158—160
а)	найти все значения параметра а Е R, при которых все решения уравнения неограничены при t 0 (не требуется отыскивать решения);
б)	выяснить, являются ли эти решения устойчивыми или асимптотически устойчивыми.
158.	х + ах = sin2 t. 159. 'х -h х = cos at.
160.	х + ах = cos at.
§25. ФАЗОВАЯ ПЛОСКОСТЬ
1.	Траектории линейных систем
161.	При каких соотношениях между коэффициентами а, 6, с, d особая точка системы х = ах + by, у = ex + dy является а) седлом, б) узлом?
162.	При каких а, b, с, d для каждого решения системы х = ах + by. у = сх + dy полярный угол точки (x(t). y(t)) возрастает при увеличении t?
В задачах 163—165 определить тип особой точки и нарисовать траектории системы на плоскости х, у.
§25. Фазовая плоскость
145
163.
165.
х = х + Зу, у = 5у- х.
х = у + х — 4, у = Зу - х.
164.
х = х — Ъу, У = 5х- 5у.
166.	При каких а особая точка системы х = а(х + у), у = а2 у является седлом?
167.	а) Может ли траектория системы
х = 2у — х. у = Зх — 2у
из точки (—а2 — 1, —1) попасть в точку (1, а2 + 1)?
б) Устойчиво ли положение равновесия?
168.	а) определить тип особой точки и нарисовать траектории системы
х = ах — у, у = Ъу + х
при а = —2, Ъ = —3.
б) На плоскости параметров а, Ъ указать такую область, что при любых (а, Ь) из этой области вторая компонента y(t) любого решения указанной выше системы имеет бесконечно много нулей при t 0.
169.	Рассматривается система
х = а2х — у, у = Ъх — (3 + 2а)у.
а)	Будет ли нулевое решение системы при а = 1 асимптотически устойчивым? Обосновать ответ.
б)	Нарисовать траектории системы при а = —3.
в)	Существует ли такое значение aER, при котором траектории — замкнутые кривые?
В задачах 170—173 исследовать
а)	при каких значениях параметра aER нулевое решение асимптотически устойчиво и при каких — устойчиво;
б)	при каких значениях параметра а 6 R особая точка — седло? узел? фокус?
в)	при указанном значении а дать чертеж траекторий.
146
§25. Фазовая плоскость
)х = х + ау. 1
« = I-
У — ах + у;
{х = ах + у. х а =
у = ау — (2а + 1)ж;
{х = 2ах + у.
а = 1.
у = ау — 2шг;
{х = х + (2 — а)у.
V	а = 4.
У = ах - 3?/;
2. Траектории нелинейных систем
174.	Найти и нарисовать траектории системы
х = ж3 — Зху2. у = Зх2у — у3.
175.	Имеет ли уравнение х + х5 = 0 ненулевые решения, определенные при —оо < t < оо?
176.	Имеются ли у уравнения х = 4,т — 4ж3 неограниченные решения?
177.	Перейти от уравнения х + ах + х — х3 = 0 к автономной системе двух уравнений. Для этой системы
а)	найти особые точки;
б)	указать значения а, при которых все эти точки неустойчивы;
в)	существует ли значение а, при котором ровно две особые точки устойчивы?
178.	Для уравнения х + 4,т — 6ж2 = О
а)	найти уравнение у = ф(х) траектории, проходящей через точку (1,0);
б)	нарисовать эту траекторию, учитывая значение предела lim -;
ж-юо ж’
в)	найти решение данного уравнения с начальными условиями ж(0) = 1, ж(0) = 0.
179.	Для уравнения х = —гл'(ж), где и(х) = —ж4 + х2 — 1, а) дать чертеж траекторий на фазовой плоскости;
§25. Фазовая плоскость
147
б)	найти особые точки и исследовать их на устойчивость;
в)	найти наклоны сепаратрис и периоды малых колебаний;
г)	добавить +ах в левую часть уравнения и для а > О исследовать типы особых точек полученного уравнения.
180.	Для уравнения х = 2х - 2ж3 провести такое же исследование, как в предыдущей задаче.
181.	Для уравнения х + х = х2
а)	найти и исследовать особые точки на фазовой плоскости;
б)	найти решение ж(£), убывающее и стремящееся к 1 при t +оо, а также его траекторию на фазовой плоскости;
в)	выяснить, при каких а решение с начальными условиями ж(0) = 0, ж(0) = а периодическое;
г)	указать на фазовой плоскости область, заполненную замкнутыми траекториями;
д)	устойчиво ли решение с начальными условиями ж(0) = 0, ж(0) = ^=?
В задачах 182 и 183
а)	дать чертеж траекторий на фазовой плоскости;
б)	найти особые точки и исследовать их на устойчивость;
в)	выяснить, определены ли все решения при —оо < t < оо.
182.
х = 1 — х2.
У = у2-
183.
X = х — X , k У = ~У3-
184*. Для системы
х = у — х2у — у3, у = х5 + х3у2 — х3
а)	найти все особые точки;
б)	линеаризовать систему в каждой из точек (0, 0), (1, 0), ( 1 1 .
\д/2’ V2J '
в)	исследовать устойчивость этих линеаризованных систем;
г)	исследовать на устойчивость те же три особые точки для исходной системы;
д)	дать чертеж траекторий на фазовой плоскости;
148	§ 26. Дифференцирование решения по параметру
е)	выяснить, имеет ли данная система неограниченные решения;
ж)	описать множество точек, через которые проходят периодические решения.
§ 26. ДИФФЕРЕНЦИРОВАНИЕ РЕШЕНИЯ ПО ПАРАМЕТРУ И ПО НАЧАЛЬНЫМ УСЛОВИЯМ
1. Дифференцирование по параметру
185.	Сформулировать теорему о дифференцируемости решения системы дифференциальных уравнений по параметру. Написать систему дифференциальных уравнений в вариациях.
В задачах 186—194 найти производную от решения данного дифференциального уравнения (или системы) по параметру pt при /х = 0.
186.	у' = ух+± (х> 0), у(1) = 1—2/1.
187.	у' = I + цхе~У (х > 0), 2/(1) = 1 + 2/1.
188.	у' = у — х + рьхе2у, ?/(1) = 2 — /х.
189.	у' = цх + sin?/, ?/(0) = 2/х.
190.	х = #sin£ + sin(a?2), ж(0) = /х, ж(0) = /х.
191.	х = х + sin(£2), ж(0) = /х, ж(0) = /х2.
192.	х + х = 2/х sin t + /хж2, ж(0) = 0, ж(0) = 0.
193.	х — 2х = pLtx, ж(0) = 4, ж(0) = /х2 + 3/х.
194.	х = у, у = х + 3/х?/2, ж(0) = 2 — 4/х, ?/(0) = 0.
2. Дифференцирование по начальным условиям
195.	Сформулировать теорему о дифференцируемости решения системы дифференциальных уравнений по началь
§ 27. Уравнения с частными производными
149
ным условиям. Написать систему уравнений в вариациях и начальные условия для нее.
196.	Доказать, что в случае у Е R1 производная по уо от решения задачи у' = f(x, у), у(х^) = уо всегда положительна (предполагается /ЕС1).
В задачах 197—199 найти производную от решения по уъ при ?/0 = 0.
Указание. При уо = 0 каждая из этих задач имеет нулевое решение.
197.	у' = 2ху + siny, у(1) = у0.
198.	у' = у3 sin х + у cos ж, т/(0) = у0.
199.
X = у — X + ж2, <
У = У - 2ж + ху,
ж(0) = 0, у(0) = у0.
200*. х + sin ж = 0, ж(0) = а, ж(0) = /3.
Найти У при а = 3 = 0. оосор	'
3.	Разложение решения по степеням параметра
В задачах 201 и 202 найти разложение решения по сте-пеням параметра у до у включительно.
201.	у' = 5/ix +
202.	х = 2х - 2ж3,
(«>1), 2/(1) = 1 — Л/.
ж(0) = 1, ж(0) = fi.
§27. УРАВНЕНИЯ
С ЧАСТНЫМИ ПРОИЗВОДНЫМИ ПЕРВОГО ПОРЯДКА
1. Теоретические вопросы
203.	Написать общий вид квазилинейного уравнения с частными производными первого порядка. Что называется характеристикой этого уравнения?
150	§ 27. Уравнения с частными производными
204.	Сформулировать и доказать утверждение о связи решения уравнения с его характеристиками.
205.	Как можно использовать первые интегралы некоторой вспомогательной системы дифференциальных уравнений для получения решения данного уравнения с частными производными?
206.	Сформулировать постановку задачи Коши для квазилинейного уравнения с частными производными и теорему существования ее решения.
207.	Сформулировать и доказать теорему о существовании решения задачи Коши для квазилинейного уравнения с частными производными первого порядка.
2. Задачи
208.	Найти общее решение уравнения
dz	dz
z— +XZ— = у. dx	dy
Решить следующие задачи Коши (209—215).
209.	xy^+xz^=yz, z = 1 + у2 при х = 1.
210.	|| + (z — ®2)fj = 2х. z = х2 + х при у = 2ж2.
211.	уВ + xz^ = У2’ z = ~У2 п₽и х = °-
212.	xz^ + yz^ = х3 + у- г = 4У3 ПРИ х = %У2-
213.	+ ху^ = x^z. z = е2*2/2 при х = 2у.
214.	х^- + z^ = z + 2х2, z = х при у = 4 — ж2.
215.	+ у^ =x + y + z. z = x + y при у = X + 1.
Решить следующие задачи Коши (216—218) в тех случаях, когда решение существует.
216.	ff + 2|| = 5, z = 0 при у = кх.
217.	й +	= 2, а) z = у2 при х = 1;
б) z = 2х при у — Зх.
218.	2^ -	= 2, z = 2ау при х = (а2 + а - 2)у.
§ 27. Уравнения с частными производными	151
219.	Имеют ли решения в окрестности точки (1,0) следующие задачи Коши:
a)	— х^ = 0’ z ~ 2У ПРИ х ~
б)	= °’ Z = 2У при X = 1 + у?
220*. Имеют ли решения в окрестности точки (1,1) следующие задачи Коши для уравнения
(ж3 - Зжу2)|| + (Зж2у - ?/3)|| = о
a)	z = sin?/ при х2 + у2 = 2:
б)	z = sin?/ при х = 1?
221. Какому условию должна удовлетворять функция </?(ж) 6 С1 для того, чтобы задача Коши
dz dz
у—----х— = 0, z = ч>\х) при у = 0, —оо < х < оо,
dx dy
имела решение на всей плоскости ж, у!
ОТВЕТЫ
15.	/(ж, у) = 0;	f'.<U (max),	f'>0 (min). 16. а) у = ж2	+
+ 2ж; б)	ж = 2ch?/; в)	ху3 = -(1 -	ж2)2;?/ = 0; г) /' + f • fy =	0.
17. у = ^ху'/у. 18. у' = 3?/2/3. 19. ху' = Зу. 20. у2 + у'2 = 1. 21. х2у'	— ху = уу'. 22. 2хуу’ — у2	= 2ж3. 23. у'3 = 4у(ху' — 2у).
24. у' = cos ху^1~У .	25. ж(ж — 2)у" — (ж2 — 2)у' + 2(ж — 1)у	=
= 0. 26. (уу" + у'2)2 = —у3у"- 27. 7/,,z/2(ln?/ — 1) = у,2(ху' — у). 28. х3у",-Зх2у"^ху,-6у = 0. 29. у"'у' = Зу"2. 30. (у-2х)2(у'2 + + 1) =	+ О2- 31. ху12 = y(2yf - 1). 32. (ху' - у)2 = 2ху(у'2 +
+ 1). 33. х2у" - 2ху' + 2у = 0. 34. (у" у + у12 + I)2 = (у'2 + I)3. 35. уу' + zz =0, у2 + 2жг/ = ж2/? 36. ж2 + у2 = z2 — 2z(y — — ху'); х + уу' = zz — z (у — ху'). 37. куу' = —ж. 38. у' = —2у. 39. (ж2 + у)у' = —ж. 40. (ж + у)у' = у — ж; (ж — у)у' = ж + у. 41. (ж =F уу/з)у' = у ± ж\/3. 42. (Зж =р уу/3)у' = у ± 3ж\/3. 43. (2ж=р =Р2/л/3)2/' = у ± 2ж\/3. 44. г sin# = г2. 45. г = |rctg0. 46. г = = rctg(0 ± 45°). 47. (ж + 2у)у' = —Зж — у; (Зж + 2у)у' = у — ж. 48. у'^2ху ± (ж2 — у2)] = у2 — ж2 ± 2ху. 49. ж(1 + у'2) = — 2уу . 50. уу'3 + ху'2 = —1. 51. у = С(ж + 1)е-ж; ж = —1. 52. In|ж| = С + + д/?/2 + 1;ж = 0. 53. ?/(1п |ж2 —1|Ч-С7) = 1, у = 0; ?/[1п(1-ж2) + 1] = 1. 54. у = 2 + С cos ж; у = 2 — Зсо8ж. 55. у = (ж — С)3; у = 0; у = = (ж - 2)3; у = 0. 56. у(1 -Сх) = 1; у = 0; у(1 + ж) = 1. 57. у2 -- 2 = Се.1/х. 58. (Се"®2 - 1)</ = 2; у = 0. 59. e“s = 1 + Се‘. 60. z = -lg(C- 10*). 61. x2+f — 2t = С. 62. ctg^ = х + С; у-- х = 2тгк, к = 0, ±1, ... 63. 2х + у - 1 = Се*. 64. х + 2у + 2 = = Се»; х + 2у + 2 = 0. 65. ^4х + 2?/ - 1 - 2	+ 2«/ - 1 + 2) =
= ж + С. 66. у = arctg(l — |) + 2тг. 67. у = 2. 68. а) 2у2 + ж2 = С; б) у2 + 2ж = С; в) у2 = Сеж2+г/2. 71. (С ± х)у = 2а2. 72. Ыну — — у = ±ж + С, (}<у<Ь. 73. aln(a ± у/а2 — у2) =F у/а2 — у2 = ж + + С. 74. у = Сх2. 75. у = Сх2; у2 = Сх. 76. r(l ± cos</>) = С. 77. Количество азота (в литрах) x(t) = 20 — 4е-^200; x(t) = 19,8 при t = 200 In 20 « « 600 сек = Ю мин. 78. Количество соли x(t) = = Юе-*/20; ж(60) = 10е-3 « 0,5 кг. 79. Объем СО2 (в м3) x(t) = = 0,08 + O,22e-t/10; x(t) = 0,1 при t = 10 In 11« 24 мин. 80. Темпе-
Ответы
153
ратура тела x(t) = 20 + 80 • 2“*/10; x(t) = 25 при t = 40жкн. 81. Разность температур воды и предмета x(t) = 55 • (3/5)*; x(t) = 1 при t = In55/(ln5 — 1пЗ) « « 8 мин. 82. Температура металла x(t) = = а + (t - x-efc ; яг(60) = b- |=^(1 -е~60к). 83. Скорость (в м/сек) v(t) = (2/3)(4/4)-1; v(t) = 0,01 при 1 = 4 (lg2 5 + 1) и 50 сек; путь s = 1пв 5 ~15 м. 84. Оставшееся количество вещества x(t) = = ж(О)2-*/30; x(t) = 0,01ж(0) при t = 60/1g 2 « 200 дней. 85. Оставшееся количество радия x(t) = ж(0) • (1— 0.00044)*; x(t) = |ж(0) при t = 1п0,5/1п(1 — 0,00044)^1600 лет. 86. Количество урана x(t) = = ж(0)е"а* а = In2/(4,5-109); x(t) = 100, ж(0) = 100 + 14-Щ = = 116,2; t = 4,5 • 109 • lg11’^62 « 970 • 106 лет. 87. Количество све-та, прошедшего через слой в х см, у(х) = ?/(0) • 2“*^35; ?/(200) = = ?/(0)2—40/7 » 0,02 • ?/(0); поглощается 100% — 2% = 98%. 88. Скорость v(t) = 50 th путь (в метрах) s(t) = 250 Inch s(t) = = 1000 при ch | = е4, t « 5(4 + In2) « 23 сек. 89. Скорость v(t) = = д/f Vkg(C-t), g = 10, k = 0,012, C = arctg v(0)	1,75;
v(t) = 0 при t = C^l,75 сек; наибольшая высота h =	1п^г;2(0)-|-
+ 1)«16,3 м (без сопротивления воздуха t = 2 сек. h = 20 ж). 90. Скорость v(t) = y/^thy/kgt, путь s(t) = llnchy^fcgt; s(t) = = h = 16,3 ж при t = ln(efeh + y/e2kfl — 1) ~ 1,87 сек, v(t) = = д/f (1 — e~2kh) « 16,4 м/сек. 91. Высота уровня воды h(t); y/H — - Vh = 0,3y/2g^t; h(t) = 0 при t = 5^72^/^ ~ 1050 сек = = 17,5 мин. 92. (2R - fo(£))3/2 = 0,457rr2y/2g-^, h(t) = 0 при t = =	~ 1040 сек. 93. y/H - а/Ж = kt, k =	(1 - -И;
h(t) = 0 при 1 = 5(2 + д/2)«17жин. 94. Hs/2-[7i(l)]s/2 = ^^y/2g-, h(t) = 0 при t = (4B2/3d2) у/ZH/g Л 27 сек. 95. Объем воды в баке в литрах x(t); t = |f In	Q = 1,8, a = 10“3^2;
x(t) = 360 при t = 260 сек (для бака без отверстия в дне t = = 200 сек). 96. Удлинение нижнего куска длины х равно у(х) = =	, а всего шнура — у (Г) =	97. На высоте h км давление
p(h) = e-0,12h (кГ/сж2). 98. Сила натяжения каната на расстоянии
(в радианной мере) по дуге от начальной точки равна /(</?) = = /(0)е</?/3; /(бтг) = 10е27Г « 5000 кГ. 99. Количество оставшейся воды m(t) = mo —	— до) (1 —	, к — коэффициент про-
порциональности. 100. После сгорания массы х топлива скорость ракеты v(x) = с In ^_х; v(M — m) = cln^. 101. х + у = Сх2;
154
Ответы
х = 0. 102. 1п(ж2+?/2) = С — 2 arctg(?//a?). 103. х(у — х) = Су; у = 0. 104. х = ±?/д/1п Сх; у = 0. 105. у = Сеу^х. 106. у2 — ж2 = Су; у = 0. 107. sin = Сх. 108. у = —ж!п1пСж. 109. In = Сх. 110. 1пСж = ctgQ In ^): у = же27г/г, к = 0. ±1, ±2, ... 111. 2^/ху = = xlnCx; у = 0; х = 0. 112. arcsin = 1пСх • sgna?; у = ±ж. 113. (у - 2ж)3 = С(у - х - I)2; у = х + 1. 114. 2ж + у - 1 = = Се2у~х. 115. (у - х + 2)2 + 2ж = С. 116. (у - х + 5)5(ж + 2у -- 2) = С. 117. (у + 2)2 = С(ж + у - 1); у = 1 - х. 118. у + 2 = = Ce"2arctg^i. 119. ln^ = 1 +	120. sin 2^ = С(ж + 1).
®+3	&+у	® + 1	v '
121. х2 = (ж2 — ?/)1пСж; у = х2. 122. х =	— у21пСх;	у	=	0.
123. ж2?/41пСж2 = 1; у = 0; х = 0. 124. у2е~1/ху = С;	у	=	0;
х = 0. 125. (2у/у — ж) In С (2у/у — ж) = ж: 2^/у	= ж. 126. 1	—	ху	=
= Сж3(2 + ху); ху = —2. 127. 2д/(1/ж?/2) — 1	= — 1пСж;	у	=	0;
ху2 = 1. 128. arcsin j^3j = 1пСж3; |ж3| = у2. 129. х2у In Су = 1; у = = 0. 130. а) у2 = С(ж + у); у = -ж; б) (у + ж)2(?/ - 2ж)4 = С(у- ж)3; у = ж. 131. у = С(ж2 + ?/2). 132. ж2 +у2 = Сх. 133. При д —	= 1.
136. у = Сх2 + ж4. 137. у = (2ж + 1)(С + In |2ж + 1|) + 1. 138. у = = sin ж + Ссо8ж. 139. у = еж(1п |ж| + С); ж = 0. 140. ху = С — In |ж|. 141. у = ж(С + sin ж). 142. у = С С — ж2 — 1. 143. у = С In2 ж — 1пж. 144. ху = (ж3 -|- С)е~х. 145. ж = у2 + Су; у = 0. 146. ж = еу + + Се~у 147. ж = (С — cos?/) sin?/. 148. ж = 2 In?/ — у + 1 + Су2. 149. ж = С?/3 + у2; у = 0. 150. (у — 1)2ж = у — 1пС?/: у = 0; у = 1. 151. у(ех + Сс2ж) = 1; у = 0. 152. ?/(ж + 1)(1п|ж + 1| + С) = 1; у = = 0. 153. ?/-3 = С cos3 ж — 3 sin ж cos2 ж; у = 0. 154. ?/3 = Сж3 — — Зж2. 155. у2 = Сж2 — 2ж; ж = 0. 156. у = ж4 In2 Сх; у = 0. 157. у~2 = ж4(2еж + С); у = 0. 158. у2 = ж2 - 1 + Сд/|ж2 - 1|. 159. ж2(С — cos?/) = у; у = 0. 160. ху(С — In2?/) = 1. 161. ж2 = = Се2у + 2у. 162. у2 = С(ж + I)2 - 2(ж + 1). 163. е~у = Сж2 + ж. 164. cos?/ = (ж2 - 1)1пС(ж2 - 1). 165. у = 2ех - 1. 166. у = -2еж. 167. ?/ = 2 + 7^4—; У = -• 168. у = 1 Л. ; у=~. 169. у = = X + у = х. 170. у = х + 2 + Ce41_t; У = х + 2. 171. у = еж -- 4с! у = е*- 172- Зх = CVW\ ~У2-У = 0- 173. ху = Сх3 + 2а2. 174. ху = а2 + Су2. 175. Через 20 мищ 3,68 кГ. 176. Через 62 дня. 177. у = ?/i + С(?/2 — ух)- 178. у = tgж — весж. 179. Ь/а. 180. Ь/а.
181. x(t) = J es~tf{s) ds = f ezf(z + t) dz. 182. ?/(ж) = xf ex ~tdt—^ — oc	— ОС	+oo
| при ж +oo. 183. y(x) = f e_s-sins'cos(s+2a;) 8т(ж + s) ds. 0
186.	Зж2?/ - у3 = С. 187. ж2 - Зж3?/2 + у4 = С. 188. хе~у - у2 = С.
Ответы
155
189.	4?/1пж + ?/4 = С. 190. ж + р- + ^ = С. 191. ж2 + |(ж2 — ?/)3/2 = С. 192. х — у2 cos2 х = С. 193. ж3 + ж3 In?/ — у2 = С. 194. ж2 + 1 = 2(С — — 2ж) sin у. 195. 2ж + 1п(ж2 + у2) = С. 196. х + arctg = С. 197. ху + + С = д/1 + У2’ 198« 2ж3?/3 - Зж2 = С. 199. у2 = х2(С — 2?/); х = 0. 200. (ж2 — С)у = 2ж. 201. ж2 + In?/ = Сж3; ж = 0. 202. ysinxy = С. 203.	+ ху + In |?/| = С; у = 0. 204. —ж + 1 = ху (arctg?/ + С);
ж = 0; у = 0. 205. ж + 2 In |ж| + |?/2 —	=	С; ж = 0. 206. sin =
=	Се~х2. 207. In |j/| - уеГх = С; у = 0. 208. 1п(^ + 1) = 2у + С; у = 0. 209. х2у \nCxy = —1; ж = 0; у = 0. 210. ж2 + у2 = у + Сж; ж = 0. 211. х2у + 1п|ж/?/| = С; ж = 0; у = 0. 212. 2ху2 + (1/ж?/) = = С; х = 0; у = 0. 213. 1п|*±*| +	= С; у = 0; у = -х.
214.	sin2 у = Сж — ж2; ж = 0. 215. у = С1пж2?/. 216. sin?/ = —(ж2 + + 1) 1пС(ж2 -I-1). 217. ху(С — ж2 — у2) = —1; ж = 0; у = 0. 218. у2 = = Сж2еж у . 219. жд/1 + (?/2/ж2) + In [у/х + д/1 + (?/2/ж2)) = С: ж = 0. 220. ж3 — 4?/2 = Су^ху, х = 0; у = 0. 221. а) ?/о = 0, ?/i = ж2/2, у2 = (ж2/2) - (ж5/20). б) у0 = 1, yi = ж3, ?/2 = 1+ж3-- ж + (ж7 - 1)/7. в) уо = 1, ?/i	= 1 + 2ж, ?/2	= |(е2ж + 1) + ж + ж2,
г) уо	=	2тг, ?/i = 7г + ж, ?/2 = 2тг	+ ж + ж cos ж	— sin ж. 222. а) уо = 1,
zq =	0;	?/i = ж2,	= ж — 1; ?/2	= ж2 + (ж —	1)2/2, z2 = (ж3 — 1)/3.
б) жо	=	1, уо = 2; Ж1 = l + 2t, yi	= 2-И; х2 =	1 + 2t + (t2/2), ?/2 = 2 +
-\-t-\-2t2 + (4/3)t3. в) уо = 1, yi = 1, у2 = 1 + ж2. г) жо = 2, Ж1 = 3-t, х2 = 5 - 4t+ С. 223. а) -0,5^ж^0,5. б) 0,87^ж^1,13. в) 0,8^1,2. г) -O.l^O.l. 224. ?/3 = < - < + < -	\у - 7/31<0.00003.
225. а) Вся плоскость, б) у^2х. в) ж^2, ?/>0. г) у^^ + кк, к = = 0, ±1, ±2, ... д) ж>0, ?/#ж. е) ж^0, |?/|>|ж|. 226. При 0<а<1 в точках оси Сж. 228. а) жо и у'о любые, ?/07^f + тг&, к = 0, ±1, ±2, ... б) ж0^ - 1, T/о>0, ?/о любое, в) жо^?/о, жо?/о>0, ?/о^0, Уо любое, г) хо^Уо, ?/о^0, ?/о любое, д) to и уо любые, жо^О. е) to> — — 1, жо^О, yo^to- 229. а) Нет. б) Да. 230. а) Нет. б) Нет. в) Да. 231. В случае п = 1 нет решений, при п = 2 одно решение, при п = = 3 бесконечно много решений. 232. В случае п = 1 нет решений, если tga^f^o, уо), и одно решение, если tga = /(жо, ?/о); в случае п = 2 одно решение, а при п 3 бесконечно много. 233. п^Ь. 234. п>4. 236. а) 3. б) 2. в) 4. г) 4. д) 3. е) 1. 237. а) 0^а^1. б) в) 1^а^|. г) —241. у = Се±ж. 242. у2 = (ж + С)3; у = 0. 243. у + ж = (ж + С)3: у = —ж. 244. (ж + С)2 + у2 = 1; у = ±1. 245. ?/(ж + С)2 = 1; у = 0. 246. ?/[1 + (ж — С)2] = 1; у = 0; у = 1. 247. (?/ - ж)2 = 2С(ж + у) - С2; у = 0. 248. (ж - 1)4/3 + + 2/4/3 = С. 249. 4?/ = (ж + С)2; у = Сеж. 250. ?/2(1 - у) = (ж + С)2;
156
Ответы
у = 1. 251. у = Сеж; у = Се~х + х — 1. 252. х2у = С; у = Сх. 253. ж2 + С2 = 2Су\ у = ±ж. 254. (ж + С)2 = 4Ср; у = 0: у = х. 255. In |1 ± 2д/2р — ж| = 2 (ж + С ± у/2у — х); 8у = 4ж+1. 256. (ж+ + 2)4/3 + С = 4е"2//3. 257. у = 2ж2 + С; у = -ж2 + С. 258. у = Сж"3± ±2д/ж/7. 259. In Су = ж±2еж/2; у = 0. 260. In Су = ж ± sin ж; у = 0. 261. arctgiz + | ln|(tt — l)/(tt + 1)| = ±ж + С, где и = у/1 — (1/у2); у = 0; у = ±1. 262. ж2 + (Ср + 1)2 = 1; у = 0. 263. (Сж+1)2 = 1 - у2; у = ±1. 264. 2(ж - С)2 + 2у2 = С2-, у = ±ж. 265. у = Се±х - ж2. 266. у2 = С2х — С-, 4ху2 = —1. 267. ж = р3 + р, 4у = Зр4 + 2р2 + С. 268. х = ^СУ =	- In |р2 - 1| + С. 269. х = р^Р^+Р Зу =
= (2р2 —1)^/р2 + 1+С. 270. х = 1пр+(1/р), у = р—1пр+С. 271. х = = Зр2+2р+С, у = 2р3+р2; у = 0. 272. х = 2 arctgp+C, у = 1п(1+р2); У = 0. 273. х = In |р| ± | In |	1 ± 3Vp+ 1 + С, у = р ± (р +
+ 1)3/2; у = ±1. 274. х = ер + С, у = (р - 1) ер; у = -1. 275. х = = ± ^2 \/р2 — 1 + arcsin + С, у = ±р^/р2 — 1; у = 0. 276. х = = ±(ln|i^|| +ЗУТ^)+С', у = ±	у = 0. 277. х =
= ±2^/1 +р2 - 1п(т/р2 + 1 ± 1) + С, у = -р± ±р^/р2 + 1; У = 0. 278. 4у = С2 - 2(х - С)2; 2у = х2. 279. х = -f + С, 5у = С2 -х2 = 4?/. 280. ±xpV21nCp = 1, у = т ( л/21пСр- ~y21tCp)  281. рху = у2 + р3, у2(2р + С) = р4; у = 0. 282. у2 = 2Сх — С In С; 2ж = 1 + 2 In |р|. 283. Сх = In Су, у = еж. 284. хр2 = Сд/|р| — 1, у = хр - ж V; у = 0. 285. 2р2х = С- С2р2. ру = С; З2ж3 = —27?/4; у = 0. 286. у2 = 2С3ж + С2; 27ж V = 1. 287. у = Сх - С2; ±у = ж2. 288. Ху/р = lnp+С, у = д/р(4 —Inp —С); у = 0. 289. ж = Зр2 + Ср-2, у = 2р3 + 2Ср-1; у = 0. 290. у = Сх — С — 2. 291. С3 = 3(Сж — р); 9р2 = 4ж3. 292. ж = С(р— I)-2 + 2р + 1, у = Ср2(р — I)-2 +р2; у = 0; у = ж — 2. 293. у = Сх — In С; у = In ж + 1. 294. у = ±2\/Сж + С; у = —ж. 295. 2С2(р — Сх) = 1; 8р3 = 27ж2. 296. хр2 = р + С,у = = 2 + 2Ср-1 — Inp. 297. а) 4р = ж4: б) у = 0, у = —4ж; в) у = 0, 27р = 4ж3; г) у = 4ж. 298. ху =	299. ж2+р2 = 1. 300. ж = р (р2 +
+ 2)/(\/р + 1)® У = Р^Ку/^2 + I)3 и х = р/(д/р2 + х)3 У = (2Р2 + + 1)/(д/р2 + 1)? 301. у = х(Се~х - 1). 302. (Сх + 1)у = Сх - 1; у = 1. 303. у(х2 — С) = х; у = 0. 304. х(С — у) = С2', х = 4у. 305. у(х + С) = х + 1; у = 0. 306. х = Су + у3-, у = 0. 307. у = С; у = С ±е.х. 308. ylnCx = —ж; у = 0. 309. у2 = С(х2 — 1); х = = ±1. 310. 2у = 2С(х-1) + С2; 2у = -(х-1)2. 311. х = Су + 1п2у. 312. у = Сх2е~3/Х. 313. (х-С)2+у2 = С-, 4(у2-х) = 1. 314. 4Ж2г/ = = (х+2С)2-, у = 0. 315. х = Сеу+у2+2у+2. 316. Зу = ЗС(х-2)+С3;
Ответы
157
9р2 = 4(2 - ж)3. 317. у2 = С(ху - 1); ху = 1. 318. 4(ж - С)3 = 27(р -— С)2; у = х — 1. 319. х + у = tg(j/ — С). 320. х3у2 + 7х = С. 321. у(ху - 1) = Сх. 322. -<ГУ = 1пС(ж - 2). 323. х = у2(С -- 2 In |г/|); у = 0. 324. Зху = С ± 4ж3/2. 325. у2{С^ + 1) = 1; у = 0. 326. у2 = 2ж In Ср; у = 0. 327. 1п(ж2 + у2) + arctg(?//ic) = С. 328. (ж - I)2?/ = ж - 1п|ж| + С. 329. С2х2 + 2у2 = 2С; 2ж2р2 = 1. 330. у(Су/\х2 - 1| — 2) = 1; у = 0. 331. р2(Се2ж+ж + 0,5) = 1; у = 0. 332. р2-1 = С(ж + 1)4е"4ж(р2 + 1); ж = -1. 333. у 8тж-= С. 334. ж = Зр2 +р-1. у = 2р3 — In |р| + С. 335. Зр2 = 2 sin ж 4- С sin” 2 ж. 336. ж(е2/ 4- ху) = С. 337. ж(р — I)2 = In Ср — р, у = жр2 + р; у = 0; у = ж + 1. 338. (ж + 1)у = ж2 + ж!пСж. 339. у2 + д/ж4 + р4 = С. 340. рж = Су/р — 1, у = Inp — Су/р + 1. 341. у = ж1£1пСж; ж = 0. 342. р2/3 = Се2ж + (ж/3) + (1/6); у = 0. 343. ж = Cesiny -2(l + sinp). 344. Су = С2ех + 1; у = ±2еж/2. 345. у2 = (ж2 + С)е2ж. 346. у = = Сж - ^С3 - 1; р3 = (ж3/2 ± I)2. 347. ж(р2 - I)2 = р3 - Зр + С; у = ±1. 348. у/у — х — у/х = С; у = ж. 349. Xy/у = sin ж + С; р = 0. 350. ж = 4р3—In Ср, у = Зр4—р; у = 0. 351. р2+2ж2 In Ср = 0; р = 0. 352. 4ж + у — 3 = 2 tg(2ж + С). 353. жр cos ж — у2 = С. 354. 4Сжр = = С2ж4 — 1. 355. жр(1п2 ж + С) = 1. 356. 2д/р — ж2 = ж 1пСж; у = ж2. 357. (р2/2) - (1/ж) - жр = С; ж = 0. 358. ж = Ср2 - р2(р + 1)е”2/; р = 0. 359. p(lnp — In ж — 1) = С. 360. ж = 2р — 1пр, у = р2 — р + + С. 361. 2ж3 - ж2р2 + р3 + ж = С. 362. (р - 4ж + 2)4(2р + 2ж -— 1) = С. 363. у3 = (С — ж3)8т3 ж. 364. р2ж = psinp + cosp + С, ру = psinp + 2cosp + 2С; у = 0. 365. ж2р2 — 1 = жр!пСр2; у = 0. 366. у = С cos ж + sin ж. 367. |ж| = 1п^ + ^/1 +	+	С; ж = 0.
368. (р-ж)2 = 2С(ж+р)-С2; р2/3-ж2/3 = С; у = 0. 369. 27(р-2ж)2 = = (С — 2ж)3; у = 2ж. 370. 8т(р/ж) = — 1пСж. 371. ж2 (д/1 + ж4р2+ +ж2р ) = С. 372. Зу/y = ж2 —1+С^/|ж2 - 1|; р = 0. 373. ж = ^~р~ -%, У = С (f - Х) = х+2’ У = °- 374- (2®+3«/-7)3 = Се*+Ч 375. (ж2 + у + InСу)у = ж; у = 0. 376. ж = 2д/р2 + 1 — 1п(1 + + д/р2 + 1) -|- In Ср, у = рд/р2 + 1; р = 0. 377. р2 = С In2 ж + 2 In ж. 378. ж = СиС/ 4р = C2e2w(2n2 + 2n+1); ж2 = 2р. 379. жр2 1пСжр = = 1; ж = 0; р = 0. 380. ж2 sin2 у = 2 sin3 р+С. 381. 1—жр = (Сж —I)2; жр = 1; р = 0. 382. хеу = еж + С. 383. sin(p — 2ж) — 2cos(p — 2ж) = = Сех+2у. 384. у = (2ж + С)д/ж2 + 1-ж2-Сж-2. 385. (р+ж2)2(2р-— ж2) = С. 386. (ж — I)2 = р2(2ж — 21пСж); у = 0. 387. ж = р[1п(1 + + д/р2 + 1) — In Ср], 2р = жр — д/р2 + 1; 2р = —1. 388. (р + Зж + + 7)(р — ж — I)3 = С. 389. sinp = Се-Ж +ж — 1. 390. у = С2(ж — С)2;
158
Ответы
16г/ = ж4. 391. у2 = х — (ж + 1)1пС(ж + 1). 392. еу = х2]пСх. 393. (у — 2х^/у — х2)(2у/у — ж2 + х) = С. 394. ху2 = In ж2 — In Су: х = 0; у = 0. 395. х(у2 + ж2)3 = |?/5 + |ж2?/3 + 2ж4?/ + Сж5; ж = 0. 396. (и — 1) 1пСж6(?? — 1)5(?? + 2)4 = 3, где и3 = (?/2/ж2) — 2; у2 = Зж2. 397. у/у = (ж2 - 1)(2 In |ж2 - 1| + С); у = 0. 398. ж2 - (ж - 1) 1п(^ + + 1)— у = С. 399. tgу = ж2 + Сж; у = (2к + 1)тг/2, к = 0, ±1, ±2, ... 400. у2 = Сх2 + С2. 401. ж3 = Сеу — у — 2. 402. ?/ + 1 = ж1пС(?/ + 1); у = -1. 403. у2 = 2С2(ж—С); 8ж3 = 27у2. 404. ж6 = у3(С-у In?/+?/); у = 0. 405. lnC(?z — v)3(u2 + uv + v)= arctg(l + 2t6/v), где и3 = у, v2 = ж; у2 = ж3. 406. (у — I)2 = ж2 + Сж. 407. (ж2 + у2)(Сх +1) = ж. 408. Зж + ?/3— 1 = tg(Зж^-C). 409. (С — х2}у/у2 + 1 = 2ж. 410. (ж2+ +?/2+1)2 = 4ж2+С. 411. ху — х = у(у — ж) 1п|С?//(?/ — ж)|; ж = 0; у = 0; у = ж. 412. у = ±жсЬ(ж + С); у = ±ж. 413. д/?/2 + 1 = ж(Се® — 1). 414. (?/ —ж)1пС^=у = 2; у = ж. 415. (Се®2 + 2ж2 + 2) cos у = 1. 416. (у2— Сж2 + 1)2 = 4(1 — С)у2; у = ±ж. 417. у2 +ху — 1 = Се®2/2. 418. 6ж3?/4+2ж3?/3+Зж2?/4 = С. 419. ж+|+?/2-2?/+2 = Се~у, ж = 0. 420. ег/(С2ж2 + 1) = 2С; ж2 = е~2у. 421. С\х — С2у = In |С1ж+1|+С2; 2у = ж2 + С; у = С. 422. 9СХ2(?/ - С2)2 = 4(С1Ж + I)3; у = ±ж + С. 423. Ciy2-1 = (С1ж + С2)2. 424. у3 = С1(ж + С2)2; у = С. 425. у = = Citg(Cix + С2); 1п|	|	= 2С1Ж + С2; у = (С - х) = 1; у = С.
426. С\у = 8ш(С1ж + С2); Суу = ±sh(C^ + C2); у = С±ж. 427. у = = С1(ж-е-®) + С2. 428. у = С3-(ж + С1) 1пС2(ж + С1); у = С1ж + С2. 429. J/4-C1 In |з/| = ж + С2; у = С. 430. 2у — Ci cos 2ж + (1 + 2С1)ж2 + + С2ж + Сз- 431. у = С1[1±сЬ(ж+С2)]; у = Се±ж. 432. ж = Cip+3p2, у =	+ |С1Р4 + Cf4 + С2; у = С. 433. у = Ci^ - С?ж + С2;
у = (ж3/12) + С. 434. еу + Ci = (ж + С2)2. 435. у = С1(ж + 2)е-® + + С2ж + Сз- 436. у = ±сЬ(ж + С1) + С2. 437. еу 8т2(С1ж + С2) = 2Cf; е» sh2(Ci«+C2) = 2СЬ е^х+С')2 = 2. 438. у = С1^-С^т+С2Х+ + Сз; у = ^х3у/Зх + Cix + С2. 439. 3Ciy = (х — Ci)3 + С2; у = С; у = С - 2ж2. 440. 1п|т/2+ С1±д/«/4+2С1?/2+ 1| = 2х + С2; у = ±1. 441. ж = 3Cip2+lnC2p, у — 2Cip3+p; у = С. 442. ж = С\ер— 2р — 2, у = С^р- 1)ер - р2 + С2. 443. 12(Ci^ - ж) = С?(ж + С2)3 + С3. 444. у = ж2 + Ci + С2(ж\/ж2 — 1 — 1п|ж + у/х2 — 1|) ; у = ж2 + Ci + + С2(ж\/1 — ж2 + arcsin ж). 445. In?/ = Ci tg(Ci.T + С2); ln|(ln?/ — — Ci)/(ln?/ + Ci)| = 2С1Ж + C2; (С — ж) In?/ = 1; у = C. 446. x = u — — In |1 + ?i| + C2, где и = ±V1 + 4Ci?/: у = С; у = Се-®. 447. С2у = = (С2ж2 +1) arctg С1Ж — С1Ж + С2; 2у = &тгж2+ С, к = 0, ±1, ±2, ... 448. ж = ln|p|+2Cip—С2, у = р+С1р2+Сз; у = С1ж+С2. 449. Cf?/+ + 1 = ±сЬ(С1Ж + С2); Cly - 1 = 8ш(С1Ж + С2); 2у = (ж + С)2;
Ответы
159
у = 0. 450. у = Съ — ln|cos0^- + Ci)|. 451. &у = ж3 1п|ж| + Ci®3 +
+ С2Х2 + С3Х + С4. 452. у = х f^^dt + cos х + Cix + С2. 453. у = о
= Cl xfe^dt - |(еа’2— 1) + С2х + С3. 454. у = J ^dt - 2±1еж + .0	J	1
+ Ci®2ln|®| + C2®2 + C3® + C4. 455. C2y2-Ci = С22(® + С3)2; у = С. 456. С1У = In |Ci® + С2\ + Сз;у = С1Х + С2. 457. Cig/-1 = С2еС1а’; у = С—х’, у = 0. 458. у = Cix2 + С2х+Сз', у = iy/Ci® + С2 + С3® + +С4. 459. у2 = ®2+Ci®+C2. 460. у =	12 (Ci fe^^d® + С2) -1.
461. у = Citg(CilnC2®); у-Ct = С2(у + Ci)|®|2C1; ylnCx = -1. 462. 2 In 1 = Ci®2 + C2; у = 4Citg(Ci®2 + C2); y(C - ®2) = = 4; у = C. 463. у = C2eCr. 464. Ci® + 4®s^2 = 1пС2у; у = = 0. 465. у = C2(x + V®2 + 1)C1. 466. y2 = Ci®3 + C2. 467. у = = C2x&~Cl,x. 468. у = С2|Ж|С1-<1/2>lnl-L 469. у = C2 |^-|1/C1; У = С; у = Ce’1^. 470.	= C2 (x -	|® + Ci|ci: у =
= C. 471. у = C2®(lnCi®)2; у = Cx. 472. ln|g/| = ln|®2 — 2® + + Ci| +	+C2-,y = C. 473. 4C13/2 = 4® + ®(Ci lnC2®)2.
474. у = -®ln(C2lnCi®); у = Cx. 475. f = C2 - 31n || - Ci|; у = = Cx. 476. x2y = Ci tg(Ci lnC2®), C2(x2y + Ci)|®|2C1 = x2y — Ci; x2ylnCx = -1. 477. 4(Ciy-l) = C? ln2C2®. 478. Cy = ®3/2(C2®c+ + 2); у = C®3/2; у = -2®3/2 InC®. 479. 2C2x2y = (C2x - Ci)2 - 1; xy = ±1. 480. 2CiC2y = Ci|®|2+C1 +|®|2-C1. 501. (3-®)?/s = 8(® + + 2). 502. y(x + 2) = —x - 6. 503. (1 - ln®)2y = ®2. 504. у = = 3 th2	- 2. 505. Intg (| + f) = 2® + 2. 506. а) 4(Сц/ - 1) =
= Cl(®+C2)2; 6) y^fiCijy') — 1+Ci arccos y/yJCi = C2±®. 507. у = = C2 — A; Ineos (|+ +Ci). 508. у =	+ Ci® + C2; p — нагрузка
на единицу длины горизонтальной проекции, Т — горизонтальная
составляющая силы натяжения нити. 509. ау = ch(a® + Ci) + С2; а = q/T, q — вес единицы длины нити, Т — см. ответ к задаче 508. 511. у = С1еж+С2с-2ж. 512. у = С1е"ж+С2с"3ж. 513. у = Ci+ + С2е2® 514. у = С1б2ж + С2сж/2. 515. у = е2ж(С1 cos® + С2 sin®). 516. у = е-ж(С1 cos Зх + С2 sin3®). 517. у = Ci cos 2ж + С2 sin2®. 518. у = С1е2аЧ-е-ж(С2 cos ж^/З+Сз sin®V3). 519. у = С1еж+С2е-ж+ + Сз cos х + С4 sinх. 520. у = еж (Ci cos х + С2 sinх) + е-ж (Сз cos х + + С4 sin®). 521. у = ежг/з(С1СО8 ® + С2 sin.®) + Сз cos 2® + С4 sin2® + + е-жг/з(С5 cos® + Cesin®). 522. у = еж(С1 + С2®). 523. у = = е"ж/2(С1 + С2®). 524. у = Ci + С2® + С3®2 + с3ж(С4 + С5®). 525. у = Ci + С2сж + С3с"ж + С4с3ж + С5с"3ж. 526. у = (Ci +
160
Ответы
+ С2ж)со8ж + (Сз + C4a?)sina?. 527. у = еж(С1 + С2х + Сзж2). 528. у = еж(С1 + С2х) + С3е"ж. 529. у = Ciex + С2с"ж + С3е2ж + + С4е-2ж. 530. у = Ci + (С2 + Сзж)сов2ж + (С4 + С$х) 8т2ж. 531. у = еж(С1 + С2х) + Сзе-2ж. 532. у = Cicosa? + С28шж + +С3 со8жд/3+С4 втжд/З- 533. у = Cie~x+С2е3ж + (1/б)е4ж. 534. у = = Ci cos х + С28тж + (2ж — 2)еж. 535. у = С\ех + С2е~х + хех + + х2 + 2. 536. у =	- |)еж + Cie-2® + С2еж. 537. у =	+
+ С2 е2ж+0,1 sin ж + 0,3 cos ж. 538. у = Cicos ж + С2 sin ж — 2жсо8ж. 539. у = С1СЖ+ С2с4ж- (2ж2 - 2ж + 3)с2ж. 540. у = Сюж + С2с2ж + + (0,1ж — 0,12) cos ж — (0,Зж + 0,34) sin ж. 541. у = С\ех + С2е-4ж — -	- (| + ^) е"”. 542. у =	+С^Х + С2е~3х.
543. у = е2ж(Cicos 2ж + C^sin2ж) + 0,25е2ж + 0,1 cos 2ж + 0,05 sin2ж. 544. у = С1в3ж+ С2е"3ж + е3ж(^8тж — ^со8ж). 545. у = (Ci + С2ж + + ж3)еж. 546. у = (б1 — cos ж + (С2 + |) sin ж. 547. у = (Ci + +С2ж)с"2ж+(^ - ^)е2х 548. у = С1+С2с5ж-0,2ж3-0.12ж2-0,048ж+ + 0,02(cos 5ж — 8ш5ж). 575. у = еж(ж 1п|ж| + С1ж + С2). 576. у = (е-ж+ +с"2ж) 1п(еж+1) + С1С-ж+С2е-2ж. 577. у = (Ci+ln | 8тж|) 8тж + (С2-— ж) cos ж. 578. у = sin 2ж In | cos ж| — ж cos 2ж + Ci sin 2ж + С2 cos 2ж. 579. у = е-ж 0(ж + 1)5/2 + Ci + С2х). 580. у = Ci cos ж + С2 sin ж — 581. у = -|+С1Сж+С2е-ж. 582. у = (7-Зж)сж"2. 583. у = = 2 cos ж — 5 sin ж + 2еж. 584. у = е2ж-1 — 2еж + е — 1. 585. у = = е"ж(ж - 8тж). 586. у = 2 + е"ж. 587. у = (ж - 1)(е2ж - е"ж). 588. у = ж — ж sin ж — 2 cos ж. 589. у = Cix2 + С2х3. 590. у = С1Ж3 + + С2ж-1. 591. у = ж(С1 + С2 1п|ж| + Сз 1п2|ж|). 592. у = Ci + С21п|ж| + + Сзх3. 593. у = ж(С1 + С21п|ж|) + 2ж3. 594. у = Cicos(21п|ж|) + + C2sin(2 In |ж|) + 2ж. 595. у = С1Ж2+ 1 (С2 — | In ж — In2 ж). 596. у = = ж2(С1 cos In |ж| + С2 sin In |ж| +3). 597. у = С1Ж3+С2ж-2+ж3 1п|ж| — — 2ж2. 598. у = С1Ж2+ С2ж-1+0,1 cos 1пж — 0,3 sin In ж. 599. у = (ж — - 2)2(Ci + С2 In |ж - 2|) + х - 1.5. 600. у= С1(ж+ |)+С2|ж+ ||3/2+ + Сз|® + ||1/3 601. у = (Ci + С2х + ^)е~х+	602. у = ^е3х-
-	+(f| + С1Х + С2)е®. 603. у = С^-1+^х + [С2 + (г - 1)а:]х
_ e(i+O* 604. у = (2ж2 + С1« + C2)e~ix - eix. 605. у = =	С2е<г-'/5’а,+ (Сз - Д) e~2ix + £e.2ix. 606. у =	+
+ [^2 — 11п(— х) + 11п2(—ж)] х2. 607. у = (Ci + С2® + я:1п|®|)е_а’ + + ^еж. 608. у = [| + (С1-|)со82ж+(С2 + | + |1п|со8ж|)х х sm2x]e-a!. 609. у = х2 In	+ 1 — 11пС2(х + 1). 610. у =
= x[Ci + (С2 + In 11пж|) 1пж] +	611. у = Ci cos ж + С2 sin® +
Ответы
161
X	XX
+ f sin( x — s )/(e)de. 612. f f(s) cos s ds и J f(s) sin s ds ограничены 0	0	0
при x—> + oo. 613. у" — Зу" + Зу — у = 0. 614. у" — 4yf + 5y = = 0. 615.	+ 2y" + у = 0. 616. у™ - dy'" + 141/" - 20t/' + 251/ =
= 0. 617. у'" - у" - у' + у = 0. 618. у™ + у" = 0. 619. а = О, Ь>0. 620. а>0, Ь>0. 621. Ь<0 или Ь^О, а>0. 622. Ь>0,	— 2y/b.
623. а2 <46. 624. а>2, b>a - 1. 625. а = 2Vb. 626. w/ ± к-, ш = к = = 0. 627. х =	амплитуда А =	1
(Ь_ш2)2+о2и,2	5/(b-W2)2+o2w2’
шах 4 достигается при ш2 = b —	628. х =	— 629. x(t) =
— oo	0
630. 27r/f. 631. В случае h2>dkm x = ^.(е<-“+^‘ - e(-“-7)t), ° = 2TT< 7 =	 В случае h2<dkm x = sin/%, a =
А/4ктп — Н2	z»\	b(k — ma>2)sin cut — bhui cos cut	л кВ
P = ---2m---' 632’ X^) =	’ 633’ A = k^^ ’
634. x = 4 - 2cost 635. I =	(1 - e-?*) . 636. I = ^e~t/RC.
ft X	J	n
637. I = -&e~t,RC. 638. I = -^-e-Rt/2Lsinut. CR2<dL, ш = __________ijjCL
=	639- 1 =	- <p), A =	4> = arctg
640. I = Asin(wt -99), A =	, v 2, 99 = arctg ;
Vя +(u’b-ZJc)
max A = при cv2 = ^. 641. Нет. 642. Да. 643. Нет. 644. Нет. 645. Да. 646. Нет. 647. Да. 648. Нет. 649. Нет. 650. Да. 651. Нет. 652. Да. 653. Да. 654. Да. 655. Нет. 656. Нет. 657. Да. 658. Нет. 659. Да. 660. Нет. 661. Да. 662. Нет. 663. а) Нет. б) Нет. 664. Линейно независимы. 665. Могут быть линейно зависимы или независимы. 666. а) Ж=(): б) ничего нельзя сказать. 667. Линейно независимы. Уравнение не удовлетворяет условиям теоремы. 669. Два. 670. а) — 1<ж<оо. б) |тг<ж<|тг. 671. а) Могут при п^2. б) Могут при п^З. 672. п^4. 673. п^2. 674. у" — у' ctgx = 0. 675. (х — - 1)у" - ху' + у = 0. 676. у" - у" = 0. 677. (2ж2 + 6ж - У)у" -— (4ж + 6)у' + 4у = 0. 678. у" — у = 0. 679. (ж2 — 2х + 2}ут — х2у" + + 2ху' — 2у = 0. 680. х2у" — Зху' + Зу = 0. 681. у = С±х + С2е-2ж. 682. у = Ci (1 + |)+С2 (f + 1 - In |ж + 1|) . 683. у = еж(С1Ж2 + + С2). 684. ху = Cie-® + С2еж. 685. у = Citg® + C2(l + a?tga?). 686. у = Ci(l+ ж1п|ж|) + С2ж. 687. у = С1(еж -1) + ^_. 688. у = = С1ж + С2(1пж + 1). 689. у = Ci 8тж + С2 (2 — sin ж х х In • 690. у = Ci(x - 3) +	691. у = Сю2ж + С2(3ж + 1)е-ж. 692. у =
= (Ci + С2ж)е“ж2. 693. у = С1(2ж + 1) + С2е2ж. 694. у = С1(ж +
162
Ответы
+ 1) + С2х~\ 695. у = Ci(x + 2) + С2х2. 696. у = Ст(®2+ 2) + + С2х3. 697. у = Ci(x2 + 1) + С2[х + (ж2 + 1) arctg»]. 698. у = = Сц/Щ + С2(х - 2). 699. у = С1Х + С2ех + С3е~х 700. у = = Cix + С2х-1 + Сз(®1п|®| + 1). 701. у = Cix + С2ех + Сз(®2 — 1). 702. у = Ci(»+2)+^-+(f + 1)1п |ж|4-|. 703. у = Ci(2x-1)+C2e~x+ +	704. У = £т + £21+х. 705. у = Ci(x2 + 1) + Сг®-1 + 2®.
706. z" + z = 0. 707. z" — z = 0. 708. z" = 0. 709. x2z” - 2z = 0. 710. 4®2й" + (4®2 + l)z = 0. 711. y"t — у = 0. 712. y"t + у = 0. 713. (t2 - l)y"t - 2y = 0. 714. y'tt + t2y = 0. 715. 8i/"f + t2y = = 0. 716. у = 1 + Ci(x — 1) + C2(x2 — 1). 717. J p(x) d;r—> + оо при x—> + oo. 719. На прямых у = 0 и х = Xi, где q(xt) = 0. 720. а) Нет. б) Да. в) Нет. г) Нет. 726. тг/д/ш; [(б — а)у/т/тг\ нулей или на один больше (квадратные скобки означают целую часть числа). 727. 0,33«/<0,5. 728. 15,7«/<32. 729. 0,49<Д<1. 730. 0,15<Д<1,2. 737. u't't + (±1 +	= 0, t =	у = ^и.
В тех из ответов 738—750, где решение у2 не указано, оно получается из yi заменой cos на sin.
738. yi = |cos^ + О(1/®4). 739. 1/1,2 = ®“1/2е±а,2/2(1 + + О(яГ2)). 740. у! = ^cos^ + O(®“5/2). 741. yi = е-®^2 cose®+ + О(е“3ж/2). 742. 1/1,2 = ж1/4е±2'/®(1 + О(ж-1/2)). 743. У1.2 = = ж-1^4е±?а,3/2(1 + О(»-3/2)). 744. yi = ж-3^4 cos2y/® + О(®-5^4). 745. yi = e(a!~1)2/2[(2»)~1/4cos (2a!f/2 + О(®~7/4)]. 746. yi = = | cos + О (У) . 747. i/i, 2 = ®(1±'/К)/2(1 + О(ж-2)). 748. У1 = = -/ЁгЕ [cos(lln2 х ~ I Inin®) + О(1п-2ж)]. 749. 1/1,2 = [1 ± 5^2 + + MjBr+°(;';“6)]'V5r- 750- 3/1 = ж1/4 С1 + ёЬ) cos (2V® + IeVs) + + О(ж-5/4). 751. у = (sh»/shl) — 2ж. 752. у = ж + е7х — е-1. 753. у = е® — 2. 754. у = 1 — sin® — cos®. 755. Решений нет. 756. у = 2х — тг + тгcosх + Csinx, С — произвольное. 757. у = = —2е~х. 758. у = е~х - 1. 759. у = -e(-1-i)®. 760. у = 2х3. 761. у = Зж2. 762. у = -ж“3. 763. а = (2п - 1)2тг2, п = 1, 2, 3, ... 764. G = (в - 1)ж (0^®^s), G = s(® - 1) (s^x^l). 765. G = sins cos ж (0^®^s), G = cos s sin® (s^x^tt). 766. G = = es(e~x - 1) (O^x^s), G = 1 - es (s^®^l). 767. G = -e~s chx (0^®^s), G = —e~x chs (s^x^2). 768. G = | sin |® — s|. 769. G = = 1-1 (1^® < s), G = 1 - 1 (s^®^3). 770. G = (l^s), G =	(s^®^2). 771. G = 1^ (l^s), G = Й
(s^®^2). 772. G = -ж (О^ж^в), G = -в (в^ж<оо). 773. G = -1 (O^.r^.s), G = — es~x (s^®<oo). 774. G = — In® (l^x^s), G = — Ins
Ответы
163
(s^®<oo). 775. G = |еа(е-3ж -e-ic) (0^®^s), G = |е-31С(еа - e3s) (s^x<oo). 776. G = (1 — x2)/2s2x (l^ar^s), G = (1 — s2)/2s2x (s^®<oo). 777. G = x(s3 - l)/3s2 (0<.r^.s), G = s(x3 - l)/3x2 (s^x^l). 778. G = -(l/2)e~lx~sl. 779. G = -x2/3s3 (O^x^s), G = = —1/3® (s^®<oo). 780. a / fc2?r2, к = 1, 2, 3, ... 781.
782- Xk = —k2ir2/l2, yh = sin(knx/l), к = 1,2,3, ... 783. Xk = —k2ir2/l2, yk = cos(k-nx/l), к = О, 1, 2, ... 784. A,.. = = — (fc — |)2 yk = sin (fc - I) к = 1, 2, 3, ... 785. Xk = = -(i^)2 -l. yk = V^™^,k = l,2,3, ... 786. x = Cie* + + С2е54, у = -Ci e* +3C2 e5t. 787. x = Cie~* + C2e3t, у = 2Cie"‘ -- 2C2e3f. 788. x = 2Cie3t - 4C2e~3*, у = Ci e3t +C2 e~3*. 789. x = = e2*(C'icos t+C2sint), у = e2<[(C'i+C'2)cosi+(C,2—Ci)smi]. 790. x = = e4(Ci cos3i4C2 sin3i), у = e4(Ci sin3i—C2 cos3i). 791. x = (2C2 — — Ci)cos 2t — (2Ci +C2) sin2t, у = Cicos 2t 4 C2sin2i. 792. x = (Ci 4 + C2t)e3f, у = (Ci + C2 + C2t)e3t. 793. x = (Ci + C2t)e*, у = (2Ci -- C2 + 2C2t)e*. 794. x = (Ci + 2C2i)e-t, у = (Ci + C2 4 2С24)е“! 795. x = (Ci + 3C2t)e2*, у = (C2 - Ci - 3C2t)e2*. 796. x = Cie* 4 4 C2e24 4 C3C *, у = Cic4 — ЗС3С *, z = Cie4 + C2e2< — 5Сзс *. 797. x = Ci + 3C2e2t, у = -2C2e2t 4 C3e~*, z = Ci + C2e2< -— 2C3e-<. 798. x = C2e2< 4C3e3<, у = Cie‘ + C2e2<, z = Cie4 + C2e2t + 4 Сзе3<. 799. x = Cie* + C2c24 4 C3C54, у = Cie* — 2C2e2* 4 C3C54, z = —Cie* - 3C2e2* 4 3C3e5t. 800. x = Cie* 4 C3e~*, у = Cie* 4 + C2e2*, z = 2C2e2* — C3e-f. 801. x = e*(2C2sin2t 4 2C3cos2t), у = e*(Ci — C2 cos 2t + C3 sin2t), z = e*(—Ci — 3C2 cos 2t + 3C3 sin2t). 802. x = Cie2* + e3*(C2cost 4 C3sint), у = e3*[(C2 + C3)cost 4 4 (C3 — C2) sin t], z = Cie2* 4 e3* [(2C2 — C3) cos t 4 (2C3 4 C2) sin t]. 803. x = C2 cosi4(C242C3)sini, у = 2Cie*4C2cosi4(C242C3) sint, z = Cie* 4 C3cost — (C2 4 C3)sint. 804. x = Cie2t 4 (C2 4 C3)e3t, У = Cie2t 4 C2e3t, z = Cie2t 4 C3e3t. 805. x = Ci 4 C2e‘, у = 3Ci 4 4 С3е4, z = -Ci 4 (C2 - C3)e‘. 806. x = Cie3t 4 С2е~\ у = -Cie3t 4 4 (C2 4 2Сз)е“‘, z = -3Cie3t 4 С3е~*• 807. x = Cie24 4 C3e~M, у = C2e2t4 3C3e“5<, z = (Ci - 2C2)e2t 4 2C3e~M. 808. x = (Ci 4 4 C2i)e4 4 C3e2<, у = (Ci - 2C2 4 C2t)e4, z = (Ci - C2 4 C2t)e4 4 4 C3 e2t. 809. x = (C2 4 С3^е~*, у = 2Cie4 - (2C2 4 C3 4 2Сз^е~*, z = Cie* - (C2 4 C3 4 С31)е~*. 810. x = Ci 4 C2t 4 4C3e3t, у = C2 -- 2Ci - 2C2t 4 4C3e3t, z = Ci - C2 4 C2t 4 C3e3t. 811. x = (Ci 4 4 C3t)e*, у = (C2 4 2C3t)e‘, z = (Ci - C2 - Сз - C3t)e*. 812. x = = (Ci 4 C2t 4 C3t2)e2*, у = [2Ci - C2 4 (2C2 - 2C3)t 4 2C3t2]e2*, z = [Ci - C2 4 2C3 4 (C2 - 2C3)t 4 C3t2]e2*. 813. x = 3Cie* 4 4 3C2e~* 4 C3 cost 4 C4 sini, у = Ci e* +C2 е~* 4C3 cost 4 C4 sini. 814. x = -2е‘(С14С24С24)-2е-‘(С3-С44С44), У = e‘(Ci4C2t)4
164
Ответы
4-е *(Сз4-С^). 815. х = e*(Ci cos£+62 sint)-|-e *(Сз cost-hCi sint), у = e*(Ci sint — C2 cost) + e-*(C4 cost — C3 sint). 816. x = Сю* 4-4- C2C 4- Сзе2* 4- C§e 2*, у = Сю* 4- C2C 4- Сю2* 4- Cqc 2*, z = = Сю* + С2е-* — (C3 + C4)e2* — (C5 + C6)e-2*. 817. x = ЗСю* + С2е-*, У =	+ C2e-*. 818. X = Сю* + C2e-* + 2C3e-2*, у = 2Сю* +
+ С3е-2*. 819. х = ЗСе-*, у = Се~\ 820. х = -2С2е3* + С3е*, у = Сю-* + Сге3*. 821. х = 2Сю2* 4- 2C2O~2t 4- 2Сз cos 2t 4- 2С4 sin 2t, у = 3Cie2* — ЗСге-2* — C3 sin2t 4- C4 cos 2t. 822. x = Cic? — 4C2e-2*, у = Cici + C2e~2t. 823. x = (Ci + С21)с* + C3e-*, у = (-2Ci -- C2 - 2C2t)e* - 4C3e“*. 824. x = Сю* + С2с1 + C3e2* + C4e-2*, у = Cic1 4- 5G*2C t 4- 2Сзе2* 4- 2C4e 2*. 825. x = Ci 4- C*2C* 4- C3 cos t 4-+ C4sint, у = —Ci - C2Ot + (|C4 — |Cs)cost — (|Сз + |C4)sint. 826. x = Сю* + Сге-* 4- te+ — t2 — 2, у = Сю* — Сге-* 4- (t — l)e* — 2t. 827. x = Сю2* + Сге-* — 2sint — cost, у = 2Сю2* — Сге-* 4- sint 4-4- 3cos t. 828. x = Сю* + 2C2e4t 4- Зе^*, у = —Cic* 4- Сге4* 4- e5*. 829. x = Ci(cos2t — sin2t) 4- C2(cos2t 4- sin2t), у = Cicos2t 4-4- C2 sin 2t 4- e~2t. 830. x = Cie2t 4- C2O3t 4- (t 4- l)c2*, у = — 2Cie2t — - C2e3t - 2te2t. 831. x = (Ci + 2C2t)c*- 3, у = (Ci + C2 + 2C2t)c*- 2. 832. x = Сю2* + 3C2e4* — e-* - 4e3*, у = Сю2* + C2e4* - 2e-* - 2e3*. 833. x = Сю* cost 4- Сге* sint 4- e* 4-1 4- 1, у = Сю*(— cost — sint) 4-4-C2C*(cos£—sint) — 2c* — 2t — 1. 834. x = Cic“*4-2C2e2*—cost-|-3sint, у = —Сю-* 4-Сге2* 4-2 cost —sint. 835. x = 4Сю*4-С2е-2* — 4te*, у = = Cic*4-C2c 2* — (t — l)e*. 836. x = Cic3*4-3f24-2f4-C2, у = —Сю3*4~ 4-6f2 —2f4-2C2 —2. 837. x = 2Сю2ЧС2с-3*-(12^+13)с*, у = C\a2t--2C2c-3*-(8t+6)c*. 838. x = 2Cic8t-2C2-6f+l, у = ЗСю8ЧС2 + 4-3£. 839. x = 3Cic* 4-C2C-* 4-3 sinf, у = Сю* 4-Сге-* — cos £4-2 sin£. 840. x = Ci cost 4- C2 sinf 4- t sinf — t cost, у = Ci(sin£ 4- cosf) 4-4- C2(sinf — cosf) — 2t cost 4- sinf 4- cost. 841. x = (Ci 4- C2t — t2)^, у = [Ci — C24-^(Сг4-2) — f2]c*. 842. x = Cic*-ЬЗСге2* 4-cosf — 2sinf, у = Cic* 4- 2Сге2* 4- 2cos£ — 2sin£. 843. x = Сю* 4- Сге3* 4- te* — — с4*, у = —Cic* 4- Сге3* — (£ 4- l)e* — 2c4*. 844. x = Cicos2£ — — C2 sin2t 4- 2t 4- 2, у = (Ci 4- 2C2) cos2£ 4- (2Ci — C2) sin2£ 4- 10£. 845. x = Сю* 4-C2e3*4-c*(2 cosf — sinf). у = Сю* —Сге3* 4-e*(3 cos£4-4- sinf). 846. x = Ci cost 4- C2 sinf 4- tgf, у = —Ci sinf 4- C2 cost 4- 2. 847. x = Cic*4-2C2e2*—e* ln(e2*4-l)4-2e2* arctge*, у = Сю*4-ЗС2е2* — — с* ln(c2* 4- 1) 4- Зе2* arctge*. 848. x = Ci 4- 2C2©-* 4- 2e“* ln|e* — 1|, у = —2Ci — ЗСге-* — Зе-* ln|e* — 1|. 849. x = Ci cost 4- C2 sint 4-4- t(cost 4- sint) 4- (cost — sint) In | cost|, у = (Ci — C2) cost 4- (Ci 4-4- C2) sint 4- 2cost ln| cost| 4- 2tsint. 850. x = (Ci 4- 2C2t — 8t5/2)e*, у = (Ci + 2C2t - C2 - 8t5/2 + 10t3/2)e*. 851. x = Сю3*(£)+ C2e3*(0). 852. x = Cie2*(l)+Сге-^Д). 853. x = Сю"*(1) + Сзс-*^2!^.
Ответы
165
854. х = С1е‘(сов^п24) + С2е‘(8Й12^2с‘об2<). 855. х = Cie‘(o) + + С2(1) + Сзе-‘(1). 856. х = Cie2<Q) + С2е‘(-1) + С3е-‘(-1). +/0\	/ 2 cos t \	/ 2 sin t \
857. х = Cie* (i) + C2I 2cost ) + Сз I 2sin* ). 858. x = \1/	\3 cost — sin tj	\3sint+cost/
= Cie“2tf 1^ +C2e-<f-sin2t^ +C3e-<fco”2i\ 859. x = С1е~*( h + \—1/	\ cos 2t/	\sin2t/	\-l/
f. /cost-sin t\	,/cost+sint\	„	+ / Ц
+ С72О (	cost	) -|-	C3C I sint ).	860. x = Ci (-1)	+ C2e* (-1) +
\	sin t	J	\ — cos t /	\ 1/	\ 0/
+ C3e*(o). 861. x = Cie3tQ) + С2е“*(-1) + Сзе-^ о). 862. x =
= Ci + C2e* + Сзе* . 863. x = Cie *	+ C2c^ +
+ Сзе* (2Д• 864. x = Cie-t	+ C2e-t Q) + Сзе-*	• 865. x =
= Cie2< (~o) + C2e2t (?) + Сзе2< (T) • 866- X = C1 (D + °2 (2V1) + + Сз(‘^У’ 867, КЛ)’ 868,	869’ Сой’
870. f2e227 ®-e22'). 871. (	-34). 872. (o i o'). 873. | n % J |.
\2e<2e 2e-e2/	V 1 3 '	\0 0 e2/	I 0 e e I
\ 0 0 e2 /
874. e2. 875. e"1. 876. x = dcosaf, у = ^sinaf; эллипс (§)2 + + (^)2 = 1. 877. x = Ci sin + C2) , у = jCi sin + C2) ;
x = Cssin(at + Ct), у = —Cssin(at + Ci). 878.	(yj- +
879. I = Asin(cjf — c?), A =	, v =: max A =
y/K2 + (uL/(l-a>2LC))
= при uj = 0 и uj = oo, min A = 0 при w2 = 880. Л 7^	к = 0, ±1, ±2, ... 881. а) неустойчиво;
б) устойчиво; в) устойчиво; г) неустойчиво. 882. Асимптотически
устойчиво. 883. Неустойчиво. 884. Неустойчиво. 885. Устойчиво.
886. Устойчиво. 887. Неустойчиво. 888. Устойчиво. 889. Все решения стремятся к нулю. Нет, нет. 890. Устойчиво. 891. Асимптотически устойчиво. 892. Неустойчиво. 898. Нет. 899. Устойчиво. 900. Неустойчиво. 901. Неустойчиво. 902. Устойчиво. 903. Неустойчиво. 904. Устойчиво. 905. Устойчиво. 906. Неустойчиво. 907. —2 < а < -1. 908. а < -1. 909. ab < -3. 910. а < b < -1.
911. О < а < 2. 912. —be < а < —е. 913. Устойчиво. 914. Неустойчиво. 915. (О, 0) неустойчиво, (1, 2) устойчиво. 916. (1, 2) и (2, 1) неустойчивы. 917. (2&тг, 0) неустойчивы, ((2fc + 1)тг, 0) устойчивы. 918. (3, 2) неустойчиво, (0, —1) устойчиво. 919. (2, 1) устойчиво, (—2, 1) неустойчиво. 920. (1, 1) неустойчиво, (-4,-4) устойчиво. 921. (2&7Г, 0) неустойчивы, ((2fc + 1)тг, 0) устой
166
Ответы
чивы. 922. (—1, 2&7г) устойчивы, (—1, (2к + 1)тг) неустойчивы. 923. Неустойчиво. 924. Устойчиво. 925. Устойчиво. 926. Неустойчиво. 927. Устойчиво. 928. Устойчиво. 929. Неустойчиво. 930. Устойчиво. 931. Устойчиво. 932. Неустойчиво. 933. Устойчиво. 934. Устойчиво. 935. Неустойчиво. 936. Устойчиво. 937. Неустойчиво. 938. Неустойчиво. 939. Неустойчиво. 940. Устойчиво. 941. Неустойчиво. 942. Устойчиво. 943. Неустойчиво. 944. Неустойчиво. 945. Устойчиво. 946. Неустойчиво. 947. Устойчиво. 948. Неустойчиво. 949. а>0, 6>0, ab>2. 950. За>6>0. 951. 0<а<2. 952. Неустойчиво при всех а. 953. а>0, 6>0, а + Ь<1. 954. 6>0, a>b + 1. 955. а>0, 6>0, 8а — а2Ь> >4. 956. а>2, 6>0, 2аЬ — Ь2>4. 957. а>0, 6>0, 2 - у/3<|<2 + д/З. 958. 0<а<8, 0<6<8а - а2. 959. а) устойчиво; б) устойчиво; в) неустойчиво; г) неустойчиво; д) неустойчиво; е) устойчиво. 960. -4 < «6 < 0 и а = b = = 0. 961. Седло. 962. Узел. 963. Фокус. 964. Узел. 965. Седло. 966. Центр. 967. Вырожденный узел. 968. Узел. 969. Особый узел. 970. Фокус. 971. Узел. 972. Вырожденный узел. 973. Фокус. 974. Седло. 975. Центр. 976. Вырожденный узел. 977 и 978. Особые точки заполняют прямую линию. 979. (—2, —1) — узел. 980. (1, —2) — фокус. 981. (4, 2) узел, (—2, —1) фокус. 982. (1, 0) особый узел, (—1, 0) седло. 983. (1, 1) фокус, (—1, —1) седло. 984. (О, —1) вырожденный узел, (2, —3) седло. 985. (2, 4) узел, (—1, 1) седло. 986. (1, 1) фокус, (—1, —1) седло. 987. (2, 1) узел, (1, 2) седло, (—1, —2) фокус. 988. (1, —1) фокус, (О, —2) седло, (—2, 2) узел. 989. (—2, 4) узел, (1, 1) фокус, (2, 4) и (—1, 1) седла. 990. (—2, 2) вырожденный узел, (1, —1) фокус, (2, 2) и (—1, —1) седла. 991. (3, 0) фокус, (1, 1) узел, ( — 1, 1) и (—3, 0) седла. 992. (О, 1) и (0, —1) седла, (—1, 0) фокус, (3, 2) узел. 993. В области у>0 интегральные кривые расположены как у седла, в области у<0 — как у узла. 994. Через (0, 0) проходит одна кривая, имеющая там точку возврата первого рода. Остальные кривые не заходят в особую точку. 995. Из области у<0 все интегральные кривые обоими концами входят в особую точку, а из области у>0 не входит ни одна. 996. Две интегральные кривые проходят через особую точку, касаясь друг друга. Остальные кривые расположены, как у седла. 997. Из области у>0 кривые не входят в особую точку. В области ?/<0, ж<0 расположение кривых напоминает вырожденный узел, а в области ?/<0, ж>0 — седло. 1021. (О, 1) седло, (О, —1) фокус. 1022. (1, 2) седло, (—1, 2) узел. 1023. (1, 0) седло, (0, 2) вырожденный узел. 1024. (О, 1) центр, (0, —1) седло. 1025. (2, 2) узел, (0, —2) седло, (—1, —1) фокус. 1026. (2, 2) седло, (4, 1) и (—2, —2) фокусы. 1027. (1, 0) и (—1, 0) седла, (0, 1)
Ответы
167
и (О, —1) центры. 1028. (1, 1) седло, (1, —1) узел, (2, 2) и (—2, 2) фокусы. 1029. (О, 1) и (О, —1) седла, (1, 0) фокус, (—3. 2) узел. 1030. (1, -1) и (-1, 1) узлы, (3, 3) и (-3, -3) седла. 1031. (1, -1) и (—1, 1) седла, (3, 3) и (—3, —3) узлы. 1032. (О, 0) фокус, (7, 1) узел, (0, 8) и (3, —1) седла. 1033. (О, 0) фокус, (2, 4) узел, (1, 1) и (—1, 1) седла. 1034. (2, 1) узел, (—1, 2) фокус, (1, 2) и (1, —2) седла. 1035. Ift + gsin^ = 0. 1036. mlft + kl2ft\ft\ + mgsincp = 0. 1037. ft + sin 9? = |. 1038. mx + f sgn± + kx = 0. 1039. В (L/T)3 раз. 1047. /(го) = 0: при возрастании г функция /(г) меняет знак с + на —; меняет знак с — на +; не меняет знака при переходе через нуль. 1048. а< — 1/2; а> — 1/2. 1053. х = = ±6cth—7°	1054. х = у, 4Лх2 + у2) = — 2yF(y)<t] при
2yi-a2
у 0. 1056. Меньше, чем на 0,03. 1057. Меньше, чем на 0,05(е2Т -— 1). 1058. Ошибка меньше 0,081. 1059. \у — у\<0,016. 1060. \х — - ж| + \у - ?/|<0,0012. 1061. \у - у\< <0,002. 1062. \у - у\<0,015. 1063. \у - г/|<0,034. 1064. е2ж - х - 1. 1065.	+ х +
+ 1. 1066. ех~2. 1067. tie.-1 - е“‘). 1068.	 Ю69. £®.
1070. t2lnt + 2t2 - 2t. 1071. —e2< - 2e“‘ - 3e“2t. 1072.	-
-	+(£ - f)e“‘ +I-1073- т - it-1074- у = I + м (®2 - £) +
+ /z2 f + ^) +	1075- У = 2^+ 2/z(®-1/2-
—x2)+n2(\x7/2 — |я: + |fa:_1/2— ar-3/2^+O(^3). 1076. у = l+/z(x2 — - x) +	+ O(/?). 1077. у = 1 + 3#t +	(^ - Зш) + О(ц3).
1078. у = x - ц{х + 1) + (^2/2)(еа’ - x2 - 2x - 1) + Oiy3). 1079. x = = sinf +	— I cos 2f) + p2 (| sinf — | sin3f) 4- O(/z3). 1080. x =
= cos2£ + /z(^-^cos4f) + /z2(y^cos2£ + 6^ cos6£) + O(/z3). 1081. x = /zcosf + p3 (— | cost + cos3f) + O(p5). 1082. ^ = 14-+ /zsin£ — ^(1 + cos 2f) + O(/z3), Ж2 = —1 — з sin£+ (1 — | cos 2t) + + O(/z3). 1083. xi = — sin2£ + (sin2£ — sin6f) + O(/z5), X2 = = тг—у sin2£— iqqq (| sin2£— —	sin6f)+O(/z5). 1084. x = | sint+
+ | sin2£— | sin3£4-О(//). 1085. x = 2pi sinf — p (^ sin£ 4- | sin3£) + + О (/it) . 1086. x = C cos t 4- C2 (| — | cos r — | cos 2т) 4- O(C3), т = f (1- ^C2 + O(C3)) + C2. 1087. x = Ccost 4- (cos 3т -- cos г) + O(C5), T = t (1 + |C2 + O(C4)) + C2. 1088. x = C cos т + + ^(cost-cos3t) + O(C5), t = t(1 -	+ O(C4))+C2. 1089. x =
= 2cost 4- |/isinT — sin3T 4- O(/z2), т = t (1 — 4- O(/z4)) 4- C. 1090. x = cosт4~ sin3T + O(/z2), т = (1 - +O(/z3)) t + C.
168
Ответы
1091. у = 1 + х+^- + ^- + ^+ ... 1092. у = 1 + х +	+
-I- ... 1093.?/=	+v +V +	1094. у = х + х2-^-^- ...
1095.	у = 1 + 2(ж - 1) + 4(ж - I)2 + f (ж - I)3 + ^(ж - I)4 + ...
1096.	у = 1 + 2ж-^-^-^ - ... 1097. у = 4 - 2ж + 2ж2 -— 2ж3 + ^ж4 + ... 1098. 7?>0,73. 1099. Ошибка меньше 0,00024. 1100. г/1 = 1+|1 + ^_ + ..„г/2=ж+^ + 5^_+... 1101.7/1 = = 1 + ^ + ^ + у^+...,г/2=Ж+4+Й + г^+-- - =хЛ. 1102. 7/1 = 1 + ж2 + ж4 + ... =	,	7/2 = Ж + ж3 + Ж® + . . . = 1^.
1103.	7/1=1— |ж2 + |^|ж4 — . . . = (1 + ж2)“5, 7/2 = ж — |ж3+ Ж® — - . . . 1104. 7/1 = 1- ^- ^- ^- ... ,7/2=ж + ж2+^ + ^ + + . . . 1105. 7/1 = 1 + Ж - ж3 - ж4 + ж6 + ж7 - . . . = 377^^5, 7/2 = Ж7/1. 1106. 7/1 =	, У2 =	• •  1Ю7. 7/1 = 1-< +
+ <+..., Т/2=Ж-^ + ^+... 1108.7/1 = 1+< + < + <+..., У2=Х+^+^+ ... 1109. 7/1 = 1-^+ ...,У2= ж+<-^ + + . . . , 7/3 = Ж2 + ^ - . . .	1110. 7/1 = 1- ^ + ^- ...=^,
2/2 =	+	mi- У1 = 7 + 1 + ^ + ^ +
+ ... = У2 = |ж|1'2(1 + ^+^ + ^+ ...). 1112. 7/1 = — ж1/3 (1+-^—Ч---—--1- и? — ж2/3(	--------1_
— X	I 5-6 ' 5-6-11-12 ' • • ‘У ’ У2 — х	I 6-7 ' 6-7-12-13 ' * * * ) *
1113.	У1 = | + 1 + f, У2 = X2 +	+ ... =
__ ft (еж — 1	1 х\ -| -| -| л	_ 1	1	।	1	। х2	। х3	।	7 я;4 - 6	-1 “ 2) ' 1114’	- ^-я + 2 + -8-+40 + 720 +
+ ••• ,7/2=^+^-+^- + ^+ ... 1115. Т/1=ж + ж2+^- + ^у + + ... = же®. 1116. 7/1 = 1 + ^ + ^ +	+ ... 1117. 7/2 =
= (1+ 2? + 2^ + •••)1п|ж| -	- ... 1118. 7/1 И 7/2 —
обобщенные степенные ряды с иррациональными показателями.
1119. у\ и У2 — ряды с комплексными показателями. 1120. Решений в виде обобщенных степенных рядов нет, так как получаемый ряд у = 1Ч-1!жЧ-2!ж2Ч-3!ж3Ч- ... имеет нулевой радиус сходимости.
11O1 Л. _ 7Г I 1	cos(2fc—1)®
1121. у - - g + - X (2fc-l)2(fc2-fc+l)’ к = 1
1122. у = - + - V 1кь4 1ц.2.1 (cos2fca? — 2^ sin2fcaA. тг тг 16fc4 — 4fc^ + l \	4kz — 1	J
1190	____у-'' (fc3 +fc) cos fca —sin kx
у —2^	2fc[(fc3+fc)2 + l]	'
oo
1124 7/ =________-______I- — V cos 2k7rx
У 67Г2 ' 7Г4 к2 (4fc2 +1) * к = 1
Ответы
169
1125. у = £ fc2S(^4g2) + Ci cos 3® + C2sin3®. 1136.
1137. 1 + ®2<«/<l + x2 + arctg®. 1141. у = С2еС|Ж, z = 2c,c2e~Cia:-1142. у = С2еС1Х, z = x + §^е.С1Х-, у = 0, z = x + C. 1143. у = = г =	1144- У = C2eC11'2, z = ^xe~Clx2-, у = 0.
z = Cx. 1145. у =	+ ^(x + C2) - ^(ж + C2)2, z = ^(x +
+ C2)2 +	1146. у = Ciz, x = 2y - z + C2. 1147. ж2 — y2 = Ci,
x + у = C2Z. 1148. x — у = C\(y — z), (ж + у + z)(x — у)2 = C2. 1149. ж + z = Ci, (ж + у + z)(y — Зх — z) = C2- 1150. ж2 — z2 = Ci, у2 — и2 = C2, (ж + z) = Сз(и + у). 1151. ж + z = Ci, у + и = = C2, (ж — z)2 + (у — и)2 = Сз- 1152. ж2 — 2у = Ci, бху — 2ж3 — - З^2 = С2. 1153. у2 + z2 = Ci, ж - yz = С2. 1154. ж = С±у, ху — z = С2Х. 1155. ж = Cvy, ху — 2\/ z2 + 1 = С2. 1156. у = = Сцг, ж — у2 — z2 = C2Z. 1157. у2 + z2 = Ci, х(у — z) = С2. 1158. xz = Ci, ху + z2 = С2. 1159. ж + z — у = Ci, In|ж| + “ = С2. 1160. ж2 +у2 + z2 = Ci, yz = С2ж. 1161. 1) да; 2) нет. 1162. 1) нет; 2) да. 1163. Да. 1164. Зависимы. 1167. z = /(ж2 + у2). 1168. z = = f(xy + У2)- 1169. и = fty/X'Z/x). 1170. и = у ^(ж~^)/а;+^+2г) ) 1171. F(®2 - у2, х - у + г) = 0. 1172. f(b.~x-y-\z + J;1^) = = 0. 1173. F(x2 - 4z, (ж + у)2/х) = 0. 1174. F(x2 + у2, z/x) = = 0. 1175. F f—, ху - —) = 0. 1176. F (	+ 4, -J- +	= 0.
У у ’ н х J	\«+г/ г’ х — у	z )
1177. F(x2 + у\ y(z + г^ТТ)) = 0. 1178.	1п|®?/| - 4) =
= 0. 1179. Е(ж2 + у2, arctg^/?/) + (z + 1)е-г) = 0.
1180. F(z2 — у2, ж2 + (у — z)2) = 0. 1181. F (^, 2х — 4z — у2) = = 0. 1182. F(z —1п|ж|, 2ж(;г — 1) — у2) = 0. 1183. F(tg z +ctg ж, 2у — -tg2z) = 0. 1184. f{^^, (х - у)(х + у - 2z)) = 0.
1185.	F ((ж - y)(z + 1), (.'/ + t/)(z - 1)) = 0.
1186.	F(u(x - у), и(у - z), (х + у + z)/u2) = 0.
1187.	F (х/у, ху — 2гл, (z + и — ху)/х) = 0.
1188.	Р((ж — y)/z, (2и + ж + y)z, (u — x — y)/z2} = 0. 1189. z = = 2ху. 1190. z = уех - е2ж + 1. 1191. z = y2e2Vx~2. 1192. и = (1 -— ж + у)(2 — 2ж + z). 1193. и = (ху — 2z)	• 1194. у2 — ж2 —
— In у/у2 — ж2 = z — In|?/|. 1195. 2х2(у + 1) = у2 + 4z — 1. 1196. (ж + + 2у)2 = 2ж(;г + ху\ 1197. y/z/у^ sin ж = sin^/z/y. 1198. 2ху + 1 = = ж + Зу + г-1. 1199. х - 2у = х2 + у2 + z. 1200. 2х2 - у2 - z2 = = а2. 1201. [(y2z — 2)2 — ж2 + z]y2z = 1. 1202. ж2 + z2 = 5(ж;г - у). 1203. 3(ж + у + z)2 = ж2 + у2 + z2. 1204. xz = (xz — у — ж +
170	Ответы
+ 2z)2. 1205. (1 + yz)3 = 3g/^(l + yz — x) + y3. 1206. x + у + z = = 0. 1207. 2(ж3- 4z3 — 3yz)2 = 9Q/ + z2)3 1208. (ж - ?/)(Зж + у + + ^z) = ^z. 1209. xz + y2 = 0. 1210. z = xy + f(y/x)^ где f — произвольная дифференцируемая функция, для которой /(1) = 0. 1211. F(x2 - у2. 2ж2 + z2) = 0. 1212. 2у2 + z2 = z(x2 + у2 + г2). 1213. F(bx — ay, cx — az) = 0. 1214. х2 +3у2+z2 +3xy+xz+3yz = 1. 1215. F((y — Ь)/[х — a), (z — cj/(x — а)) = 0. 1216. F(x2/y, z/y) = 0. 1217. z = Cxy2. 1218. Решений нет. 1219. z = 0. 1220. Решений нет. 1221. x3y2z = С. 1222. z = у2 - ху. 1223. x2yz = С - ж3; ж = 0.
ОТВЕТЫ К ДОБАВЛЕНИЮ
9. у' = и, и = v, v' = уе х(2и — ж); уъ = 1, uq = 1,	= 0;
ух = 1 + ж, щ = 1, vi = 1 + (ж - 1) е~х . 10. у0 = 1, ух = 1 + I + + t2, У2 = 1 + t + 2t2 + t3 + t4/2 + *75. 11. a) yo = 0, yi = 2^1, 3/2 = f + ^- ^- + ^ — 55; б) например, |.r — 1|^|. 12. При .7,^(1 у = —ж, при ж>0 решение не существует. 13. б) При — 1 и а = 0. 14. а) При а^О, а^1 требуется — 1, ±3, (к + 1/2)тг (к = = 0, ±1, ±2, ...); при а = 1 требуется — 1, ±3; при а = О уравнение не дифференциальное; б) —3<£< — тг/2. 15. уо = ух = = У2 = • • • =4, lim ук = 4. 16. 7/(®0) = уо, у'(х0) = уг, у"(ж0) = к—>оо
= </2, где жо^Ьг/2 (к = 0, ±1,±2, ...), т/о>0, ух и у2 любые. 17. Начальные условия ух (0) = 1, у'х (0) = 1 и 7/2(0) = 1, 3/2(0) = д/2 различны. 18. п/1. 19. п>5. 20. п>3. 21. п>4. 22. п>4. 23. а^О, ±2 бесконечно много решений, а = 2 и а = 0 одно решение, а = — 2 нет решений. 24. а^О, ±1 бесконечно много решений, а = 0 и а = — 1 одно решение, а = 1 нет решений. 25. п 3 бесконечно много решений, п = 2 одно решение, при п = 1 для а = ±1 одно решение, для аф ± 1 нет решений. 26. п^З бесконечно много решений, п = 2 одно решение, при п = 1 для а = 1 и а = —2 одно решение, для а^1, аф — 2 нет решений. 27. При — 4 нет решений; при а> — 4 для 7/^3 бесконечно много решений, для п = 2 одно решение; для п = 1 при а = — 3 одно решение, при — 3 нет решений. 28. Да. 29. |ж|<д/2- 30. а) у = (in д/|ж2 — тг2| +	,
у = 0; б) у = 1/ln (Vtt2 — ж2 — 1) , |ж|<7г. 32. Нет. 34. |£|<1; для х = ж2 (жЕЯ1), ж(0) = 1 имеем ж = 1/(1 — t) (£<1); для х = —ж2, ж(0) = 1 имеем ж = 1/(1 +1) (t> — 1). 40. б) (J J). 44. б) ri^k + 1. 45. тп = 2. 46. ух = ж, у2 = ж2; W = ж2. 47. а) —2^ж^1; б) да; в) 1/4. 48. а) — 1<£<тг/2; б) да; в) -16(£ + I)-2; г) у = ^^>x(t) + + 6<p2(f) - ^^з(0- 49. а = -5. 50. 2. 51. 4. 52. (ж2 - 2х)у" -— 2(ж — 1)уг + 2у = 0. 53. у = 2 + 5ж — 2ж2. 54. Да, у = Зт/i — 2у2. 55. у = С1ж + С2Ж3-|-Сз(ж2-|-ж1п|ж —1|). 57. у = ж —2(еж—ж)/(е—1). 58. у = ж + Сх + С2(х + ж2). 59. у = 2 — ж2. 60. (ж2 + 4)2/" — 2ху' + + 2у = 2. 61. у =	+ ж - 1. 62. у = С1еж+С2(ж + 1) +
172
Ответы к добавлению
+ хех. 64. Т = Зтг/4. 65. Нет. 66. р = 7г2. 67. а = 2. 68. х = = (<71 4- C2t 4- у) е*+1 cos t- 69. х = Ci cos 2t 4- C2 sin 2t + (sin 2t — — cos 2t) — | cos 21. 70. у = (Ci 4- ж) cos x + (C2 4- ж2) sin ж. 71. у = = Ci со8ж+С2 8тж+(ж 4- I) е2ж—2жсо8ж. 72. у = (С1+С2О е* +Сз + 4-t2(at+b) ef +(c£ + d) ef cos £4-(ft 4-g) ef sint + ht2 + kt. 73. у = (Ci + 4- С2Ж) е2ж + (аж 4- 6)ж2 е2ж + е2ж(ссо8ж + dAnx). 74. у = Ci е^1+^ж 4-+С2е-(1’н)ж +ое(ж)ж +Ье(1"*)ж . 75. у = (Ci + С2ж) eix +аж2 eix + +be~ix. 76. у = С^^-^+Сзе^^^+ае^+^+Ьже^"2^. 77. у = Схе^+Сзе^-^^+Сзе^^-^^+аже^+Ье-2^ 78. Да.
79. Нет. 80. Нет. 81. с^О, ±2. 82. с^О, b любое или с = О, b = ±1. 83. а) ш^О, ±72: б) у = С +	= <? + Пт sill3t 84- х =
=	А = .	4 „	. 85. а) а = ±2, а = ±6;
(25-и>2)2+и>2	">	y/(25-cv2)2+cv2	7
б) а нечетное. 86. а) а = 2; б) а^2, а^О. 87. а = 0, а = 4. 88. а = О,
а = ±1. 89. а<0. 90.	± 1,	± 2. 91. а = 0, а = 4, /3 = 2.
92. а = 1, b = 0, с = —4; а = О, (3 = 2, 7 = 0. 93. ж = (Ci+ C2t) е2* + +3, у = (С\ + С2 Ч- C^t) е2^ 4~1. 94. ж = (>ie^(cos3t — 2sin3t) 4~ 4- С2 е* (2 cos 3t4- sin 3t), у = Ci e* (cos 3t 4- sin 3t) 4- C2 e* (sin 3t — cos 3t). 95. ж = Ci + [C2 + C3(t + 1)] e"*, у = [C2 + C3(t - 1)] e~\ z = Ci + 4- (C2 4- Cst)e-t. 96. Все ReA? = 0 и в жордановой форме все / 1	t	о\
клетки размера 1. 97. п>7. 98. о	1	о . ЮО. ж = 2С +
\е* -1 е* -1-t е*/
4- 2 sint, у = С 4- sint. 101. ж = —3 — cos 2t, у = —3 — cos 2t — 2 sin2t.
102. а) ж = Ci cos t 4- C2sint — sin2t — 2cos2t, у = Ci(cos t 4-4- sint) 4- C2(sint — cost) — 2sin2t. б) ж = — sin2t — 2cos2t, у = = —2sin2t. 103. a = —2. 104. При a = —2b. 105. Тот же ответ, что в задаче 96. 107.	и"). 108.	5e‘jint А •
\ t l+2t/	у -e’sint ег (cos t+2 sin t) J
109. (V?8^. 110. Ро'Л’"П- 111- СгЧЯ- 112- a) 1: \sh t 0 ch t)	\ 0 0 e2* /	\ 2-V3 /
Al, 2 = 1:6) e+2‘ 1=‘2t) . 113. a) e2‘. A1; 2 = e‘: б)	.
114. a) 1; A1>2 = cost±«sint; 6) (coss^int <^1^) • U5- (e2 -l)(e-— e x)2. 116. Bee ReAi<0. 117. eAln*. 118. Нет, например A = = (2™ 2^). 119. A2 = 0, жорданова форма может содержать только клетки вида (0) и (^ J). 120. Да. 121. Да. 122. Да. 124. Все \p,i|<1. 125. е(°+1/2)7Г. 126.	— 1/2. 131. Все ReA?^0 и для тех А*, у ко-
торых Re А« = 0, клетки в жордановой форме имеют размер 1.
133. См. ответ задачи 131. 134. a) det А = 0; б) см. ответ задачи
131. 135. Да. 136. Да. 137. Нет. 138. При п = 1 да, при п^2 нет, см.
рисунок к задаче 889. 139. ж = 0 неустойчиво. 140. ж = 0 асимптотически устойчиво. 141. ж = 0 асимптотически устойчиво, ж = = тгк (к = 4=1, ±2, ...) неустойчивы. 142. ж = 0 асимптоти
Ответы к добавлению
173
чески устойчиво. 143. х = 0 устойчиво. 144. х = 0 устойчиво. 145. х = у = 0 устойчиво. 146. (0. 0) устойчиво. (2/3, 0) неустойчиво. 147. (а, а) устойчиво, (а + 1, а) неустойчиво (aER любое). 148. а) а>0; б) а = 0; в) а<0. 149. а) а < —1; б) а = —1; в) а > —1. 150. а) а<0; б) а = 0; в) а>0. 151. а) а>0; б) а = 0; в) а<0. 152. а) а<0; б) а = 0; в) а>0. 153. а) а>2; б) а = 2; в) а<2. 154. а) а>0; б) а = 0; в) а<0. 155. а) 0<а<1; б) а = 0; в) а<0 и а^1. 156. а) а. = -1/2; х = с, у = 2с - 1/2; б) да. 157. Да. 158. а) а = 0, а = 4; б) для а = 4 устойчивы, для а = 0 нет. 159. а) а = 0. а = ±1; б) устойчивы. 160. а) а = 0, а = 1; б) для а = 1 устойчивы, для а = 0 нет. 161. a) ad<bc; б) ad>bc, (а — d)2 + 46с>0. 162. с>0, (а — — d)2 + 46с<0. 163. (0, 0) узел. 164. (0, 0) фокус. 165. (3, 1) вырожденный узел. 166. а<0. 167. а) нет; б) нет. 168. а) фокус; б) |а—6|<2 169. а) нет; б) узел; в) а = — 1. 170. а) всегда неустойчиво; б) |а|>1 седло, |а|<1 узел. 171. а) а< — 1 и —1<а<0 асимптотически устойчиво, устойчиво; б) а< — 1 и — 1<а^ — 1/2 узел (при а = —1/2 вырожденный); —1/2<а<0 и а>0 фокус; а = 0 центр. 172. а) а< — 1 асимптотически устойчиво, — 1 устойчиво; б) — 1<а<0 седло, а< — 1 и а>8 узел, 0<а<8 фокус. 173. а) а < —1 и а > 3 асимптотически устойчиво, а — 1 и а 3 устойчиво; б) — 1 < а < 3 седло, 1 — \/b<a< — 1 и 3<а<1 + у/5 узел, а<1 — у/5 и а>1 + + у/b фокус. 174. ху = С(х2 + у2)2. 175. Все решения определены при — оо<£<оо. 176. Нет. 177. а) (0, 0), (±1, 0); б) а<0; в) нет. 178. а) у = ±2жд/ж — 1; в) х = cos-2 t. 179. б) (0, 0) устойчиво, (±1/\/2, 0) неустойчивы; в) fci,2 = ±2, Т^тгд/2: г) (0, 0) устойчивый фокус (0<а<\/8), устойчивый узел (а>л/8), (±1/д/2, 0) седла. 180. б) (0, 0) неустойчиво, (±1, 0) устойчивы; в) fci, 2 = ±д/2, Т~тг; г) (0,0) седло, (±1, 0) устойчивые фокусы (0<а<4), устойчивые узлы (а>4). 181. а) (0, 0) центр, (1, 0) седло; б) траектория у = — (ж — — 1)д/(2ж -|- 1)/3, решение х = |(cth£)2 — | (0<£<оо); в) |а|<1/д/3; г) — 1/2<ж<1, 3?/2<2ж3 — Зж2 + 1; д) нет. 182. б) (±1, 0) неустойчивы; в) нет. 183. б) (0, 0) неустойчиво, (=Ы, 0) асимптотически устойчивы; в) нет. 184. а) (0, 0) и все точки окружности ж2 + у2 = = 1; б) в (0, 0) х = у, у = 0; в (1, 0) й = 0, v = 2м; в (1/^2, 1/^2) й = — и — v, v = (u + v)/2-, в) неустойчиво, неустойчиво, устойчиво: г) устойчиво, неустойчиво, устойчиво; е) нет: ж) x(to) = хо, «/(to) = = уо, где to, яго, Уо любые такие, что х% + 2«/о<1 или Xq + 2«/о>2. 186. (2х2-12/у/х)/5. 187. (2 + е-1)ж - хе~х. 188. (х - 1) е2ж+2-еж. 189. Зеж -х-1. 190. 1 + t. 191. chi. 192. sinf — t cost. 193. 2e2*-—2 — t — t2. 194. dx/dy = 6 + 2 ch2£ —12 ch£, dy/dy = 4sh2£-12sht 197. ex +ж-2. 198. esina;. 199. дх/дуъ = sinf, ду/дуъ = cosf + sinf.
174
Ответы к добавлению
200. 0. 201. у = ®1/2 + /Д 2ж2 - ЗяГ1/2) + /?(|ж7/2 - 4х + 8х~1/2-— |ж-3/2) + О(/х3). 202. х = 1 + sin2£ + /х2(| cos2i — cos4i— — ^) + О(р,3). 208. z2 = 2ху — |ж3 + f(x2 — 2у). 209. xz — у2 = = 1. 210. (z — ж2)2 + x(z — ж2) = у. 211. z(e~x +2ж — 2) = у2.
(\ 6	4	2	2	2
#) +^-V-?/+ V =0. 213. (х2-?/2)1п^ =?/2 + + 2(ж2 - у2) In |?/| — 2 In |^|. 214. (2ж 4- l)z = ж(2ж 4- 1)2/2 4- 2ху. 215. z = (1п|ж — у\ 4- 1)(ж 4- у). 216. (k — 2)z = 5(&ж — у) (А^2). 217. a) z = 2ж - 2 + (у - Зх + З)2; б) z = 2ж + f (у - Зх). feC1 произвольная функция с f (0) = 0. 218. При а ф 0, а ф —1, z = = (2ж + (4 — 2а)у)/а. при а = —1 z = ж 4- f(x-\-2y). fEC1 произвольная функция с f (0) = 0; при а = 0 решений нет. 219. а) решений нет; б) единственное решение z = —1 + д/2ж2 + 2у2 — 1. 220. а) нет; б) да. 221. </?(ж)=</?(—ж).
ТАБЛИЦЫ ПОКАЗАТЕЛЬНОЙ ФУНКЦИИ И ЛОГАРИФМОВ
Таблица!	Таблица 2
X	еж	X	In ж	1g X
0,01	1,000	1,0	0,000	0,000
0,05	1,051	1,1	0,095	0,041
0,10	1,105	1,2	0,182	0,079
0,15	1,162	1,3	0,262	0,114
0,20	1,221	1,4	0,336	0,146
0,25	1,284	1,5	0,405	0,176
0,30	1,350	1,6	0,470	0,204
0,35	1,419	1,7	0,531	0,230
0,40	1,492	1,8	0,588	0,255
0,45	1,568	1,9	0,642	0,279
0,50	1,649	2,0	0,693	0,301
X	еж	X	In ж	1g X
-3	0,050	3	1,099	0,477
—2	0,135	4	1,386	0,602
-1	0,368	5	1,609	0,699
0	1,000	6	1,792	0,778
1	2,718	7	1,946	0,845
2	7,389	8	2,079	0,903
3	20,09	9	2,197	0,954
4	54,60	10	2,303	1,000
5	148,4	11	2,398	1,041
7Г	23,14	20	2,996	1,301
27Г	535,5	100	4,605	2,000
Для отыскания значений функций при промежуточных значениях аргумента в таблице 1 можно производить линейную интерполяцию.
Филиппов Алексей Федорович
СБОРНИК ЗАДАЧ ПО ДИФФЕРЕНЦИАЛЬНЫМ УРАВНЕНИЯМ
Дизайнер М. В. Ботя Технический редактор А. В. Широбоков Корректор М. А. Ложкина
Подписано к печати 28.07.00. Формат 84 х 1081/32.
Усл. печ. л. 9,24. Уч. изд. л. 6,76.
Гарнитура Computer Modern Roman. Бумага офсетная № 1.
Печать офсетная. Тираж 1200 экз. Заказ №
Научно-издательский центр «Регулярная и хаотическая динамика» 426057, г. Ижевск, ул. Пастухова, 13.
Лицензия на издательскую деятельность ЛУ № 084 от 03.04.00. http://www.rcd.com.ru. E-mail: borisov@uni.udm.ru.
Отпечатано в полном соответствии с качеством предоставленных диапозитивов в ГИПП «Вятка». 610033, г. Киров, ул. Московская, 122.